Sei sulla pagina 1di 98

Chapter 9

Sequences and Series


9.1 Sequences
In this exercise set, the symbol
h
= is used to denote the fact that LHopitals Rule was applied to
obtain the equality.
1.
1
3
,
1
5
,
1
7
,
1
9
2.
3
2
,
1
2
,
3
10
,
3
14
3. 1,
1
2
,
1
3
,
1
4
4.
1
2
,
4
3
,
9
4
,
16
5
5. 10, 100, 1000, 10000
6.
1
10
,
1
100
,
1
1000
,
1
10000
7. 2, 4, 12, 48
8. 2, 24, 720, 40320
9. 1,
3
2
,
11
6
,
25
12
10.
1
2
,
3
4
,
7
8
,
15
16
11. Let > 0. Then

1
n
0

< implies
1
n
< or n >
1

. Take N to be the smallest integer


greater than
1

.
570
9.1. SEQUENCES 571
12. Let > 0. Then

1
n
2
0

< implies
1
n
2
< or n >
1

. Take N to be the smallest integer


greater than
1

.
13. Let > 0. Then

n
n + 1
1

1
n + 1

< implies
1
n + 1
< or n >
1

1. Take N to be
the smallest integer greater than
1

1.
14. Let > 0. Then

e
n
+ 1
e
n
1

1
e
n

< implies
1
e
n
< or n > ln
1

. Take N to be the
smallest integer greater than ln
1

.
15. lim
n
10

n + 1
= 0
16. lim
n
1
n
3/2
= 0
17. lim
n
1
5n + 6
= 0
18. lim
n
4
2n + 7
= 0
19. lim
n
3n 2
6n + 1
= lim
n
3 2/n
6 + 1/n
=
1
2
20. lim
n
n
1 2n
= lim
n
1
1/n 2
=
1
2
21. The terms alternate between 20 and 20. The sequence diverges.
22. lim
n
_

1
3
_
n
= lim
n
(1)
n
3
n
= 0
23. lim
n
n
2
1
2n
= lim
n
n 1/n
2
= . The sequence diverges.
24. lim
n
7n
n
2
+ 1
= lim
n
7
n + 1/n
= 0
25. lim
n
ne
n
= lim
n
n
e
n
h
= lim
n
1
e
n
= 0
26. lim
n
n
3
e
n
= lim
n
n
3
e
n
h
= lim
n
3n
2
e
n
h
= lim
n
6n
e
n
h
= lim
n
6
e
n
= 0
27. lim
n

n + 1
n
= lim
n
_
1 + 1/n

n
= 0
572 CHAPTER 9. SEQUENCES AND SERIES
28. lim
n
n

n + 1
= lim
n

n
_
1 + 1/n
= . The sequence diverges.
29. Since the terms alternate between 1 and 1, the sequence diverges.
30. Since sinn = 0 for all integers n, the sequence converges to 0.
31. lim
n
lnn
n
h
= lim
n
1/n
1
= 0
32. lim
n
e
n
ln(n + 1)
h
= lim
n
e
n
1/(n + 1)
= lim
n
e
n
(n + 1) = . The sequence diverges.
33. lim
n
5 2
n
7 + 4
n
=
5
7
34. lim
n
2
n
3
n
+ 1
= lim
n
1
(3/2)
n
+ 1/2
n
= 0
35. lim
n
e
n
+ 1
e
n
= lim
n
1 + 1/e
n
1
= 1
36. lim
n
_
4 +
3
n
2
n
_
= lim
n
_
4 +
_
3
2
_
n
_
= . The sequence diverges.
37. lim
n
nsin
_
6
n
_
= lim
n
sin(6/n)
1/n
h
= lim
n
[cos(6/n)](6/n
2
)
1/n
2
= lim
n
6 cos
_
6
n
_
= 6
38. Let y = (1 5/x)
x
. Then lny = xln(1 5/x) and
lim
x
lny = lim
x
ln(1 5/x)
1/x
h
= lim
x
5/x
2
1 5/x
1/x
2
= lim
x

5
1 5/x
= 5.
Thus, lim
n
(1 5/n)
n
= e
5
.
39. lim
n
e
n
e
n
e
n
+ e
n
= lim
n
1 e
2n
1 + e
2n
= 1
40. lim
n
_

4
arctann
_
=

4


2
=

4
41. Let y = x
2/(x+1)
. Then lny =
2
x + 1
lnx and lim
x
2 lnx
x + 1
h
= lim
x
2/x
1
= 0. Thus,
lim
n
n
2/(n+1)
= e
0
= 1.
42. lim
n
10
(n+1)/n
= lim
n
10
(1+1/n)
= 10
43. lim
n
ln
_
4n + 1
3n 1
_
= ln
_
lim
n
4 + 1/n
3 1/n
_
= ln
4
3
9.1. SEQUENCES 573
44. lim
n
lnn
ln3n
h
= lim
n
1/n
1/n
= lim
n
1 = 1
45. lim
n
_
n + 1

n
_
= lim
n
_
_
n + 1

n
_
_
n + 1 +

n + 1 +

n
__
= lim
n
n + 1 n

n + 1 +

n
= lim
n
1

n + 1 +

n
= 0
46. lim
n

n
_
n + 1

n
_
= lim
n
_

n
_
n + 1

n
_
_
n + 1 +

n + 1 +

n
__
= lim
n

n(n + 1 n)

n + 1 +

n
= lim
n
1
_
1 + 1/n + 1
=
1
2
47.
_
2n
2n 1
_
lim
n
2n
2n 1
= lim
n
2n 1 + 1
2n 1
= lim
n
_
1 +
1
2n 1
_
= 1
48.
_
1
n
+
1
n + 1
_
lim
n
_
1
n
+
1
n + 1
_
= 0
49.
_
(1)
n+1
(2n + 1)
_
Since the terms alternate between values that are increasingly greater than and less than 0,
the sequence diverges.
50. {(1)
n
2}
Since the terms alternate between 2 and 2, the sequence diverges.
51.
_
2
3
n1
_
lim
n
2
3
n1
= 0
52.
_
1
n2
n+1
_
lim
n
1
n2
n+1
= 0
53.
1
2
,
1
4
,
1
8
,
1
16
,
54. 3, 5, 9, 17,
55. 3, 1,
1
3
,
1
3
,
56. 2, 8, 22, 20,
57. a
n+1
=
1
4
a
n
+ 6 = lim
n
a
n+1
=
1
4
lim
n
a
n
+ 6 = L =
1
4
L + 6 = L = 8
58. a
n+1
=
1
2
_
a
n
+
5
a
n
_
= lim
n
a
n+1
=
1
2
_
lim
n
a
n
+
5
lim
n
a
n
_
= L =
1
2
_
L +
5
L
_
= L =

5
574 CHAPTER 9. SEQUENCES AND SERIES
59. For a
n
=
5
n
n!
, we have a
n+1
=
5
n+1
(n + 1)!
. Expanding, we get a
n+1
=
5 5
n
(n + 1) n!
=
5
n + 1
_
5
n
n!
_
.
The last factor of the last term is a
n
, so a
n+1
=
5
n + 1
a
n
.
60. Starting with a
1
=

3, we are given a
2
=

3 + a
1
, a
3
=

3 + a
2
, and so on. Thus, the
recursion formula is a
n+1
=

3 + a
n
.
61. Let a
n
= 0, b
n
=
sin
2
n
4
n
, and c
n
=
1
4
n
. Then lim
n
a
n
= lim
n
c
n
= 0, so lim
n
sin
2
n
4
n
= 0.
62. Let a
n
= 4, b
n
=
_
16 +
1
n
2
, and c
n
= 4 +
1
n
. [To see that b
n
c
n
, note that for x, y 0,
x
2
+ y
2
x
2
+ 2xy + y
2
= (x + y)
2
. Thus,
_
x
2
+ y
2
x + y.] Then lim
n
a
n
= lim
n
c
n
= 4,
so lim
n
_
16 +
1
n
2
= 4.
63. Let a
n
= 0, b
n
=
lnn
n(n + 2)
, and c
n
=
n
n(n + 2)
=
1
n + 2
. Then, for n 1, a
n
b
n
c
n
.
Since lim
n
a
n
= lim
n
c
n
= 0, we have lim
n
lnn
n(n + 2)
= 0.
64. Let a
n
= 0, b
n
=
n!
n
n
=
1
n
_
2
n

3
n

4
n

n
n
_
, and c
n
=
1
n
. [To see that b
n
c
n
, note that
1
n
1 and for any y 1,
1
n
y 1.] Then lim
n
a
n
= lim
n
c
n
= 0, so lim
n
n!
n
n
= 0.
65. Let y =
_
1 +
x
t
_
t
. Then lny = t ln
_
1 +
x
t
_
and using LHopitals Rule,
lim
t
lny = lim
t
ln(1 + x/t)
1/t
= lim
t
x/t
2
1 + x/t
1/t
2
= lim
t
x
1 + x/t
= x.
Thus, lim
n
_
1 +
x
n
_
n
= e
x
.
66. 1, 0.80685, 0.73472, 0.69704, 0.67390, 0.65824, 0.64695, 0.63842, 0.63174, 0.62638
(To compute the terms of this sequence, use the fact that it can be recursively dened by
a
n+1
= a
n
+ lnn +
1
n + 1
ln(n + 1), with a
1
= 1.)
67. Let a
n
be the height of the ball on the n-th bounce. Then a
0
= 15, a
1
=
2
3
(15) = 10,
a
2
=
2
3
(10) =
20
3
, a
3
=
2
3
_
20
3
_
=
40
9
ft, a
n
= 15
_
2
3
_
n
.
68. Let a
n
be the distance travelled during the n-th second. Then a
1
= 16, a
2
= 48 = 16(3),
a
3
= 80 = 16(5), a
4
= 16(7) = 112, a
5
= 16(9) = 144, a
6
= 16(11) = 176 ft.
9.1. SEQUENCES 575
69. A
1
= 15, A
2
= 15(0.2) + 15 = 18, A
3
= 18(0.2) + 15 = 18.6, A
4
= 18.6(0.2) + 15 = 18.72,
A
5
= 18.72(0.2) + 15 = 18.744, A
6
= 18.744(0.2) + 15 = 18.7488
70. First year: 1 + 1r = 1 + r; second year: (1 + r) + (1 + r)r = (1 + r)
2
; third year: (1 + r)
2
+
(1 + r
2
)r = (1 + r)
3
71. Parents: 2; grandparents: 2 2 = 4; great-grandparents: 2 4 = 8; great-great-grandparents:
2 8 = 16; great-great-great-grandparents: 2 16 = 32
72. lim
n
p
n+1
= 3 lim
n
p
n

1
400
lim
n
p
2
n
, so K = 3K
1
400
K
2
. Solving for K, we get
2K
1
400
K
2
= 0 = K
_
2
1
400
K
_
= 0 = K = 800.
With p
0
= 450, we have p
1
= 844, p
2
= 751, p
3
= 843, p
4
= 753, p
5
= 842, p
6
= 754,
p
7
= 841, p
8
= 755, p
9
= 840.
73. (a) a
n+1
= 1 +
1
1 + a
n
(b) a
5
= 1 +
1
2 +
1
2 +
1
2 +
1
2
a
6
= 1 +
1
2 +
1
2 +
1
2 +
1
2 +
1
2
(c) Letting L = lim
n
a
n
= lim
n
a
n+1
, we have L = 1 +
1
1 + L
, L
2
+ L = 1 + L + 1, and
L
2
= 2. Thus, L =

2 = lim
n
a
n
, and the sequence converges to

2.
74. a
1
1.7321, a
2
2.2795, a
3
2.6151, a
4
2.8009, a
5
2.8988, a
6
2.9489, a
7
2.9744,
a
8
2.9872, a
9
2.9936, a
10
2.9968, . . .
The sequence appears to converge to 3.
75. Since {a
n
} converges, then lim
n
a
n
is some value L. Since the limit of a product is the same as
the product of the factors limits, we have lim
n
a
2
n
= lim
n
(a
n
a
n
) = ( lim
n
a
n
)( lim
n
a
n
) = L
2
.
Thus,
_
a
2
n
_
actually does converge.
76. (a) The area of each of the four triangles between the (n1)st and n-th squares is
1
8
A
n
1.
Thus, the area of the n-th square is A
n1
4
_
1
8
_
A
n1
=
1
2
A
n1
. That is, A
1
= 1,
A
2
=
1
2
, A
3
=
1
4
, A
4
=
1
8
, . . . , and so on. In general, A
n
=
1
2
n1
.
(b) S
1
= 1, S
2
= 1 +
1
2
=
3
2
, S
3
=
3
2
+
1
4
=
7
4
, S
4
=
7
4
+
1
8
=
15
8
, S
5
=
15
8
+
1
16
=
31
16
,
S
6
=
31
16
+
1
32
=
63
32
, S
7
=
63
32
+
1
64
=
127
64
, S
8
=
127
64
+
1
128
=
255
128
, S
9
=
255
128
+
1
256
=
511
256
,
S
10
=
511
256
+
1
512
=
1023
512
576 CHAPTER 9. SEQUENCES AND SERIES
(c) The S
n
s appear to converge to 2.
77. (a) P
1
= 3, P
2
= 3
_
4
3
_
, P
3
= 3
_
4
3
_
2
, P
4
= 3
_
4
3
_
3
(b) P
n
= 3
_
4
3
_
n1
(c) lim
n
P
n
= lim
n
_
3
_
4
3
_
n1
_
=
78.
end of month 8 9 10 11 12
adult pairs 34 55 89 144 233
baby pairs 21 34 55 89 144
total pairs 55 89 144 233 377
In the bottom three rows of the table, each number (after the second one) is the sum of the
two preceding numbers in that row.
79. 2, 3, 5, 8, 13,
The recursion formula matches the pattern in Problem 78. Not surprisingly, the resulting
sequence is called the Fibonacci sequence.
80. (a) a
n
= 1 +
1
a
n1
= lim
n
a
n
= 1 +
1
lim
n
a
n1
= = 1 +
1

=
2
1 = 0
By the quadratic formula, the solutions are
(1)
_
(1)
2
4(1)(1)
2(1)
=
1

5
2
.
The limit must be positive, so =
1 +

5
2
.
(b) Since this portion of the exercise involves a research report, no solution is given. The
relationship between and the shape of the multi-chambered nautilus shell lies in how
the shells successive spirals grow at a rate that approximates for every quarter turn
that they make.
9.2 Monotonic Sequences
1. a
n+1
a
n
=
n + 1
3n + 4

n
3n + 1
=
1
(3n + 4)(3n + 1)
> 0. The sequence is increasing.
2. a
n+1
a
n
=
11 + n
n + 1

10 + n
n
=
10
n(n + 1)
< 0. The sequence is decreasing.
3. a
1
= 1, a
2
=

2, a
3
=

3. The sequence is not monotonic.


9.2. MONOTONIC SEQUENCES 577
4. a
1
= 0, a
2
= 0. Let f(x) = (x 1)(x 2) = x
2
3x + 2. Then f

(x) = 2x 3 > 0 for x 2.


The sequence is nondecreasing.
5. Let f(x) =
e
x
x
. Then f

(x) =
(x 1)e
x
x
2
> 0 for x > 1. The sequence is increasing.
6. Let f(x) =
e
x
x
5
. Then f

(x) =
(x 5)e
x
x
6
. Since f

(x) < 0 for x < 5 and f

(x) > 0 for x > 5,


the sequence is not monotonic.
7. a
1
= 2, a
2
= 2,
a
n+1
a
n
=
2
n+1
/(n + 1)!
2
n
/n!
=
2
n + 1
< 1 for n > 1.
The sequence is nonincreasing.
8.
a
n+1
a
n
=
2
2n+2
[(n + 1)!]
2
/(2n + 2)!
2
2n
(n!)
2
/(2n)!
=
4(n + 1)
2
(2n + 2)(2n + 1)
=
4n
2
+ 8n + 4
4n
2
+ 6n + 2
> 1
The sequence is increasing.
9. Let f(x) = x +
1
x
. Then f

(x) = 1
1
x
2
> 0 for x > 1. The sequence is increasing.
10. a
1
= 0, a
2
= 6, a
3
= 6
a
n+1
a
n
= (n + 1)
2
+ (1)
n+1
(n + 1) n
2
(1)
n
n
= n
2
+ 2n + 1 (1)
n
n (1)
n
n
2
(1)
n
n
= 2n + 1 2n(1)
n
(1)
n
= (2n + 1)[1 (1)
n
] 0
The sequence is nondecreasing.
11. Since n < for n = 1, 2, 3, we have sinn > 0 for n = 1, 2, 3. Since < n < 2 for n = 4, 5, we
have sinn < 0 for n = 4, 5. Thus, a
3
> 0, a
4
< 0, and a
5
> 0. The sequence is not monotonic.
12. a
n+1
a
n
= ln
_
n + 3
n + 2
_
ln
_
n + 2
n + 1
_
= ln
_
(n + 3)(n + 1)
(n + 2)
2
_
= ln
_
n
2
+ 4n + 3
n
2
+ 4n + 4
_
Since
n
2
+ 4n + 3
n
2
+ 4n + 4
< 1, ln
_
n
2
+ 4n + 3
n
2
+ 4n + 4
_
< 0 and the sequence is decreasing.
13. Since a
n+1
a
n
=
4n + 3
5n + 7

4n 1
5n + 2
=
13
(5n + 7)(5n + 2)
> 0, the sequence is monotonic.
Using
4n 1
5n + 2
> 0 and
4n 1
5n + 2
<
4n
5n + 2
<
4n
5n
=
4
5
, we see that the sequence is bounded.
Thus, the sequence converges.
14. Since a
n+1
a
n
=
6 4(n + 1)
2
1 + (n + 1)
2

6 4n
2
1 + n
2
=
10[n
2
(n + 1)
2
]
[1 + (n + 1)
2
][1 + n
2
]
< 0, the sequence is
monotonic. Using
6 4n
2
1 + n
2
< 6 and
6 4n
2
1 + n
2
>
4 4n
2
1 + n
2
=
4(1 + n
2
)
1 + n
2
= 4, we see that
the sequence is bounded. Thus, the sequence converges.
578 CHAPTER 9. SEQUENCES AND SERIES
15. Since
a
n+1
a
n
=
3
n+1
/(1 + 3
n+1
)
3
n
/(1 + 3
n
)
=
3 + 3
n+1
1 + 3
n+1
= 1 +
2
1 + 3
n+1
> 1, the sequence is monotonic.
Using
3
n
1 + 3
n
> 0 and
3
n
1 + 3
n
= 1
1
1 + 3
n
< 1, we see that the sequence is bounded. Thus,
the sequence converges.
16. Since
a
n+1
a
n
=
(n + 1)/5
n+1
n/5
n
=
n + 1
5n
=
1
5
_
1 +
1
n
_
< 1, the sequence is monotonic. Using
n
5
n
> 0 and
n
5
n

1
5
(since the sequence is decreasing), we see that the sequence is bounded.
Thus, the sequence converges.
17. Let f(x) = e
1/x
. Then f

(x) =
e
1/x
x
2
< 0 and the sequence is monotonic. Using e
1/n
> 0
and e
1/n
e (since the sequence is decreasing), we see that the sequence is bounded. Thus,
the sequence converges.
18. Since
a
n+1
a
n
=
(n + 1)!/(n + 1)
n+1
n!/n
n
=
(n + 1)n
n
(n + 1)
n+1
=
_
n
n + 1
_
n
< 1, the sequence is mono-
tonic. Using
n!
n
n
> 0 and
n!
n
n
1 (since the sequence is decreasing), we see that the sequence
is bounded. Thus, the sequence converges.
19. Since
a
n+1
a
n
=
(n + 1)!/[1 3 5 (2n + 1)]
n!/[1 3 5 (2n 1)]
=
n + 1
2n + 1
< 1, the sequence is monotonic. Using
n!
1 3 5 (2n 1)
> 0 and
n!
1 3 5 (2n 1)
1 (since the sequence is decreasing), we see
that the sequence is bounded. Thus, the sequence converges.
20. Since
a
n+1
a
n
=
[2 4 6 (2n + 2)]/[1 3 5 (2n + 3)]
[2 4 6 2n]/[1 3 5 (2n + 1)]
=
2n + 2
2n + 3
< 1, the sequence is mono-
tonic. Using
2 4 6 2n
1 3 5 (2n + 1)
> 0 and
2 4 6 2n
1 3 5 (2n + 1)

2
3
(since the sequence is
decreasing), we see that the sequence is bounded. Thus, the sequence converges.
21. Let f(x) = tan
1
x. Then f

(x) =
1
1 + x
2
> 0 and the sequence is monotonic. Since
| tan
1
n| <

2
, we see that the sequence is bounded. Thus, the sequence converges.
22. Let f(x) =
ln(x + 3)
x + 3
. Then f

(x) =
1 ln(x + 3)
(x + 3)
2
< 0 for x 0 and the sequence is
monotonic. Using
ln(n + 3)
n + 3
> 0 and
ln(n + 3)
n + 3
<
ln4
4
(since the sequence is decreasing), we
see that the sequence is bounded. Thus, the sequence converges.
23. The sequence is {(0.8)
n
}. Since
a
n+1
a
n
=
(0.8)
n+1
(0.8)
n
= 0.8 < 1, the sequence is monotonic.
Using (0.8)
n
> 0 and (0.8)
n
0.8 (since the sequence is decreasing), we see that the sequence
is bounded. Thus, the sequence converges.
9.2. MONOTONIC SEQUENCES 579
24. The sequence is
_
3
1/2
n
_
. When x > 1,

x > 1, so
a
n+1
a
n
=
3
1/2
n+1
3
1/2
n
=
1
3
1/2
n+1
< 1 and the
sequence is monotonic. Using 3
1/2
n
> 0 and 3
1/2
n

3 (since the sequence is decreasing),


we see that the sequence is bounded. Thus, the sequence converges.
25. a
n+1
=
1
2
a
n
+ 5, a
1
= 1. We will show that a
n
< 10 for all n.
For n = 1, we have a
2
=
11
2
< 10. Assume that a
k
< 10. Then a
k+1
=
1
2
a
k
+5 <
1
2
(10) +5 =
10; that is, a
k+1
< 10 whenever a
k
< 10. The sequence is bounded because 0 < a
n
< 10.
Next, we will show that the sequence {a
n
} is monotonic. Because a
n
< 10, necessarily
1
2
a
n
<
1
2
10 = 5. Therefore, from the recursion formula,
a
n+1
=
1
2
a
n
+ 5 >
1
2
a
n
+
1
2
a
n
= a
n
.
This shows that a
n+1
> a
n
for all n, and so the sequence is increasing.
Since {a
n
} is bounded and monotonic, it follows from Theorem 9.2.1 that the sequence con-
verges. Because we must have lim
n
a
n
= L and lim
n
a
n+1
= L, the limit of the sequence can
be determined from the recursion formula:
lim
n
a
n+1
=
1
2
lim
n
a
n
+ 5 = L =
1
2
L + 5 = L = 10.
26. a
n+1
=

2 + a
n
, a
1
= 0. We will show that a
n
< 2 for all n.
For n = 1, we have a
2
=

2 < 2. Assume that a


k
< 2. Then a
k+1
=

2 + a
k
<

2 + 2 = 2;
that is, a
k+1
< 2 whenever a
k
< 2. The sequence is bounded because 0 < a
n
< 2.
Also, the sequence is increasing:
a
n+1
=

2 + a
n
>

a
n
+ a
n
=

2a
n
>

a
n
a
n
= a
n
.
That is, a
n+1
> a
n
.
By Theorem 9.2.1, the sequence is bounded and monotonic, and so it is convergent. From
lim
n
a
n+1
=
1
2
lim
n

2 + a
n
= L =

2 + L = (L 2)(L + 1) = 0,
we have L = 2.
27. a
n+1
=

7a
n
, a
1
=

7. Now 0 < a
n
< 7 =

a
n
<

7, and so
a
n+1
=

7a
n
=

a
n
>

a
n

a
n
= a
n
.
Thus, a
n+1
> a
n
for all n. The sequence is therefore monotonic (increasing) and bounded.
By Theorem 9.2.1, the sequence converges. From
lim
n
a
n+1
= lim
n

7a
n
= L =

7L = L
2
= 7L = L(L 7) = 0,
we have L = 7.
580 CHAPTER 9. SEQUENCES AND SERIES
28. Since
a
n+1
a
n
= 1
1
n
2
< 1 for n 2, the sequence is monotonic. Using a
n
> 0 and a
n
2
(since the sequence is decreasing), we see that the sequence is bounded. Thus, the sequence
converges. Taking the limit of the recursion formula shows nothing, since
lim
n
a
n+1
= lim
n
_
1
1
n
2
_
a
n
= lim
n
a
n
= L = L.
29. (a) If lim
n
p
n
= L, then lim
n
p
n+1
= L and lim
n
bp
n
a + p
n
=
bL
a + L
. Thus, L =
bL
a + L
or
(a + L b)L = 0 and L = 0 or L = b a.
(b) Since p
n
> 0, p
n+1
=
bp
n
a + p
n
=
b
a + p
n
p
n
<
b
a
p
n
.
(c) If a > b then
b
a
< 1 and by part (b), p
n+1
< p
n
. The sequence is thus monotonically
decreasing. Now p
1
<
b
a
p
0
implies p
2
<
b
a
p
1
<
b
a
_
b
a
p
0
_
=
_
b
a
_
2
p
0
, which in turn
implies p
3
<
b
a
p
2
<
b
a
_
_
b
a
_
2
p
0
_
=
_
b
a
_
3
p
0
. In general, p
n+1
<
_
b
a
_
n+1
p
0
. Since
b
a
< 1, lim
n
p
n
= lim
n
p
n+1
lim
n
_
b
a
_
n+1
p
0
= 0. Since p
n
> 0 for all n, lim
n
p
n
= 0.
(d) We rst note that
|b a p
n+1
| =

b a
bp
n
a + p
n

ab + bp
n
a
2
ap
n
bp
n
a + p
n

=
a
a + p
n
|b a p
n
|.
Since 0 <
a
a + p
n
< 1, the distance from p
n+1
to b a is less than the distance from p
n
to b a. This means that p
n+1
is between b a and p
n
. Thus, if 0 < p
0
< b a, the
sequence {p
n
} is increasing and bounded above by b a. If 0 < b a < p
0
, the sequence
{p
n
} is decreasing and bounded below by b a. In either case, it follows from part (a)
that the sequence converges to b a.
30. {(1)
n
} is bounded but not convergent.
31. Since {a
n
} is convergent, it follows from Denition 9.1.2 that there exists an N such that
|a
n
L| < 1 whenever n > N. Adding |L| to both sides, we have |a
n
L| +|L| < 1 +|L|. By
the triangle inequality, |(a
n
L)+L| = |a
n
| |a
n
L|+|L|, and so |a
n
| |a
n
L|+|L| < 1+|L|
for all n > N. For n N, we have a nite set of numbers that therefore has a maximum
value M and minimum value m; thus, {a
n
} is bounded.
32. Since a
n+1
a
n
=
_
n+1
1
e
t
2
dt
_
n
1
e
t
2
dt =
_
n+1
n
e
t
2
dt > 0, the series is monotonic.
9.3. SERIES 581
Using
_
n
1
e
t
2
dt 0 and
_
n
1
e
t
2
dt <
_

1
e
t
2
dt
_

1
e
t
dt = lim
k
_
k
1
e
t
dt = lim
k
(e
t
)
_
k
1
= lim
k
(e
1
e
k
) = e
1
,
we see that the sequence is bounded. Thus, the sequence converges.
33. Note that a
1
= 1 and assume n 2. Then the area under the graph of y =
1
x
on [1, n] is
A =
_
n
1
1
x
dx = lnx
_
n
1
= lnn.
Partitioning [1, n] at 1, 2, 3, . . . , n, the upper sum is U = 1 +
1
2
+ +
1
n 1
and the lower
sum is L =
1
2
+
1
3
+ +
1
n
. Since L < A < U, we have, for n 2,
1
2
+
1
3
+ +
1
n
< lnn <
1 +
1
2
+ +
1
n 1
. Now, a
n
= 1 +
1
2
+ +
1
n
lnn, so
1
2
+
1
3
+ +
1
n
= a
n
1 +lnn and
1 +
1
2
+ +
1
n 1
= a
n

1
n
+lnn. Thus, for n 2, a
n
1 +lnn < lnn < a
n

1
n
+lnn and
a
n
1 < 0 < a
n

1
n
, or a
n
< 1 and a
n
>
1
n
> 0. Since a
1
= 1, the sequence is bounded below
by 0 and above by 1. To see that the sequence is monotonic, note that a
n
1 + lnn < lnn
implies a
n+1
1 +ln(n +1) < ln(n +1). Subtracting, we have [a
n+1
1 +ln(n +1)] (a
n

1 +lnn) < ln(n +1) lnn or a


n+1
a
n
< 0. Since the sequence is bounded and monotonic,
it is convergent.
9.3 Series
1. 3 +
5
2
+
7
3
+
9
4
+
2. 2 + 2 +
8
3
+ 4 +
3.
1
2

1
6
+
1
12

1
20
+
4.
1
3

1
8
+
1
81

1
324
+
5. 1 + 2 +
3
2
+
2
3
+
6. 1 +
24
5
+ 72 +
40, 320
17
+
7. 2 +
8
3
+
16
5
+
128
35
+
582 CHAPTER 9. SEQUENCES AND SERIES
8. 1 +
3
2
+
5
2
+
35
8

9.
1
7
+
1
9

1
11
+
1
13

10. 5 + 0 7 + 0
11. Write a
k
=
1
k

1
k + 1
. Then S
n
=
_
1
1
2
_
+
_
1
2

1
3
_
+ +
_
1
n

1
n + 1
_
= 1
1
n + 1
and

k=1
1
k(k + 1)
= lim
n
S
n
= 1.
12. Write a
k
=
1
k + 1

1
k + 2
. Then
S
n
=
_
1
2

1
3
_
+
_
1
3

1
4
_
+ +
_
1
n + 1

1
n + 2
_
=
1
2

1
n + 2
and

k=1
1
(k + 1)(k + 2)
= lim
n
S
n
=
1
2
.
13. Write a
k
=
1/2
2k 1

1/2
2k + 1
. Then
S
n
=
1
2
__
1
1
3
_
+
_
1
3

1
5
_
+ +
_
1
2n 1

1
2n + 1
__
=
1
2
_
1
1
2n + 1
_
and

k=1
1
4k
2
1
= lim
n
S
n
=
1
2
.
14. Write a
k
=
1
(k + 3)(k + 4)
=
1
k + 3

1
k + 4
. Then
S
n
=
_
1
4

1
5
_
+
_
1
5

1
6
_
+ +
_
1
n + 3

1
n + 4
_
=
1
4

1
n + 4
and

k=1
1
k
2
+ 7k + 12
= lim
n
S
n
=
1
4
.
15. Identify r =
1
5
and a = 3. The series converges to
3
1 1/5
=
15
4
.
16. Identify r =
3
4
and a = 10. The series converges to
10
1 3/4
= 40.
17. Identify r =
1
2
and a = 1. The series converges to
1
1 + 1/2
=
2
3
.
18.

k=1

k
_
1
3
_
k1
=

k=1

3
_
k1
. Identify a = and r =

3
> 1. The series diverges.
9.3. SERIES 583
19. The common ratio is
5
4
> 1. The series diverges.
20. Identify t =
3
7
and a =
3
7
. The series converges to
3/7
1 + 3/7
=
3
10
.
21. Identify r = 0.9 and a = 900. The series converges to
900
1 0.9
= 9000.
22. Identify r = 1.1 > 1. The series diverges.
23. Identify r =
1

2
> 1. The series diverges.
24. Identify r =

5
1 +

5
and a = 1. The series converges to
1
1

5/(1 +

5)
= 1 +

5.
25. 0.222 . . . = 0.2+0.02+0.002+ . Identify r = 0.1 and a = 0.2. Then 0.222 . . . =
0.2
1 0.1
=
2
9
.
26. 0.555 . . . = 0.5+0.05+0.005+ . Identify r = 0.1 and a = 0.5. Then 0.555 . . . =
0.5
1 0.1
=
5
9
.
27. 0.616161 . . . = 0.61 + 0.0061 + 0.000061 + . Identify r = 0.01 and a = 0.61. Then
0.616161 . . . =
0.61
1 0.01
=
61
99
.
28. 0.393939 . . . = 0.39 + 0.0039 + 0.000039 + . Identify r = 0.01 and a = 0.39. Then
0.393939 . . . =
0.39
1 0.01
=
39
99
=
13
33
.
29. 1.314314 . . . = 1 + (0.314 + 0.000314 + ). Identify r = 0.01 and a = 0.314. Then
1.314314 . . . = 1 +
0.314
1 0.001
= 1 +
314
999
=
1313
999
.
30. 0.5262626 . . . = 0.5 + (0.026 + 0.00026 + . Identify r = 0.01 and a = 0.026. Then
0.5262626 . . . =
5
10
+
0.26
1 0.01
=
5
10
+
26
990
=
521
990
.
31.

k=1
_
_
1
3
_
k1
+
_
1
4
_
k1
_
=

k=1
_
1
3
_
k1
+

k=1
_
1
4
_
k1
=
1
1 1/3
+
1
1 1/4
=
3
2
+
4
3
=
17
6
32.

k=1
2
k
1
4
k
=

k=1
_
1
2
_
k

k=1
_
1
4
_
k
=
1/2
1 1/2

1/4
1 1/4
= 1
1
3
=
2
3
33. lim
k
a
k
= lim
k
10 = 0, so the series diverges.
34. lim
k
a
k
= lim
k
(5k + 1) = , so the series diverges.
35. lim
k
a
k
= lim
k
k
2k + 1
= lim
k
1
2 + 1/k
=
1
2
= 0, so the series diverges.
584 CHAPTER 9. SEQUENCES AND SERIES
36. lim
k
a
k
= lim
k
k
2
+ 1
k
2
+ 2k + 3
= lim
k
1 + 1/k
2
1 + 2/k + 3/k
2
= 1 = 0, so the series diverges.
37. lim
k
a
k
= lim
k
(1)
k
does not exist, so the series diverges.
38. lim
k
a
k
= lim
k
ln
_
k
3k + 1
_
= ln
_
lim
k
k
3k + 1
_
= ln
_
lim
k
1
3 + 1/k
_
= ln
1
3
= 0, so the
series diverges.
39. 10

k=1
1
k
diverges because the harmonic series diverges. Thus

k=1
10
k
diverges.
40.
1
6

k=1
1
k
diverges because the harmonic series diverges. Thus,

k=1
1
6k
diverges.
41. Since

k=1
1
2
k1
is a geometric series with r =
1
2
, it converges. Since

k=1
1
k
is the harmonic
series, it diverges. Thus,

k=1
_
1
2
k1
+
1
k
_
diverges.
42. Let f(x) = xsin
1
x
. Then, using LHopitals Rule,
lim
x
f(x) = lim
x
sin(1/x)
1/x
h
= lim
x
(1/x
2
) cos(1/x)
1/x
2
= lim
x
cos
1
x
= 1 = 0,
so the series diverges.
43. This is a geometric series with r =
x
2
and will converge for

x
2

< 1 or |x| < 2.


44. This is a geometric series with r =
1
x
and will converge for

1
x

< 1 or |x| > 1.


45. This is a geometric series with r = x + 1 and will converge for |x + 1| < 1 or 2 < x < 0.
46. This is a geometric series with r = 2x
2
and will converge for |2x
2
| < 1 or |x| <

2
2
.
47. The total distance is 15 + 2(15)
_
2
3
_
+ 2(15)
_
2
3
_
2
+ = 15 +

k=1
30
_
2
3
_
k
. The sum is a
geometric series with a = 20 and r =
2
3
, so

k=1
30
_
2
3
_
k
=
20
1 2/3
= 60. The total distance
is 15 + 60 = 75 ft.
48. Using the formula in Example 6 of Section 9.3 in the text with s = 15, f =
2
3
, and g = 32,
we have T =
_
2(15)
32
_
1 +
_
2/3
1
_
2/3
_
9.58 s.
9.3. SERIES 585
49. N
0
+ N
0
s + N
0
s
2
+ =

k=0
N
0
s
k
=
N
0
1 s
. Solving 10, 000 =
N
0
1 0.9
, we obtain N
0
= 1000.
50. A
0
+ A
0
e
k
+ A
0
e
2k
+ =

n=0
A
0
(e
k
)
n
. This is a geometric series with a = A
0
and
r = e
k
. The sum is
A
0
1 e
k
=
A
0
e
k
e
k
1
.
51. The total amount of the drug immediately after the n-th dose is A
n
= 15+15(0.2)+15(0.2)
2
+
+ 15(0.2)
n1
. As n , the total accumulation of the drug will be
lim
n
A
n
= lim
n
n

k=1
15(0.2)
k1
=

k=1
15(0.2)
k1
=
15
1 0.2
=
75
4
= 18.75 mg.
52. The total distance is 20 + 10 + 5 + 5/2 + =

k=0
20
_
1
2
_
k
=
20
1 1/2
= 40 cm.
53. By Theorem 9.3.3 in the text, if lim
n
a
n
= 0, then the series

k=1
a
k
diverges.
54. For the series
1
1.1
+
1
1.11
+
1
1.111
+ ,
a
1
=
1
1 +
1
10
, a
2
=
1
1 +
1
10
+
1
10
2
, , a
n
=
1
1 +
1
10
+ +
1
10
n1
+
1
10
n
=
1
n

k=0
1
10
k
As n , a
n

1

k=0
1
10
k
=
1
10/9
=
9
10
= 0. The series diverges by the n-th term test.
55. The series

k=1
k and

k=1
(k) both diverge, but their sum

k=1
(k k) =

k=1
0 = 0 converges.
56. For S
n
=
1
1 1
+
1
2 2
+
1
3 3
+ +
1
n n
, the inequality 0 < S
n
< 1 +
1
1 2
+
1
2 3
+
+
1
(n 1) n
is true because, for all n 1, n > n 1, n n > (n 1) n, and therefore
1
n n
<
1
(n 1) n
. The second inequality is true because each individual term
1
(n 1) n
=
(n n) + 1
n(n 1)
=
n
n(n 1)

n 1
n(n 1)
=
1
n 1

1
n
. Simplifying this, we get
1 +
1
1 2
+
1
2 3
+ +
1
(n 1) n
= 1 +
_
1
1

1
2
_
+
_
1
2

1
3
_
+ +
_
1
n 1

1
n
_
= 1 + 1 +
_

1
2
+
1
2
_
+
_

1
3
+
1
3
_
+ +
_

1
n 1
+
1
n 1
_

1
n
= 2
1
n
.
586 CHAPTER 9. SEQUENCES AND SERIES
Thus, 0 < S
n
< 2
1
n
. Since n > 0, then 0 < S
n
< 2 for all n and so {S
n
} is bounded.
Because S
n
is a partial sum whose addends are all positive, {S
n
} is also monotonic. Thus, it
converges, and so

k=1
1
k
2
converges.
57.
1 + 9
25
+
1 + 27
125
+
1 + 81
625
+ =
1 + 3
2
5
2
+
1 + 3
3
5
3
+
1 + 3
4
5
4
+
=
1
5
2
+
_
3
5
_
2
+
1
5
3
+
_
3
5
_
3
+
1
5
4
+
_
3
5
_
4
+
=
1
5
2
+
1
5
3
+
1
5
4
+ +
_
3
5
_
2
+
_
3
5
_
3
+
=
__

k=1
1
5
k1
_
1
1
5
_
+
__

k=1
3
k1
5
k1
_
1
3
5
_
=

k=1
1
5
k1
+

k=1
_
3
5
_
k1
1 1
1
5

3
5
=

k=1
1
5
k1
+

k=1
_
3
5
_
k1
2
4
5
=
1
1 1/5
+
1
1 3/5
2
4
5
=
19
20
58. Integrating by parts, we obtain
_
xe
x
dx = xe
x
e
x
. Then

k=1
_
_
k+1
k
xe
x
dx
_
=

k=1
(xe
x
e
x
)
_
k+1
k
=

k=1
_
(k + 1)e
(k+1)
e
(k+1)
+ ke
k
+ e
k
_
=

k=1
_
(k + 1)e
k
(k + 2)e
(k+1)
_
and
S
n
= (2e
1
3e
2
) + (3e
2
4e
3
) + + [(n + 1)e
n
(n + 2)e
(n+1)
] = 2e
1

n + 2
e
n+1
.
Then, using LHopitals Rule,

k=1
_
_
k+1
k
xe
x
dx
_
= lim
n
S
n
= 2e
1
lim
n
n + 2
e
n+1
h
= 2e
1
lim
n
1
e
n+1
= 2e
1
.
59.
n

k=0
tan
k
x is a geometric series with a = 1 and r = tanx. The innite series

k=0
tan
k
x will
9.3. SERIES 587
converge to
1
1 tanx
when | tanx| < 1 or |x| <

4
. Thus, lim
n
_
1
1 tanx

n

k=0
tan
k
x
_
= 0
for |x| <

4
.
60. Suppose lim
n
f(n + 1) = L and let S
n
=
n

k=1
[f(k + 1) f(k)]. Then
S
n
= [f(2) f(1)] + [f(3) f(2)] + + [f(n + 1) f(n)] = f(n + 1) f(1)
and

k=1
[f(k + 1) f(k)] = lim
n
S
n
= lim
n
[f(n + 1) f(1)] = L f(1).
61. The general term of the series is a
n
=
n

k=1
1
k
. Since lim
n
a
n
=

k=1
1
k
and the harmonic series
diverges, then by Theorem 9.3.3, the series

n=1
a
n
=

n=1
_
n

k=1
1
k
_
diverges.
62. Since S
n
= 1 +
1

2
+
1

3
+ +
1

n

1

n
+
1

n
+ +
1

n
=
n

n
=

n, lim
n
S
n
=
and the series diverges.
63. (a) The partial sum 1 +
1
2
+
1
3
+
1
4
+ +
1
n
, n > 1 can be represented as an area. The
graph below shows an area of 1 +
1
2
on [0, 2], 1 +
1
2
+
1
3
on [0, 3], and so on:
1 2 3 4 5
1
In addition, the area under the graph of f(x) =
1
x
from x = 1 up to any n > 1 is
_
n
1
1
x
dx = lnx
_
n
1
= lnnln1 = lnn. Shifting the partial sum areas to the right allows
us to compare
_
n+1
1
1
x
dx = ln(n+1) with their corresponding partial sums. This shows
that ln(n + 1) < S
n
= 1 +
1
2
+
1
3
+
1
4
+ +
1
n
:
1 2 3 4 5
1
588 CHAPTER 9. SEQUENCES AND SERIES
Similarly, the graphs can be aligned to show that 1 +
_
n
1
1
x
dx = 1 + lnn > S
n
=
1 +
1
2
+
1
3
+
1
4
+ +
1
n
. Recall that n > 1 is stipulated, so the area on [0, 1] consists
solely of the 1 1 square (thus corresponding to the rst term in 1 + lnn):
1 2 3 4 5
1
This yields the overall inequality ln(n + 1) < 1 +
1
2
+
1
3
+
1
4
+ +
1
n
< 1 + lnn.
(b) Using the inequality in part (a), we know that S
n
10 when ln(n + 1) 10. Solving
ln(n+1) = 10, we get n+1 = e
10
, and n = e
10
1. Thus, S
n
10 for n > e
10
1 22026.
Using a calculator yields
22026

k=1
1
k
10.5772 (it should be noted that the smallest n for
which S
n
10 is 12367). Similarly, to estimate the value of n for which S
n
100, we
solve ln(n + 1) = 100, getting n = e
100
1 2.6811 10
43
(the smallest n for which
S
n
100 is approximately 1.509 10
43
).
64. (a) A
1
=

3
4
; A
2
=

3
4
+ 3
_

3
4

1
3
2
_
=

3
4
+

3
4
_
1
3
_
A
3
=

3
4
+

3
4
_
1
3
_
+ 12
_

3
4

1
9
2
_
=

3
4
+

3
4
_
1
3
_
+

3
4
_
1
3
__
4
9
_
A
4
=

3
4
+

3
4
_
1
3
_
+

3
4
_
1
3
__
4
9
_
+ 48
_

3
4

1
27
2
_
=

3
4
+

3
4
_
1
3
_
+

3
4
_
1
3
__
4
9
_
+

3
4
_
1
3
__
4
9
_
2
(b) A
n
=

3
4
+ 3
_

3
4
_
_
1
3
2
_
+ 12
_

3
4
_
_
1
9
2
_
+ 48
_

3
4
_
_
1
27
2
_
+
+ 3(4
n2
)
_

3
4
_
_
1
3
2n2
_
=

3
4
+

3
4
n1

k=1
3(4
k1
)
3
2k
=

3
4
_
1 +
n1

k=1
4
k1
3
2k1
_
=

3
4
_
1 +
n1

k=1
1
3
_
4
3
2
_
k1
_
=

3
4
_
1 +
1
3

1 (4/9)
n1
1 4/9
_
=

3
4
_
1 +
3
5
_
1
_
4
9
_
n1
__
=

3
4
_
8
5

3
5
_
4
9
_
n1
_
=

3
20
_
8 3
_
4
9
_
n1
_
9.3. SERIES 589
(c) lim
n
A
n
=

3
20
(8) =
2

3
5
65. (a) We observe L
1
= d, L
2
= d+L
1
pL
1
= d+d(1p), L
3
= d+L
2
pL
2
= d+(1p)L
2
=
d + d(1 p) + d(1 p)
2
. In general, L
n
= d + d(1 p) + d(1 p)
2
+ + d(1 p)
n1
.
This is a geometric series with rst term d and common ratio (1 p). Thus
L
n
=
d[1 (1 p)
n
]
1 (1 p)
=
d
p
[1 (1 p)
n
].
Since 0 < p < 1, lim
n
L
n
= d/p.
(b) With d = 1.4 and p = 0.009, lim
n
L
n
= 1.4/0.009 155.56 155.56 mg. To determine
when the various symptoms occur, we solve L
n
=
d
p
[1 (1 p)
n
] for n, obtaining
n =
ln(1 pL
n
/d)
ln(1 p)
. With d = 1.4 and p = 0.009, this becomes n =
ln(1 9L
n
/1400)
ln0.991
.
parasthesia : L
n
= 25, n 19.4
ataxia : L
n
= 55, n 48.3
dysarthia : L
n
= 90, n 95.6
Neither deafness nor death can occur at these values of d and p.
(c) Solving 200 =
d
0.009
[1 (1 0.009)
100
] for d, we obtain d 3.02 mg.
66. Suppose the tortoise starts 10 feet (120 inches) in front of Achilles, the tortoise travels at 1
inch per second, and Achilles runs at 5 feet (60 inches) per second. Assuming constant speeds,
conventional reasoning states that Achilles and the tortoise will have reached the same point
at some time t such that (1)t + 120 = 60t, 59t = 120, and t = 120/59 2.1864 seconds.
Achilles then passes the tortoise after approximately 2.1864 seconds.
As phrased by Zeno, it will take 2 seconds for Achilles to reach the tortoises starting point
of 120 inches. By this time, the tortoise will be 2 inches in front of Achilles. It will then take
Achilles 1/30 second to reach that point, by which time the tortoise will be 1/30 inches in
front of Achilles. The distance travelled by Achilles can thus be written as a geometric series:
120 + 2 + 1/30 + = 120 + 120(60
1
) + 120(60
2
) + + 120(60
1n
) +
In this example, we have a = 120 and r = 1/60; in general, a is the tortoises head start, and
r is the reciprocal of Achilless speed. |r| < 1, so the series converges. At a constant speed,
Achilles will reach this nite distance in a nite amount of time.
The issue with Zenos statement of the problem is that Achilles reaches the tortoises previous
position in less and less time innitely less, in fact. This innite division of time constitutes
the trick, so to speak, behind the paradox. Time in the story never exceeds the time that it
takes to reach the sum of the innite series, thus resulting in the tortoises apparent victory.
67. This exercise involves a research report, and thus a preset solution is not applicable. It
should be noted, however, that the series of the reciprocal of primes (i.e., the harmonic series
of primes) does diverge, with multiple proofs available in the literature.
590 CHAPTER 9. SEQUENCES AND SERIES
68. AP
1
+ P
1
P
2
+ P
2
P
3
+ P
3
P
4
+ P
4
P
5
+ P
5
P
6
+
=

2 + 1 +

2
2
+
1
2
+

2
4
+
1
4
+
=

2
_
1 +
1
2
+
1
2
2
+
_
+
_
1 +
1
2
+
1
2
2
+
_
= (1 +

2)
_
1 +
1
2
+
1
2
2
+
_
= (1 +

2)
_
1
1 1/2
_
= 2 + 2

2
69. AP
1
+ P
1
P
2
+ P
2
P
3
+ P
3
P
4
+ P
4
P
5
+ P
5
P
6
+
= sin30

+ (cos 30

) sin30

+ (cos 30

)
2
sin30

+ (cos 30

)
3
sin30

+
=
1
2
+
_

3
2
_
1
2
+
_

3
2
_
2
1
2
+
_

3
2
_
3
1
2
+
=
1
2
_
_
1 +

3
2
+
_

3
2
_
2
+
_

3
2
_
3
+
_
_
=
1
2
_
1
1

3/2
_
=
1
2

3
= 2 +

3
70. The function f is nonzero on the intervals
(1 1/2, 1 + 1/2), (2 1/4, 2 + 1/4), (3 1/8, 3 + 1/8), . . . , (n 1/2
n
, n + 1/2
n
), . . .
so that
_

0
f(x) dx = lim
n
_
n+1/2
n
0
f(x) dx = lim
n
[A(1) + A(2) + A(3) + + A(n) + ]
where A(n) is the area of the isosceles triangles whose base is centered at n. Thus,
_

0
f(x) dx = lim
n
[A(1) + A(2) + A(3) + + A(n) + ]
= lim
n
_
1
2
_
2
2
_
1 +
1
2
_
2
4
_
1 +
1
2
_
2
8
_
1 + +
1
2
_
2
2
n
_
1 +
_
= lim
n
_
1
2
+
1
2
2
+
1
2
3
+ +
1
2
n
+
_
= lim
n
n

k=1
1
2
k
=

k=1
1
2
k
=
1/2
1 1/2
= 1.
71. (a) The overhangs from the edge of the table are:
d
2
=
L
2
+
L
4
=
L
2
_
1 +
1
2
_
=
L
2
H
2
=
1
2
_
3
2
_
= 0.75
d
3
=
L
2
+
L
4
+
L
6
=
L
2
_
1 +
1
2
+
1
3
_
=
L
2
H
3
=
1
2
_
11
6
_
0.917
d
4
=
L
2
+
L
4
+
L
6
+
L
8
=
L
2
_
1 +
1
2
+
1
3
+
1
4
_
=
L
2
H
4
=
1
2
_
25
12
_
1.0417,
9.3. SERIES 591
where H
n
=
n

k=1
1
k
are the harmonic numbers. If m denotes the mass of a book and x
k
denotes the x-coordinate of the center of mass of the kth book, then the centers of mass
are dened by x
n
=
mx
1
+ mx
2
+ + mx
n
nm
=
1
n
n

k=1
x
k
.
Therefore, x
2
=
L
2
+
_
L
2
+
L
2
_
2
=
L
_
2
2
+
1
2
_
2
=
3
4
L
x
3
=
L
2
+
_
L
2
+
L
4
_
+
_
L
2
+
L
4
+
L
2
_
3
=
L
_
3
2
+
2
4
+
1
2
_
3
=
5
6
L
x
4
=
L
2
+
_
L
2
+
L
6
_
+
_
L
2
+
L
6
+
L
4
_
+
_
L
2
+
L
6
+
L
4
+
L
2
_
4
=
L
_
4
2
+
3
6
+
2
4
+
1
2
_
4
=
7
8
L.
In other words, the center of mass for each stack of books is at the edge of the table.
(b) d
4
=
1
2
H
4
=
1
2
_
25
12
_
1.0417 > 1 means that the fourth book is completely beyond
the edge of the table.
(c) The overhang of n books from the edge of the table is
d
n
=
L
2
+
L
4
+
L
6
+ +
L
2(n 1)
+
L
2n
=
L
2
_
1 +
1
2
+
1
3
+ +
1
n 1
+
1
n
_
=
L
2
H
n
,
where H
n
=
n

k=1
1
k
are the harmonic numbers. The x-coordinate of the center of mass is
x
n
=
1
n
_
L
2
+
_
L
2
+
L
2(n 1)
_
+
_
L
2
+
L
2(n 1)
+
L
2(n 2)
_
+
+
_
L
2
+
L
2(n 1)
+
L
2(n 2)
+ +
L
4
+
L
2
+
L
2
_
_
=
L
n
_
n
2
+
n 1
2(n 1)
+
n 2
2(n 2)
+ +
3
6
+
2
4
+
1
2
_
=
L
n
_
n
2
+ (n 1)
1
2
_
=
L
n
_
2n 1
2
_
= L
_
2n 1
2n
_
= L
L
2n
Since the overhang of the rst (or bottom) book in the stack from the edge of the table
is
L
2n
, x
n
is the distance to the edge of the table. That is, the center of mass of n books
is again at the edge of the table.
592 CHAPTER 9. SEQUENCES AND SERIES
(d) For n = 30 and n = 31, Mathematica gives
d
30
=
9304682830147
465817912560
1.99749L
d
31
=
L
2
_
290774257297357
72201776446800
_
2.01362L,
which means that for a stack of 31 books, the overhang of the top book from the edge
of the table is over twice the length of the book.
(e) There is no theoretical limit to the number of books that can be stacked in this manner
because the overhang d
n
=
L
2
H
n
for large n behaves as the divergent harmonic series.
Namely, d
n
as n .
72. The total distance is:
20
_
2
3
_
+ 20
_
2
9
_
+ 20
_
2
27
_
+ = 40
_
1
3
_
+ 40
_
1
3
2
_
+ 20
_
1
3
3
_
+
= 40
_
1/3
1 1/3
_
= 40
_
1/3
2/3
_
= 20 miles
Alternatively, note that the y ies at a constant rate of 20 mph for 1 hour. Thus, the distance
covered is 20(1) = 20 miles.
9.4 Integral Test
1. The function f(x) =
1
x
1.1
is continuous and decreasing on [1, ). Since
_

1
1
x
1.1
dx = lim
t
_
t
1
x
1.1
dx = lim
t
_

1
0.1
x
0.1
__
t
1
= 10(0 1) = 10,
the integral converges and

k=1
1
k
1.1
converges.
2. The function f(x) =
1
x
0.99
is continuous and decreasing on [1, ). Since
_

1
1
x
0.99
dx = lim
t
_
t
1
x
0.99
dx = lim
t
_
1
0.01
x
0.01
__
t
1
= 100 lim
t
(t
0.01
1) = ,
the integral diverges and

k=1
1
k
0.99
diverges.
3. Rewriting the series, we have
1 +
1
2

2
+
1
3

3
+ =

k=1
1
x

x
=

k=1
1
x
3/2
.
9.4. INTEGRAL TEST 593
The function f(x) =
1
x
3/2
is continuous and decreasing on [1, ). Since
_

1
1
x
3/2
dx = lim
t
_
t
1
x
3/2
dx = lim
t
_
2x
1/2
_

t
1
= 2(0 1) = 2,
the integral converges and the series 1 +
1
2

2
+
1
3

3
+ converges.
4. Rewriting the series, we have
1
100
+
1
100

2
+
1
100

3
+ =
1
100
_
1 +
1

2
+
1

3
+
_
=
1
100

k=1
1

k
The function f(x) =
1

x
is continuous and decreasing on [1, ). Since
_

1
1

x
dx = lim
t
_
t
1
x
1/2
dx = lim
t
_
2x
1/2
_

t
1
= ,
the integral diverges and the series
1
100
+
1
100

2
+
1
100

3
+ diverges.
5. The function f(x) =
1
2x + 7
is continuous and decreasing on [1, ). Since
_

1
1
2x + 7
dx = lim
t
_
t
1
1
2x + 7
dx = lim
t
_
1
2
ln(2x + 7)
_

t
1
= ,
the integral diverges and

k=1
1
2k + 7
diverges.
6. Since lim
n
n
3n + 1
=
1
3
= 0, the series diverges by the n-th term test.
7. The function f(x) =
1
1 + 5x
2
is continuous and decreasing on [1, ). Since
_

1
1
1 + 5x
2
dx = lim
t
_
t
1
1
1 + 5x
2
dx = lim
t
_

5
5
tan
1
(

5x)
_

t
1
=

5
5
_

2
tan
1
(

5)
_
,
the integral converges and

k=1
1
1 + 5k
2
converges.
594 CHAPTER 9. SEQUENCES AND SERIES
8. The function f(x) =
x
x
2
+ 5
is continuous and decreasing on [1, ). Since
_

1
x
x
2
+ 5
dx = lim
t
_
t
1
x
x
2
+ 5
= lim
t
_
ln(x
2
+ 5)
2
_

t
1
= ,
the integral diverges and

k=1
k
k
2
+ 5
diverges.
9. Using the limit comparison test with a
n
= ne
n
2
and b
n
= 1/n
2
, we have (using LHopitals
Rule)
lim
n
a
n
b
n
= lim
n
n/e
n
2
1/n
2
= lim
n
n
3
e
n
2
h
= lim
n
3n
2
2ne
n
2
= lim
n
3n
2e
n
2
h
= lim
n
3
4ne
n
2
= 0.
Since

n=1
b
n
is a p-series with p = 2 > 1, it converges and

n=1
ke
k
2
converges.
10. Since
e
1/k
k
2

e
j
2
, the series converges by comparison with the p-series

k=1
e
k
2
.
11. The function f(x) =
x
e
x
is continuous and decreasing on [1, ). Since
_

1
x
e
x
dx = lim
t
_
t
1
x
e
x
dx
= lim
t
_
(x 1)e
x
_

t
1
=
2
e
,
the integral converges and

k=1
k
e
k
converges.
12. The function f(x) = x
2
e
x
is continuous and decreasing on [2, ). Since
_

2
x
2
e
x
dx = lim
t
_
t
2
x
2
e
x
dx = lim
t
_
(x
2
2x 2)e
x
_

t
2
=
10
e
,
the integral converges and

k=2
k
2
e
k
converges.
13. The function f(x) =
1
xlnx
is continuous and decreasing on [2, ). Since
_

2
1
xlnx
dx = lim
t
_
t
2
1
xlnx
dx = lim
t
ln(lnx)
_
t
2
= lim
t
[ln(lnt) ln(ln2)] = ,
the integral diverges and

k=2
1
k lnk
diverges.
9.4. INTEGRAL TEST 595
14. Since k > lnk for k 2, we have
k
lnk
> 1 for k 2. Therefore, since the terms do not
converge to zero, the series

k=2
k
lnk
diverges.
15. The function f(x) =
10
x(lnx)
2
is continuous and decreasing on [2, ). Since
_

2
10
x(lnx)
2
dx = 10 lim
t
_
t
2
(lnx)
2
x
dx = 10 lim
t
[(lnx)
1
]
_
t
2
= 10 lim
t
[(ln2)
1
(lnt)
1
] = 10/ ln2,
the integral converges and

k=2
10
k(lnk)
2
converges.
16. The function f(x) =
1
x

lnx
is continuous and decreasing on [2, ). Since
_

2
1
x

lnx
dx = lim
t
_
t
2
(lnx)
1/2
x
dx = lim
t
2(lnx)
1/2
_
t
2
= lim
t
(2

lnt 2

ln2) = ,
the integral diverges and

k=2
1
k

lnk
diverges.
17. The function f(x) =
arctanx
1 + x
2
is continuous and decreasing on [1, ). Since
_

1
arctanx
1 + x
2
dx = lim
t
_
t
1
arctanx
1 + x
2
dx = lim
t
(arctanx)
2
2
_
t
1
= lim
t
_
(arctant)
2
2


2
32
_
=

2
8


2
32
=
3
2
32
,
the integral converges and

k=1
arctank
1 + k
2
converges.
18. The function f(x) =
x
1 + x
4
is continues and decreasing on [1, ). Since
_

1
x
1 + x
4
dx = lim
t
_
t
1
x
1 + x
4
dx
= lim
t
_
tan
1
(x
2
)
2
_

t
1
=

4


8
=

8
,
The integral converges and

k=1
k
1 + k
4
converges.
596 CHAPTER 9. SEQUENCES AND SERIES
19. The function f(x) =
1
1 +

x
is continuous and decreasing on [1, ). Since
_

1
1
1 +

x
dx = lim
t
_
t
1
1
1 +

x
dx
= lim
t
_
2

x + 1
_

t
1
= ,
the integral diverges and

k=1
1

1 + k
diverges.
20. The function f(x) =
1

1 + x
2
is continuous and decreasing on [1, ). Since
_

1
1

1 + x
2
dx = lim
t
_
t
1
1

1 + x
2
dx
= lim
t
ln(
_
x
2
+ 1 + x)

t
1
= ,
the integral diverges and

k=1
1

1 + k
2
diverges.
21. The function f(x) =
x
(x
2
+ 1)
3
is continuous and decreasing on [1, ). Since
_

1
x
(x
2
+ 1)
3
dx = lim
t
_
t
1
x
(x
2
+ 1)
3
dx
= lim
t
_
1
4(x
2
+ 1)
2
_

t
1
=
1
16
,
the integral converges and

k=1
k
(k
2
+ 1)
3
diverges.
22. The function f(x) =
1
(4x + 1)
3/2
is continuous and decreasing on [1, ). Since
_

1
1
(4x + 1)
3/2
dx = lim
t
_
t
1
1
(4x + 1)
3/2
dx
= lim
t
_
1
2

4x + 1
_

t
1
=
1
2

5
,
the integral converges and

k=1
1
(kx + 1)
3/2
converges.
23. Since lim
x
xsin
1
x
= lim
t0
sint
t
= 1, the series diverges by the n-th term test.
9.4. INTEGRAL TEST 597
24. Since the terms ln(1 +3k) are increasing and positive, we see that the terms do not converge
to zero. Therefore, the series diverges.
25. The function f(x) =
1
x(x + 1)
is continuous and decreasing on [1, ). Since
_

1
1
x(x + 1)
dx = lim
t
_
t
1
1
x(x + 1)
dx
= lim
t
ln
_
x
x + 1
_

t
1
= 0 ln
_
1
2
_
= ln(2),
the integral converges and

k=1
1
k(k + 1)
converges.
26. The function f(x) =
2x + 1
x(x + 1)
is continuous and decreasing on [1, ). Since
_

1
2x + 1
x(x + 1)
dx = lim
t
_
t
1
2x + 1
x(x + 1)
dx
= lim
t
ln(x(x + 1))|
t
1
= ,
the integral diverges and

k=1
2k + 1
k(k + 1)
diverges .
27. The function f(x) =
1
(x + 1)(x + 7)
is continuous and decreasing on [1, ). Since
_

1
1
(x + 1)(x + 7)
dx = lim
t
_
t
1
1
(x + 1)(x + 7)
dx
= lim
t
_
1
6
ln
_
x + 1
x + 7
__

t
1
= 0
1
6
ln
_
2
8
_
=
1
6
ln(4),
the integral converges and

k=1
1
(k + 1)(k + 7)
converges.
28. The function f(x) =
1
x(x
2
+ 1)
is continuous and decreasing on [1, ). Since
_

1
1
x(x
2
+ 1)
dx = lim
t
_
t
1
1
x(x
2
+ 1)
dx
= lim
t
_
1
2
ln
_
x
2
x
2
+ 1
__

t
1
= 0
1
2
ln
_
1
2
_
=
1
2
ln(2),
the integral converges and

k=1
1
k(k
2
+ 1)
converges.
598 CHAPTER 9. SEQUENCES AND SERIES
29. The function f(x) =
2
e
x
+ e
x
is continuous and decreasing on [1, ). Since
_

1
2
e
x
+ e
x
dx = lim
t
_
t
1
2
e
x
+ e
x
dx
= lim
t
2 tan
1
(e
x
)

t
1
= 2 tan
1
(e),
the integral converges and

k=1
2
e
k
+ e
k
converges.
30. The function f(x) =
1

e
3x
is continuous and decreasing on [0, ). Since
_

0
1

e
3x
dx = lim
t
_
t
0
1

e
3x
dx
= lim
t
_
2e
3x
2
3
_

t
0
=
2
3
,
the integral converges and

k=0
1

e
3k
converges. Therefore,

k=0
1

e
3k
converges.
31. Since

k=1
2
k
diverges while

k=1
3
k
2
converges, the entire series

k=1
2
k
+
3
k
2
diverges.
32. Since

k=1
5k
1.6
and

k=1
10k
1.1
both converge by the p-series test, the entire series

k=1
5k
1.6
10k
1.1
converges.
33.

k=1
1
k
2
donverges by the p-series test,

k=1
1
2
k
is a geometric series with r =
1
2
< 1. Therefore,

k=1
1
k
2
+
1
2
k
converges.
34.

k=1
1
k
2
converges by the p-series test.

k=1
4

k
k
2
=

k=1
4
k
3/2
also converges by the p-series test.
Therefore,

k=1
1 + 4

k
k
2
converges.
35. For

k=2
1
k(lnk)
p
, we have
_

2
(lnx)
p
_
1
x
dx
_
= lim
b
(lnx)
p+1
_
b
2
= lim
b
_
1
(lnb)
p1

1
(ln2)
p1
_
.
9.4. INTEGRAL TEST 599
For p > 1, this limit converges to
1
(ln2)
p1
. For p = 1,
_

2
1
lnx
_
1
x
_
dx = lim
b
[ln(lnb)
ln(ln2)] = . For p < 1, the integral also diverges.
36. For

k=3
1
k lnk[ln(lnk)]
p
, we have
_

3
[ln(lnx)]
p
_
1
xlnx
dx
_
= lim
b
[ln(lnx)]
p+1
_
b
3
= lim
b
_
1
[ln(lnb)]
p1

1
[ln(ln3)]
p1
_
.
By the same reasoning as in Exercise 35, the series converges for p > 1 and diverges for p 1.
37. For p 0, f(x) = x
p
lnx is not decreasing. So for p < 0, integration by parts gives

k=2
k
p
lnk =
_

2
x
p
lnxdx = lim
b
x
p+1
(p + 1)
2
[1 (p + 1) lnx]
_
b
2
= lim
b
_
b
p+1
(p + 1)
2
[1 (p + 1) lnb]
2
p+1
(p + 1)
2
[1 (p + 1) ln2]
_
=
2
p+1
(p + 1)
2
[1 (p + 1) ln2] for p + 1 < 0 or p < 1.
The integral converges for p < 1 and diverges for 1 p < 0 and for p 0 (that is, the
integral diverges for p 1).
38. Since f is decreasing and f(k) = a
k
, we have f(x) f(k) = a
k
on [k, k + 1]. This implies
_
k+1
k
f(x)dx
_
k+1
k
a
k
dx = a
k
. Therefore,
_
n+1
1
f(x)dx =
n

k=1
_
k+1
k
f(x)dx
_
k+1
k
a
k
.
Also, note that f(x) f(k+1) = a
k+1
on .[k, k+1]. This implies
_
k+1
k
f(x)dx
_
k+1
k
a
k+1
dx =
a
k+1
. Therefore,
n

k=1
a
k
= a
1
+
n

k=2
a
k
= a
1
+
n1

k=1
_
k+1
k
a
k+1
dx
a
1
+
n1

k=1
_
k+1
k
f(x)dx = a
1
+
_
n
1
f(x)dx
39. The function f(x) =
1
1 + x
2
is continuous, positive, and decreasing on [1, ). Using the result
from Problem 38, we have
_
n+1
1
1
1 + x
2
dx
n

k=1
1
1 + k
2

1
2
+
_
n
1
1
1 + x
2
dx
Letting n , we have
_

1
1
1 + x
2
dx

k=1
1
1 + k
2

1
2
+
_

1
1
1 + x
2
dx
600 CHAPTER 9. SEQUENCES AND SERIES
After integrating, this becomes

k=1
1
1 + k
2

1
2
+

4
40. (a) The function f(x) =
1
x
is continuous, positive, and decreasing on [1, ). Using the
result from Problem 38, we have
_
10
10
+1
1
1
x
dx
10
1
0

k=1
1 +
_
10
1
0
1
1
x
dx.
Performing the integration, we have
23.02585
10
10

k=1
24.02585
(b) To n d the number of terms necessary to guarantee S
n
100, we need to nd n such
that
100
_
n+1
1
1
x
dx = ln(x)|
n+1
1
= ln(n + 1).
Therefore, e
100
n + 1 or n e
100
1.
41. Since f is decreasing and f(k) = a
k
, we have f(x) f(k) = a
k
on [k, k + 1]. This implies
_
k+1
k
f(x)dx
_
k+1
k
a
k
dx = a
k
. Therefore,
_
n+p
n+1
f(x)dx =
n+p1

k=n+1
_
k+1
k
f(x)dx
n+p1

k=n+1
a
k
.
Since this is true for every integer p, we can let p to get
_

n+1
f(x)dx

k=n+1
a
k
or
_

n+1
f(x)dx R
n
. Also, note that f(x) f(k + 1) = a
k+1
on [k, k + 1]. This implies
_
k+1
k
f(x)dx
_
k+1
k
a
k+1
dx = a
k+1
. Therefore,
n+p

k=n+1
a
k
=
n+p1

k=n
a
k+1

n+p1

k=n
_
k+1
k
f(x)dx =
_
n+p
n
f(x)dx.
Since this is true for every integer p, we can let p to get

k=n+1
a
k

_

n
f(x)dx or
R
n

_

n
f(x)dx. Hence
_

n+1
f(x)dx R
n

_

n
f(x)dx.
42. Letting f(x) =
1
x
2
and using the result from Problem 41, we need to nd n such that
_

n
1
x
2
dx < 0.001. Performing the integration,
1
n
< 0.001 or n > 1000.
9.5. COMPARISON TESTS 601
9.5 Comparison Tests
1. Since
1
(k + 1)(k + 2)
<
1
k
2
, the series converges by comparison with the p-series

k=1
1
k
2
.
2. Since
1
k
2
+ 5
<
1
k
2
, the series converges by comparison with the p-series

k=1
1
k
2
.
3. Since
1

k 1

1

k
for k 2, the series diverges by comparison with the p-series

k=1
1
k
1/2
.
4. Since
2k
2
+ 1
k
3
k

2k
2
+ 1
k
3

2k
2
k
3
=
2
k
for k 2, the series diverges by comparison with the
harmonic series

k=1
2
k
.
5. Since
1
lnk
>
1
k
for k 2, the series diverges by comparison with the harmonic series

k=1
1
k
.
6. Since
lnk
k
5
<
k
k
5
=
1
k
4
, the series converges by comparison with the p-series

k=3
1
k
4
.
7. Since
1 + 3
k
2
k
=
1
2
k
+
_
3
2
_
k
> 1 for all k 1 the series diverges by comparison with the series

k=1
1 .
8. Since
1 + 8
k
3 + 10
k

1 + 8
k
10
k

9
k
10
k
=
_
9
10
_
k
for all k 1, the series converges by comparison
with the geometric series

k=1
_
9
10
_
k
.
9. Since
2 + sink
3

k
4
+ 1
<
3
k
4/3
, the series converges by comparison with the p-series

k=1
3
k
4/3
.
10. Since 2k+1 > lnk, then
2k + 1
k lnk
>
1
k
, and the series diverges by comparison with the harmonic
series

k=1
1
k
.
11. Since j +e
j
< j +9, then
j + e
j
5
j
(j + 9)
<
1
5
j
, and the series converges by comparison with the
geometric series

j=1
_
1
5
_
j
.
602 CHAPTER 9. SEQUENCES AND SERIES
12. Since
ie
i
i + 1
=
_
i
i + 1
_
1
e
i
<
1
e
i
, the series converges by comparison with the geometric series

i=1
_
1
e
_
i
.
13. Since

k + 1

k
k
=
1
k(

k + 1 +

k)

1
k(

k +

k)
=
1
2k
3/2
, the series converges by
comparison with the p-series
1
2

k=1
1
k
3/2
.
14. Since
1
n3
n
<
1
3
n
, the series converges by comparison with the geometric series

k=1
_
1
3
_
n
.
15. Using the limit comparison test with a
n
=
1
2n + 7
and b
n
=
1
n
, we have
lim
n
a
n
b
n
= lim
n
1/(2n + 7)
1/n
= lim
n
n
2n + 7
=
1
2
.
Since

n=1
b
n
diverges,

n=1
1
2n + 7
diverges.
16. Using the limit comparison test with a
n
=
1
10 +

n
and b
n
=
1

n
, we have
lim
n
a
n
b
n
= lim
n
1/(10 +

n)
1/

n
= lim
n

n
10 +

n
= 1.
Since

n=1
b
n
is a p-series with p =
1
2
< 1, it diverges and

k=1
1
10 +

k
diverges.
17. Using the limit comparison test with a
n
=
1
n

n
2
1
and b
n
=
1
n
2
, we have
lim
n
a
n
b
n
= lim
n
1/n

n
2
1
1/n
2
= lim
n
n
2
n

n
2
1
= lim
n
1
_
1 1/n
2
= 1.
Since

n=1
b
n
is a p-series with p = 2 > 1, it converges and

n=2
1
n

n
2
1
converges.
18. Using the limit comparison test with a
n
=
1
_
(n + 1)(n + 2)
and b
n
=
1
n
, we have
lim
n
a
n
b
n
= lim
n
1/
_
(n + 1)(n + 2)
1/n
= lim
n
n

n
2
+ 3n + 2
= lim
n
1
_
1 + 3/n + 2/n
2
= 1.
Since

n=1
1
n
diverges,

n=1
1
_
(n + 1)(n + 2)
diverges.
9.5. COMPARISON TESTS 603
19. Using the limit comparison test with a
n
=
n
2
n + 2
3n
5
+ n
2
and b
n
=
1
n
3
, we have
lim
n
a
n
b
n
= lim
n
(n
2
n + 2)/(3n
5
+ n
2
)
1/n
3
= lim
n
n
5
n
4
+ 2n
3
3n
5
+ n
2
= lim
n
1 1/n + 2/n
2
3 + 1/n
3
=
1
3
.
Since

n=1
b
n
is a p-series with p = 3 > 1, it converges and

n=1
n
2
n + 2
3n
5
+ n
2
converges.
20. Using the limit comparison test with a
n
=
n
(4n + 1)
3/2
and b
n
=
1
n
1/2
, we have
lim
n
a
n
b
n
= lim
n
n/(4n + 1)
3/2
1/n
1/2
= lim
n
n
3/2
(4n + 1)
3/2
= lim
n
_
1
4 + 1/n
_
3/2
=
1
8
.
Since

n=1
b
n
is a p-series with p =
1
2
< 1, it diverges and

n=2
n
(4n + 1)
3/2
diverges.
21. Using the Limit Comparison Test with a
k
=

k + 1
3

64k
9
+ 40
and b
k
=

k
3

64k
9
, we have
lim
k
a
k
b
k
= lim
k
_
k + 1
3

64k
9
+ 40
_
_

k
3

64k
9
_
= lim
k

k + 1

64k
9
3

64k
9
+ 40
= lim
k

k + 1

k
lim
k
3

64k
9
3

64k
9
+ 40
= 1 1 = 1.
Since

k=1
b
k
=

k=1

k
3

64k
9
=

k=1
1
4k
5/2
is a p-series with p = 5/2 > 1, it converges and

k=1

k + 1
3

64k
9
+ 40
converges.
22. Using the Limit Comparison Test with a
k
=
5k
2
k
2k
3
+ 2k
2
8
and b
k
=
1
k
, we have
lim
k
a
k
b
k
= lim
k
5k
2
k
2k
3
+ 2k
2
8
1
k
= lim
k
5k
3
k
2
2k
3
+ 2k
2
8
=
5
2
.
Since

k=2
1
k
diverges,

k=2
5k
2
k
2k
3
+ 2k
2
8
diverges.
604 CHAPTER 9. SEQUENCES AND SERIES
23. Using the limit comparison test with a
n
=
n + lnn
n
3
+ 2n 1
and b
n
=
1
n
2
, we have
lim
n
a
n
b
n
= lim
n
(n + lnn)/(n
3
+ 2n 1)
1/n
2
= lim
n
n
3
+ n
2
lnn
n
3
+ 2n 1
= lim
n
1 + (lnn)/n
1 + 2/n 1/n
= 1.
(By LHopitals Rule, lim
n
lnn
n
= lim
n
1/n
1
= 0.) Since

n=1
1
n
2
is a p-series with p = 2 > 1,
it converges and

n=2
k + lnk
k
3
+ 2k 1
converges.
24. Using the Limit Comparison Test with a
k
=
10
e
k
2
and b
k
=
1
e
k
, we have
lim
k
a
k
b
k
= lim
k
_
10
e
k
2
_
_
1
e
k
_ = lim
k
10e
k
e
k
2
= 10.
Since

k=1
b
k
is a geometric series with r =
1
e
< 1, it converges and

k=1
10
e
k
2
converges.
25. Using the limit comparison test with a
n
= sin
1
n
and b
n
=
1
n
, we have
lim
n
a
n
b
n
= lim
n
sin(1/n)
1/n
h
= lim
n
cos(1/n)(1/n
2
)
1/n
2
= lim
n
cos
1
n
= 1.
Since

n=1
1
n
diverges, the series

k=1
sin
1
k
diverges.
26. Using the limit comparison test with a
n
= 1 cos
1
n
and b
n
=
1
n
2
, we have
lim
n
a
n
b
n
= lim
n
1 cos(1/n)
1/n
2
h
= lim
n
sin(1/n)(1/n
2
)
2/n
3
=
1
2
lim
n
_
nsin
1
n
_
=
1
2
lim
n
sin(1/n)
1/n
=
1
2
lim
t0
sint
t
=
1
2
.
Since

n=1
1
n
2
is a p-series with p = 2 > 1, it converges and

k=1
_
1 cos
1
k
_
converges.
27. Using the limit comparison test with a
n
=
_
1
2
+
1
2n
_
n
and b
n
=
_
1
2
_
n
, we have
lim
n
a
n
b
n
= lim
n
_
1
2
+
1
2n
_
n
(1/2)
n
= lim
n
_
1 +
1
n
_
n
= e.
9.5. COMPARISON TESTS 605
Since

n=1
b
n
is a geometric series with r = 1/2 < 1, it converges and the series

k=1
_
1
2
+
1
2k
_
k
converges.
28. Using the limit comparison test with a
n
=
n
(n + 1)(n + 2)
and b
n
=
1
n
, we have
lim
n
a
n
b
n
= lim
n
n/(n + 1)(n + 2)
1/n
= lim
n
n
2
n
2
+ 3n + 2
= 1.
Since

n=1
1
n
diverges, the series

n=1
n
(n + 1)(n + 2)
diverges.
29. Since lim
k
k
100

k
2
+ 1
=
1
100
, the series diverges by the n-th term test.
30. Using the limit comparison test with a
n
=
1
n +

n
and b
n
=
1
n
, we have
lim
n
a
n
b
n
= lim
n
1/(n +

n)
1/n
= lim
n
n
n +

n
= lim
n
1
1 + 1/

n
= 1.
Since

n=1
1
n
diverges, the series

k=1
1
k +

k
diverges.
31. Since lim
k
ln
_
5 +
k
5
_
= , the series diverges by the n-th term test.
32. Using the limit comparison test with a
n
= ln
_
1 +
1
3
n
_
and b
n
=
1
3
n
, we have (using
LHopitals Rule)
lim
n
a
n
b
n
= lim
n
ln(1 + 3
n
)
3
n
h
= lim
n
(3
n
ln3)/(1 + 3
n
)
3
n
ln3
= lim
n
1
1 + 3
n
= 1.
Since

n=1
b
n
is a geometric series with r = 1/3 < 1, it converges and the series

k=1
ln
_
1 +
1
3
k
_
converges.
33. The function f(x) =
x
(x
2
+ 1)
2
is continuous and decreasing on [1, ). Since
_

1
x
(x
2
+ 1)
2
dx = lim
t
_
t
1
x
(x
2
+ 1)
2
dx =
1
2
lim
t
1
x
2
+ 1
_
t
1
=
1
2
lim
t
_
1
t
2
+ 1

1
2
_
=
1
4
,
the integral converges and

k=1
k
(k
2
+ 1)
2
converges.
(The direct comparison test and limit comparison test can also be used.)
606 CHAPTER 9. SEQUENCES AND SERIES
34. Since
k

k 1
3

k
2
2
>
1
k
1/6
for k 2, the series diverges by comparison with the p-series

k=2
1
k
1/6
.
35. Since
1
9 + sin
2
k
>
1
10
for k 1, the series diverges by comparison with the series

k=1
1
10
.
36. Using the limit comparison test with a
n
=
3
n
3
2n
1
and b
n
=
1
3
n
, we have
lim
n
a
n
b
n
= lim
n
3
n
/(3
2n
1)
1/3
n
= lim
n
3
2n
3
2n
1
= lim
n
1
1 1/3
2n
= 1.
Since

n=1
b
n
is a geometric series with r = 1/3 < 1, it converges and the series

k=1
3
k
3
2k
1
converges.
37. Since
2
2 + k2
k
<
2
k2
k
<
2
2
k
=
1
2
k1
, the series converges by comparison with the geometric
series

k=1
1
2
k1
.
38. Since lim
k
2
2 + k2
k
= 1, the series diverges by the n-th term test.
39. Since ln
_
1 +
1
k
_
> ln
1
k
= lnk, the series diverges by comparison with the series

k=2
lnk.
40. Since
(0.9)
k
k
(0.9)
k
, the series converges by comparison with the geometric series

k=1
(0.9)
k
.
41. Since

a
k
converges, then lim
k
a
k
= 0. Therefore, for a suciently large n and k n, we
can say that 0 < a
k
< 1 and so a
2
k
< a
k
. Thus,

a
2
k
converges by the direct comparison
test.
42. Assuming q(k) = 0 for k 1, the series will converge if m n + 2.
43. The statement is false. The condition of a positive-term series is missing: as a counterexample,
consider the convergent series

b
k
=

0 = 0 and the divergent series a


k
=

1
k
.
44. For a
k
> 0 for all k, the limit comparison test
lim
n
ln(1 + a
n
)
a
n
= lim
x0
ln(1 + x)
x
h
= lim
x0
1/(1 + x)
1
= 1
shows that if

a
k
, then

ln(1 + a
k
) converges.
9.6. RATIO AND ROOT TESTS 607
45. The series

k=1
1
k
1+1/k
diverges by the limit comparison test with

k=1
1
k
:
lim
n
a
n
b
n
= lim
n
1/n
1+1/n
1/n
= lim
n
1
n

n
= 1.
46. Since 1 +2 +3 + +n =
1
2
n(n +1), we have

k=1
1
1 + 2 + 3 + + k
=

k=1
2
k(k + 1)
, which
converges by either the direct comparison test or the limit comparison test with

k=1
2
k
2
.
47. Of the set of integers a
i
used in the decimal representation of the number, let a
B
be the biggest
integer. Then for the series

k=1
a
k
10
k
, we have
a
k
10
k

a
B
10
k
for all k. Since

k=1
a
B
10
k
= a
B

k=1
1
10
k
and

k=1
1
10
k
=

k=1
_
1
10
_
k
is a convergent geometric series, then

k=1
a
k
10
k
converges by the
direct comparison test.
48. This exercise involves a research report, and thus a preset solution is not given.
9.6 Ratio and Root Tests
1. Since lim
n
a
n+1
a
n
= lim
n
1/(n + 1)!
1/n!
= lim
n
n!
(n + 1)!
= lim
n
1
n + 1
= 0 < 1,
the series converges by the ratio test.
2. Since lim
n
a
n+1
a
n
= lim
n
2
n+1
/(n + 1)!
2
n
/n!
= lim
n
2
n + 1
= 0 < 1,
the series converges by the ratio test.
3. Since lim
n
a
n+1
a
n
= lim
n
(n + 1)!/1000
n+1
n!/1000
n
= lim
n
n + 1
1000
= ,
the series diverges by the ratio test.
4. Since lim
n
a
n+1
a
n
= lim
n
(n + 1)(2/3)
n+1
n(2/3)
n
= lim
n
_
1 +
1
n
__
2
3
_
=
2
3
< 1,
the series converges by the ratio test.
5. Since lim
n
a
n+1
a
n
= lim
n
(n + 1)
10
/(1.1)
n+1
n
10
/(1.1)
n
= lim
n
_
1 +
1
n
_
10
1
1.1
=
1
1.1
< 1,
the series converges by the ratio test.
6. Since lim
n
a
n+1
a
n
= lim
n
1/(n + 1)
5
(0.99)
n+1
1/n
5
(0.99)
n
= lim
n
_
n
n + 1
_
5
1
0.99
=
1
0.99
> 1,
the series diverges by the ratio test.
608 CHAPTER 9. SEQUENCES AND SERIES
7. Since lim
n
a
n+1
a
n
= lim
n
4
n
/(n + 1)3
n1
4
n1
/n3
n2
= lim
n
4
3
_
n
n + 1
_
=
4
3
> 1,
the series diverges by the ratio test.
8. Since lim
n
a
n+1
a
n
= lim
n
(n + 1)
3
2
n+4
/7
n
n
3
2
n+3
/7
n1
= lim
n
_
1 +
1
n
_
3
_
2
7
_
=
2
7
< 1,
the series converges by the ratio test.
9. Since
lim
n
a
n+1
a
n
= lim
n
(n + 1)!/(2n + 2)!
n!/(2n)!
= lim
n
n + 1
(2n + 1)(2n + 2)
= lim
n
n + 1
4n
2
+ 6n + 2
= 0 < 1,
the series converges by the ratio test.
10. Since
lim
k
a
k+1
a
k
= lim
k
(2k + 2)!/(k + 1)!2
k+1
(k + 1)
k+1
(2k)!/k!2
k
k
k
= lim
k
(2k + 1)(2k + 2)
2(k + 1)(k + 1)
_
k
k + 1
_
k
= lim
k
(2k + 1)(2k + 2)
2(k + 1)
2
_
k + 1
k
_
k
= lim
k
4k
2
+ 6k + 2
(2k
2
+ 4k + 2)
_
1 +
1
k
_
k
=
2
e
< 1,
the series converges by the ratio test.
11. Since
lim
n
a
n+1
a
n
= lim
n
99
n+1
((n + 1)
3
+ 1)
99
n
(n
3
+ 1)

n
2
100
n
(n + 1)
2
100
n+1
= lim
n
_
99
100
_

_
(n + 1)
3
+ 1
n
3
+ 1
_

_
n
2
(n + 1)
2
_
=
_
99
100
__
lim
n
(n + 1)
3
+ 1
n
3
+ 1
__
lim
n
n
2
(n + 1)
2
_
=
99
100
1 1 =
99
100
< 1,
the series converges by the ratio test.
12. Since
lim
n
a
n+1
a
n
= lim
n
(n + 1)!
n!

e
n
2
e
(n+1)
2
= lim
n
(n + 1)
e
n
2
e
n
2
e
2n
e
= lim
n
n + 1
e
2n+1
= 0 < 1,
the series converges by the ratio test.
9.6. RATIO AND ROOT TESTS 609
13. Since
lim
n
a
n+1
a
n
= lim
n
5
n+1
5
n

n
n
(n + 1)
n+1
lim
n
5
n
n
n
n+1
= lim
n
5
n
= 0 < 1,
the series converges by the ratio test.
14. Since
lim
k
a
k+1
a
k
= lim
k
(k + 1)!3
k+1
/(k + 1)
k+1
k!3
k
/k
k
= lim
k
3
_
k
k + 1
_
k
= lim
k
3
(1 + 1/k)
k
=
3
e
> 1,
the series diverges by the ratio test.
15. Since lim
k
a
k+1
k
n
= lim
k
1 3 5 (2k + 1)/(k + 1)!
1 3 5 (2k 1)/k!
= lim
k
2k + 1
k + 1
= 2 > 1,
the series diverges by the ratio test.
16. Since lim
k
a
k+1
k
n
= lim
k
(k + 1)!/[2 4 6 (2k + 2)]
k!/[2 4 6 (2k)]
= lim
k
k + 1
2k + 2
=
1
2
< 1,
the series converges by the ratio test.
17. Since lim
n
_
1
n
n
_
1/n
= lim
n
1
n
= 0 < 1, the series converges by the root test.
18. Since lim
k
_
_
ke
k + 1
_
k
_
1/k
= lim
k
ke
k + 1
= lim
k
e
1 + 1/k
= e > 1,
the series diverges by the root test.
19. Since lim
n
__
n
lnn
_
n
_
1/n
= lim
n
n
lnn
= , the series diverges by the root test.
20. Since lim
n
_
1
(lnn)
n
_
1/n
= lim
n
1
lnn
= 0 < 1, the series converges by the root test.
21. Since lim
n
_
_
n
n + 1
_
n
2
_
1/n
= lim
n
_
n
n + 1
_
n
= lim
n
1
_
n + 1
n
_
n
= lim
n
1
_
1 +
1
n
_
n
=
1
e
< 1,
the series converges by the root test.
22. Using the root test, lim
n
_
_
1
2
n
_
n
2
_
1/n
= lim
n
_
1
2
n
_
n
. Let y =
_
1
2
x
_
x
.
610 CHAPTER 9. SEQUENCES AND SERIES
Then lny = xln
_
1
2
x
_
=
ln(1 2/x)
1/x
and by LHopitals Rule,
lim
x
lny = lim
x
ln(1 2/x)
1/x
h
= lim
x
2/x
2
(1 2/x)
1/x
2
= lim
x
_

2
1 2/x
_
= 2.
Thus, lim
n
_
1
2
n
_
n
2
= e
2
< 1, and the series converges.
23. Since lim
n
_
6
2n+1
n
n
_
1/n
= lim
n
6
2+1/n
n
= 0 < 1, the series converges by the root test.
24. Since lim
k
_
k
k
e
k+1
_
1/k
= lim
k
k
e
1+1/k
= , the series diverges by the root test.
25. Since
k
2
+ k
k
3
+ 2k + 1

k
2
k
3
+ 2k + 1

k
2
k
3
+ 2k
3
+ k
3
=
1
4k
for k 1, the series
diverges by comparison with the series
1
4

k=1
1
k
.
26. Since lim
n
__
3n
2n + 1
_
n
_
1/n
= lim
n
3n
2n + 1
=
3
2
> 1, the series diverges by the root
test.
27. Since
e
1/n
n
2

e
n
2
for n 1, the series converges by the comparison with the series

n=1
e
n
2
.
28. Since lim
n
a
n+1
a
n
= lim
n
(n + 1)
2
+ (n + 1)
n
2
+ n

e
n
e
n+1
= lim
n
__
n
2
+ 3n + 2
n
2
+ n
__
lim
n
_
1
e
_
=
1
e
< 1,
the series converges by the ratio test.
29. Since lim
n
a
n+1
a
n
= lim
n
(5
n+1
(n + 1)!
5
n
n!

(n + 1)!
(n + 2)!
= lim
n
5(n + 1)
n + 2
= 5 > 1,
the series diverges by the ratio test.
30. Since
3
2
k
+ k

3
2
k
= 3
_
1
2
_
k
for k 1, the series converges by comparison with
the geometric series,

k=1
3
_
1
2
_
k
..
9.6. RATIO AND ROOT TESTS 611
31. Since
2
k
3
k
+ 4
k

2
k
3
k
=
_
2
3
_
k
for k 0, the series converges by comparison with

k=0
_
2
3
_
k
..
32. Since a
n
=
n
n + 2
and lim
n
a
n
= 1, we see that the terms do not converge to zero. Therefore,
the series diverges.
33. Applying the ratio test, we have lim
n
a
n+1
a
n
= lim
n
(n + 1)p
n+1
np
n
= lim
n
_
1 +
1
n
_
p = p.
Thus, the series converges for 0 p < 1 and diverges for p > 1. For p = 1, the series is

k=1
k,
which diverges.
34. Applying the ratio test, we have lim
n
a
n+1
a
n
= lim
n
(n + 1)
2
2
n+1
/p
n+1
n
2
2
n
/p
n
= lim
n
2
p
_
1 +
1
n
_
2
=
2
p
. For 2/p < 1 or p > 2, the series converges, and for 0 < p < 2, it diverges. For p = 2, the
series is

k=1
k
2
, which diverges.
35. Applying the ratio test, we have lim
n
a
n+1
a
n
= lim
n
(n + 1)
p
/(n + 1)!
n
p
/n!
= lim
n
_
n + 1
n
_
p
1
n + 1
=
0. The series converges for all real values of p.
36. For p 1 and k 3, the general term of the series
lnk
k
p

1
k
and the series diverges by
comparison with the harmonic series. For p > 1, we note that f(x) =
lnx
x
p
has derivative
f

(x) =
1 p lnx
x
p+1
and that f

(x) < 0 for x 3. Thus, f(x) is continuous and decreasing on


612 CHAPTER 9. SEQUENCES AND SERIES
[3, ) and we may apply the integral test:
_

3
lnx
x
p
dx t = lnx, x = e
t
, dx = e
t
dt
=
_

ln 3
t
e
pt
e
t
dt =
_

ln 3
te
(1p)t
dt
u = t, du = dt; dv = e
(1p)t
dt, v =
1
1 p
e
(1p)t
=
1
1 p
te
(1p)t
_

ln 3

_

ln 3
1
1 p
e
(1p)t
dt
=
1
1 p
te
(1p)t
_

ln 3

1
(1 p)
2
e
(1p)t
_

ln 3
=
1
1 p
_
lim
t
t
e
(p1)t
(ln3)3
1p
_

1
(1 p)
2
_
lim
t
e
(1p)t
3
1p
_
h
=
1
1 p
_
lim
t
1
(p 1)e
(p1)t
3
1p
ln3
_

1
(1 p)
2
(3
1p
)
=
3
1p
(1 p)
2

3
1p
ln3
1 p
Thus, the integral converges and

k=2
lnk
k
p
converges for p > 1.
37. (a) F
n
+ F
n1
=
1

5
_
1 +

5
2
_
n

5
_
1

5
2
_
n
+
1

5
_
1 +

5
2
_
n1

5
_
1

5
2
_
n1
=
1

5
_
1 +

5
2
_
n1
_
1 +

5
2
+ 1
_

5
_
1

5
2
_
n1
_
1

5
2
+ 1
_
=
1

5
_
1 +

5
2
_
n1
_
1 +

5
2
_
2

5
_
1

5
2
_
n1
_
1

5
2
_
2
=
1

5
_
1 +

5
2
_
n1

5
_
1

5
2
_
n1
= F
n+1
(b) F
1
=
1

5
_
1 +

5
2
_

5
_
1

5
2
_
= 1
F
2
=
1

5
_
1 +

5
2
_
2

5
_
1

5
2
_
2
= 1
F
3
=
1

5
_
1 +

5
2
_
3

5
_
1

5
2
_
3
= 2
F
4
=
1

5
_
1 +

5
2
_
4

5
_
1

5
2
_
4
= 3
9.6. RATIO AND ROOT TESTS 613
F
5
=
1

5
_
1 +

5
2
_
5

5
_
1

5
2
_
5
= 5
38. lim
n
F
n+1
F
n
= lim
n
1

5
_
1+

5
2
_
n+1

5
_
1

5
2
_
n+1
1

5
_
1+

5
2
_
n

5
_
1

5
2
_
n
= lim
n
1
2

(1 +

5)
n+1
(1

5)
n+1
(1 +

5)
n
(1

5)
n
Dividing numerator and denominator by (1 +

5)
n
we have
lim
n
F
n+1
F
n
=
1
2
lim
n
1 +

5 (1

5)
_
1

5
1 +

5
_
n
1
_
1

5
1 +

5
_
n
.
Now
1

5
1 +

5
=
1

5
1 +

5
1 +

5
=
1 2

5 + 5
1 5
=

5 3
2
. Since |(

53)/2| < 1, lim


n
_
1

5
1 +

5
_
n
=
lim
n
_

5 3
2
_
n
= 0, and lim
n
F
n+1
F
n
=
1
2
(1 +

5).
39. Applying the ration test to

n1
1
F
n
, we have
lim
n
a
n+1
a
n
= lim
n
1/F
n+1
1/F
n
= lim
n
F
n
F
n+1
=
2
1 +

5
< 1.
Thus, the series

n=1
1
F
n
converges.
40. (a) Using the ratio test, we have
lim
n
a
n+1
a
n
= lim
n
1103 + 36390(n + 1)
1103 + 26390n
(4n + 4)!
[(n + 1)!]
4
[4.99]
n+1
(n!)
4
(4.99)
n
(4n)!
= lim
n
1103 + 36390(n + 1)
1103 + 26390n
(4n + 4)(4n + 3)(4n + 2)(4n + 1)
(n + 1)
4
(4)(99)
= lim
n
4 4 4 4
4 99
=
64
99
< 1.
Thus, the series converges.
(b) The rst term of the series is 1/ 2

2(1103)/9801, which gives 9801/2

2(1103)
3.14159273. This is accurate to 6 decimal places.
(c) The rst two terms of the series give
1/
2

2
9801
_
1103 + 27, 493
_
4!
(4 99)
4
__
=
2

2[1103 99
4
+ 27493 6]
9801 99
4
.
Then 9801(99)
4
/2

2[1103 99
4
+ 27493 6] 3.14159265358979388,
which is accurate to 14 decimal places.
614 CHAPTER 9. SEQUENCES AND SERIES
9.7 Alternating Series
1. Since a
k+1
=
1
k + 3
<
1
k + 2
= a
k
and lim
k
1
k + 2
= 0, the series converges.
2. Since a
k+1
=
1

k + 1
<
1

k
= a
k
and lim
k
1

k
= 0, the series converges.
3. Since lim
k
k
k + 1
= 1 = 0, the series diverges.
4. Let f(x) =
x
x
2
+ 1
. Then f

(x) =
1 x
2
(x
2
+ 1)
2
< 0 for x > 1 and a
k+1
< a
k
. Since
lim
k
k
k
2
+ 1
= lim
k
1
k + 1/k
= 0, the series converges.
5. Let f(x) =
x
2
+ 2
x
3
. Then f

(x) =
x
2
+ 6
x
4
< 0 for x 1 and a
k+1
< a
k
. Since lim
k
k
2
+ 2
k
3
=
lim
k
1/k + 2/k
3
1
= 0, the series converges.
6. Since lim
k
3k 1
k + 5
= 3 = 0, the series diverges.
7. Since a
k+1
=
1
k + 1
+
1
3
k+1
<
1
k
+
1
3
k
= a
k
and lim
k
_
1
k
+
1
3
k
_
= 0, the series converges.
8. Since a
k+1
=
k + 2
4
k+1
=
k/4 + 1/2
4
k
<
k + 1
4
k
= a
k
and using LHopitals Rule, lim
k
k + 1
4
k
h
=
lim
k
1
4
k
ln4
= 0, the series converges.
9. Let f(x) =
4

x
2x + 1
. Then f

(x) =
2 4x

x(2x + 1)
2
< 0 for x 1 and a
n+1
< a
n
. Since
lim
n
4

n
2n + 1
= lim
n
4
2

n + 1/

n
= 0, the series converges.
10. Let f(x) =
x
1/3
x + 1
. Then f

(x) =
1 2x
3x
2/3
(x + 1)
2
< 0 for x 1 and a
n+1
< a
n
. Since
lim
n
3

n
n + 1
= lim
n
1
n
2/3
+ n
1/3
= 0, the series converges.
11. Note that cos n = (1)
n
. Let f(x) =

x + 1
x + 2
. Then f

(x) =
2x
2

x + 1(x + 2)
2
< 0 for x 2
and a
n+1
< a
n
. Since lim
n

n + 1
n + 2
= lim
n
_
1 + 1/n

n + 2/

n
= 0, the series converges.
12. Let f(x) =

x
2
+ 1
x
3
. Then f

(x) =
2x
2
3
x
4

x
2
+ 1
< 0 for x 2 and a
k+1
< a
k
. Since
lim
n

k
2
+ 1
k
3
= lim
n
_
1 + 1/k
2
k
2
= 0, the series converges.
9.7. ALTERNATING SERIES 615
13. Using LHopitals Rule, lim
x
x
lnx
= lim
x
1
1/x
= lim
x
x = 0, and the series diverges.
14. Since a
k+1
=
1
ln(k + 1)
<
1
lnk
= a
k
and lim
k
1
lnk
= 0, the series converges.
15. Apply the limit comparison test to

k=1
1
2k + 1
with a
k
=
1
2k + 1
and b
k
=
1
k
:
lim
k
a
k
b
k
= lim
k
1/(2k + 1)
1/k
= lim
k
k
2k + 1
=
1
2
.
Since

k=1
b
k
diverges, the given series is not absolutely convergent. Since a
k+1
=
1
2k + 3
<
1
2k + 1
= a
k
and lim
k
1
2k + 1
= 0, the series is conditionally convergent.
16. Apply the limit comparison test to

k=1
1

k + 5
with a
k
=
1

k + 5
and b
k
=
1

k
:
lim
k
a
k
b
k
= lim
k
1/

k + 5
1/

k
= lim
k
1
_
1 + 5/k
= 1.
Since

k=1
b
k
diverges, the given series is not absolutely convergent. Since a
k+1
=
1

k + 6
<
1

k + 5
= a
k
and lim
k
1

k + 5
= 0, the series is conditionally convergent.
17. Since

k=1
_
2
3
_
k
is a geometric series with r =
2
3
< 1, the series is absolutely convergent.
18. Since
2
2k
3
k
=
_
4
3
_
k
, then lim
k
a
k
= lim
k
_
4
3
_
k
= , and the series diverges by the n-th term
test.
19. Since lim
k

a
k+1
a
k

= lim
k
(k + 1)/5
k+1
k/5
k
= lim
k
k + 1
5k
=
1
5
< 1,
the series is absolutely convergent by the ratio test.
20. Since lim
k

a
k+1
a
k

= lim
k
(k + 1)
2
[2
(k+1)
]
2
k
2
(2
k
)
2
= lim
k
(k + 1)
2
k
2
4
=
1
4
< 1,
the series is absolutely convergent by the ratio test.
21. Since lim
k

a
k+1
a
k

= lim
k
1/(k + 1)!
1/k!
= lim
k
1
k + 1
= 0 < 1,
the series is absolutely convergent by the ratio test.
616 CHAPTER 9. SEQUENCES AND SERIES
22. Since lim
k

a
k+1
a
k

= lim
k
[(k + 1)!]
2
/(2k + 2)!
(k!)
2
/(2k)!
= lim
k
(k + 1)
2
(2k + 2)(2k + 1)
= lim
k
k
2
+ 2k + 1
4k
2
+ 6k + 2
=
1
4
< 1,
the series is absolutely convergent by the ratio test.
23. Since lim
k

a
k+1
a
k

= lim
k
(k + 1)!/100
k+1
k!/100
k
= lim
k
k + 1
100
= ,
the series is divergent by the ratio test.
24. Since
lim
k
a
k
= lim
k
5
2k3
10
k+2
= lim
k
5
3
10
2
_
5
2k
10
k
_
=
1
12500
lim
k
25
k
10
k
=
1
12500
lim
k
_
5
2
_
k
= ,
the series diverges by the n-th term test.
25. Apply the limit comparison test to

k=1
k
1 + k
2
with a
k
=
k
1 + k
2
and b
k
=
1
k
: lim
k
a
k
b
k
=
lim
k
k/(1 + k
2
)
1/k
= lim
k
k
2
1 + k
2
= 1. Since

k=1
b
k
diverges, the given series is not absolutely
convergent. Let f(x) =
x
1 + x
2
. Then f

(x) =
1 x
2
(1 + x
2
)
2
< 0 for x > 1 and a
k+1
< a
k
. Also,
lim
k
a
k
= lim
k
k
1 + k
2
= 0, so the series is conditionally convergent.
26. Since
k
1 + k
4
=
1
k
3
+ 1/k
<
1
k
3
, the series is absolutely convergent by comparison with the
p-series

k=1
1
k
3
.
27. Since cos k = (1)
k
and lim
k
(1)
k
is not 0, the series diverges by the n-th term test.
28. Since sin
_
2k + 1
2
_
= sin
_
k +

2
_
= (1)
k
, the series is

k=1
(1)
k

k + 1
. Apply the limit
comparison test to

k=1
1

k + 1
with a
k
=
1

k + 1
and b
k
=
1

k
: lim
k
a
k
b
k
= lim
k
1/

k + 1
1/

k
=
lim
k

k + 1
= 1. Since

k=1
b
k
is a p-series with p =
1
2
< 1, it is divergent and the given
series is not absolutely convergent. Now, a
k+1
=
1

k + 2
<
1

k + 1
= a
k
and lim
k
a
k
=
lim
k
1

k + 1
= 0, so the original series is conditionally convergent.
9.7. ALTERNATING SERIES 617
29. Apply the limit comparison test to

k=1
sin
_
1
k
_
with a
k
= sin
_
1
k
_
and b
k
=
1
k
:
lim
k
a
k
b
k
= lim
k
sin(1/k)
1/k
h
= lim
k
(1/k
2
) cos(1/k)
1/k
2
= lim
k
cos(1/k) = 1.
Since

k=1
b
k
diverges, the given series is not absolutely convergent. Let f(x) = sin
_
1
x
_
. Then
f

(x) =
_

1
x
2
_
cos
_
1
x
_
< 0 for x 1 and a
k+1
< a
k
. Since lim
k
a
k
= lim
k
sin
_
1
k
_
= 0,
the original series is conditionally convergent.
30. Apply the limit comparison test to

k=1
1
k
2
sin
_
1
k
_
with a
k
=
1
k
2
sin
_
1
k
_
and b
k
=
1
k
3
:
lim
k
a
k
b
k
= lim
k
[sin(1/k)]/k
2
1/k
3
= lim
k
sin(1/k)
1/k
h
= lim
k
(1/k
2
) cos(1/k)
1/k
2
lim
k
cos
_
1
k
_
= 1.
Since

k=1
b
k
is a p-series with p = 3 > 1, it converges and the original series is absolutely
convergent.
31. Since
1
k + 1

1
k
=
1
k
2
+ k
, the series can be written as

k=1
(1)
k+1
1
k
2
+ k
. Now,
1
k
2
+ k
<
1
k
2
, so the series is absolutely convergent by comparison with the p-series

k=1
1
k
2
.
32. Since

k + 1

k = (

k + 1

k)
_

k + 1 +

k + 1 +

k
_
=
1

k + 1 +

k
,
the series can be written as

k=1
(1)
k
1

k + 1 +

k
. Apply the limit comparison test to

k=1
1

k + 1 +

k
with a
k
=
1

k + 1 +

k
and b
k
=
1

k
:
lim
k
a
k
b
k
= lim
k
1/(

k + 1 +

k)
1/

k
= lim
k

k + 1 +

k
= lim
k
1
_
1 + 1/k + 1
=
1
2
> 0.
Since

k=1
b
k
is a p-series with p =
1
2
< 1, it diverges and the given series is not absolutely
convergent. Now let f(x) =

x + 1

x. Then f

(x) =
1
2

x + 1

1
2

x
< 0 for x 1
and a
k+1
< a
k
. Since lim
k
a
k
= lim
k
1

k + 1 +

k
= 0, the original series is conditionally
convergent.
618 CHAPTER 9. SEQUENCES AND SERIES
33. Since lim
k
_
2k
k + 50
_
k
= lim
k
_
2
1 + 50/k
_
k
= , the series diverges by the n-th term test.
34. Since lim
k
|a
k
|
1/k
= lim
k
_
6
3k
k
k
_
1/k
= lim
k
216
k
= 0 < 1, the series is absolutely convergent
by the root test.
35. We must have a
n+1
=
1
(2n + 1)!
< 0.000005. Taking n = 4 we have a
5
=
1
9!
0.000003 <
0.000005. Thus, S
4
=
1
1!

1
3!
+
1
5!

1
7!
0.84147 has the desired accuracy.
36. We must have a
n+1
=
1
(n + 1)!
< 0.0005. Taking n = 6 we have a
7
=
1
7!
0.0002 < 0.0005.
Thus, S
6
=
1
1!

1
2!
+
1
3!

1
4!
+
1
5!

1
6!
0.632 has the desired accuracy.
37. We must have a
n+1
=
1
(n + 1)
3
< 0.005. Taking n = 5 we have a
6
=
1
6
3
0.0046 < 0.005.
Thus, S
5
has the desired accuracy.
38. We must have a
n+1
=
1

n + 1
< 0.0005 =
1
2000
. Taking n+1 = (2000)
2
+1 = 4, 000, 001 we
have a
4,000,001
< 0.005. Thus, S
4,000,000
has the desired accuracy.
39. We must have a
n+1
=
1
4
n+1
< 0.001. Taking n = 4 we have a
5
=
1
4
5
0.00098 < 0.001.
Thus, S
4
= 1
1
4
2
+
1
4
3

1
4
4
0.9492 has the desired accuracy.
40. We must have a
n+1
=
n + 1
5
n+1
< 0.0001. Taking n = 6 we have a
7
=
7
5
7
0.00009 < 0.0001.
Thus, S
6
= 1
2
5
2
+
3
5
3

4
5
4
+
5
5
5

6
5
6
0.93882 has the desired accuracy.
41. The error will be less than a
101
=
1
101
0.009901.
42. The error will be less than a
7
=
1
7(2
7
)
0.00112.
43. This is not an alternating series since, for k = 1 to k = 5, the terms are positive, while for
k = 7 to k = 11, the terms are negative. Since |a
k+1
|
1
k
2
, the series is absolutely convergent
by comparison with the p-series

k=1
1
k
2
. Hence, the series is convergent.
44. This is not an alternating series since for k 6, the terms are positive. For k 7, the terms
alternate but do not satisfy |a
k+1
| |a
k
| since a
7
=
100 2
7
3
7
0.013 and a
8
=
100 + 2
8
3
8

0.054. Write

k=1
100 + (1)
k
2
k
3
k
=

k=1
_
100
3
k
+ (1)
k
_
2
3
_
k
_
and apply Theorem 9.3.5 in the
9.7. ALTERNATING SERIES 619
text. Since

k=1
100
3
k
and

k=1
(1)
k
_
2
3
_
k
are geometric series with |r| < 1, they both converge
and the original series is convergent.
45. This is not an alternating series. Since |a
k
| =
1
2
k1
,

k=1
|a
k
| is a geometric series with
r =
1
2
< 1, and the original series is absolutely convergent.
46. This is not an alternating series. Since |a
k
| =
1
k
2
,

k=1
|a
k
| is a p-series with p = 2 > 1, and
the original series is absolutely convergent.
47. The terms of the series do not satisfy |a
n+1
| |a
n
| since |a
5
| =
2
3
>
1
2
= |a
4
|. Grouping pairs
of terms, we obtain the harmonic series 1 +
1
2
+
1
3
+ . Thus, the sequence of partial sums
{S
2n
} is the same as the sequence of partial sums for the harmonic series. Since the latter
sequence diverges, so does {S
2n
}. Finally, if {S
2n
} diverges, so must {S
n
}. Thus, the original
series diverges.
48. This is not an alternating series. The sequence of partial sums S
2
, S
5
, S
9
, S
14
, S
20
, . . . is 1,
0, 1, 0, 1, . . . . Since the sequence diverges, so must {S
n
}. Thus, the original series diverges.
49. The terms do not approach 0, so the series diverges.
50. The terms of the series are all 0, so the series converges.
51. All terms of the series after the rst are 0, so the series converges.
52. All odd terms of the series are 1 or 1, so the terms do not approach 0 and the series diverges.
53. The statement is true because a positive-term series

a
k
is the same as the series of its terms
absolute values

|a
k
|. If this series is convergent, then it is also absolutely convergent, and
so, as stated in the discussion, its terms can be rearranged in any manner and the resulting
series will converge to the same number as the original series.
54. S can be written as S =

k=1
(1)
k+1
1
k
. When rearranged as shown in the exercise, the
resulting series is
1
2

1
4
+
1
6

1
8
+
1
10

1
12
+
1
14
, which can be written as

k=1
(1)
k+1
1
2k
.
Factoring, we get

k=1
(1)
k+1
1
2k
=

k=1
(1)
k+1
_
1
2
__
1
k
_
=
1
2

k=1
(1)
k+1
1
k
=
1
2
S.
620 CHAPTER 9. SEQUENCES AND SERIES
55. Let

n=1
a
n
= 1
1
2
+
1
3

1
4
+
1
5

1
6
+ , and from Problem 54 let

n=1
b
n
= 0 +
1
2
+ 0
1
4
+ 0 +
1
6
+ . Then by Theorem 9.3.5 in the text,
3
2
S = S +
1
2
S =

n=1
(a
n
+ b
n
) = 1 + 0 +
1
3

1
2
+
1
5
+ 0 +
1
7

1
4
+
= 1 +
1
3

1
2
+
1
5
+
1
7

1
4
+ .
56. Write the series in the form

k=0
(1)
k+1
1
3
k+(1)
k
. Applying the ratio test, we have
lim
k

a
k+1
a
k

= lim
k
1/3
k+1+(1)
k+1
1/3
k+(1)
k
= lim
k
1
3
1+(1)
k+1
(1)
k
= lim
k
1
3
1+2(1)
k+1
.
This limit does not exist, so the ratio test is inconclusive.
Applying the root test, we have
lim
k
|a
k
|
1/k
= lim
k
_
1
3
k+(1)
k
_
1/k
= lim
k
1
3
1+(1)
k
/k
=
1
3
< 1.
Thus, the series is absolutely convergent by the root test.
57. Since

k=1
a
k
is absolutely convergent, lim
n
a
k
= 0. Thus, for n suciently large, |a
n
| < 1
and a
2
n
< |a
n
|. Therefore,

k=1
a
2
k
converges by the comparison test.
58.

k=1
(1)
k1

k
converges (see Exercise 2), but

k=1
_
(1)
k1

k
_
2
=

k=1
1
k
is the harmonic series,
which diverges.
59. The alternating harmonic series converges (see Example 2 in this section) and the series
consisting of its terms squares,

k=1
1
x
2
, is a p-series with p = 2 > 1, which also converges.
60. The harmonic series

k=1
1
k
is divergent while the p-series with p = 2 > 1

k=1
1
k
2
converges.
61. e
x
sinx + e
2x
sin2x + e
3x
sin3x + can be written as

k=1
e
kx
sinkx, and for x > 0,
|e
kx
sinkx| |e
kx
| = (e
x
)
k
. Now,

k=1
(e
x
)
k
is a convergent geometric series since
9.8. POWER SERIES 621
e
x
< 1 for x > 0. Thus,

k=1
|e
kx
sinkx| converges by the direct comparison test for all
positive x, and so

k=1
e
kx
sinkx must also converge for all positive x.
9.8 Power Series
1. lim
n

a
n+1
a
n

= lim
n

x
n+1
/(n + 1)
x
n
/n

= lim
n
n
n + 1
|x| = |x|
The series is absolutely convergent on (1, 1). At x = 1, the series

k=1
1
k
is the harmonic
series which diverges. At x = 1, the series

k=1
(1)
k
k
converges by the alternating series test.
Thus, the given series converges on (1, 1].
2. lim
n

a
n+1
a
n

= lim
n

x
n+1
/(n + 1)
2
x
n
/n
2

= lim
n
_
n
n + 1
_
2
|x| = |x|
The series is absolutely convergent on (1, 1). At x = 1, the series

k=1
(1)
k
k
2
converges by
the alternating series test. At x = 1, the series

k=1
1
k
2
is a convergent p-series.. Thus, the
given series converges on [1, 1].
3. lim
n

a
n+1
a
n

= lim
n

2
n+1
x
n+1
/(n + 1)
2
n
x
n
/n

= lim
n
2n
n + 1
|x| = 2|x|
The series is absolutely convergent for 2|x| < 1 or |x| < 1/2. At x = 1/2, the series

k=1
(1)
k
k
converges by the alternating series test. At x = 1/2, the series

k=1
1
k
is the harmonic series
which diverges. Thus, the given series converges on [1/2, 1/2).
4. lim
n

a
n+1
a
n

= lim
n

5
n+1
x
n+1
/(n + 1)!
5
n
x
n
/n!

= lim
n
5
n + 1
= 0
The series is absolutely convergent on (, ).
5. lim
n

a
n+1
a
n

= lim
n

(x 3)
n+1
/(n + 1)
3
(x 3)
n
/n
3

= lim
n
_
n
n + 1
_
3
|x 3| = |x 3|
The series is absolutely convergent for |x 3| < 1 or on (2, 4). At x = 2, the series

k=1
(1)
k
k
3
converges by the alternating series test. At x = 4, the series

k=1
1
k
3
is a convergent p-series.
Thus, the given series converges on [2, 4].
622 CHAPTER 9. SEQUENCES AND SERIES
6. lim
n

a
n+1
a
n

= lim
n

(x 3)
n+1
/

n + 1
(x + 7)
n
/

= lim
n
_
n
n + 1
|x + 7| = |x + 7|
The series is absolutely convergent for |x+7| < 1 or on (8, 6). At x = 8, the series

k=1
(1)
k

k
converges by the alternating series test. At x = 6, the series

k=1
1

k
is a divergent p-series.
Thus, the given series converges on [8, 6).
7. lim
n

a
n+1
a
n

= lim
n

(x 5)
n+1
/10
n+1
(x 5)
n
/10
n

= lim
n
1
10
|x 5| =
1
10
|x 5|
The series is absolutely convergent for
1
10
|x 5| < 1, |x 5| < 10, or on (5, 15). At
x = 5, the series

k=1
(1)
k
(10)
k
10
k
=

k=1
1 diverges by the n-th term test. At x = 15, the
series

k=1
(1)
k
10
k
10
k
=

k=1
(1)
k
diverges by the n-th term test. Thus, the series converges
on (5, 15).
8. lim
n

a
n+1
a
n

= lim
n

(n + 1)(x 4)
n+1
/(n + 3)
2
n(x 4)
n
/(n + 2)
2

= lim
n
(n + 1)(n + 2)
2
n(n + 3)
2
|x 4| = |x 4|
The series is absolutely convergent for |x4| < 1 or on (3, 5). At x = 3, the series

k=1
(1)
k
k
(k + 2)
2
converges by the alternating series test. At x = 5, the series

k=1
k
(k + 2)
2
diverges by the
limit comparison test with

k=1
1k
k
. Thus, the series converges on [3, 5)..
9. lim
n

a
n+1
a
n

= lim
n

(n + 1)!2
n+1
x
n+1
n!2
n
x
n

= lim
n
2(n + 1)|x| = , x = 0
The series converges only at x = 0.
10. lim
n

a
n+1
a
n

= lim
n

nx
n+1
/(n + 1)
2(n+1)
(n 1)x
n
/n
2n

= lim
n
nn
2n
(n 1)(n + 1)2
2n+2
|x|
= lim
n
n
(n + 1)(n + 1)
2
_
n
n + 1
_
2n
|x| = lim
n
n
(n 1)(n + 1)
2

1
__
n + 1
n
_
n
_
2
|x|
= lim
n
n
(n 1)(n + 1)
2

1
[(1 + 1/n)
n
]
2
|x| = 0
1
e
2
|x| = 0
The series is convergent on (, ).
11. lim
n

a
n+1
a
n

= lim
n

(3x 1)
n+1
/[(n + 1)
2
+ (n + 1)]
(3x 1)
n
/(n
2
+ n)

= lim
n
n
2
+ n
n
2
+ 3n + 2
|3x1| = |3x1|
The series is absolutely convergent for |3x 1| < 1 or on (0, 2/3). At x = 0, the series
9.8. POWER SERIES 623

k=1
(1)
k
k
2
+ k
converges by the alternating series test. At x = 2/3, the series

k=1
1
k
2
+ k
con-
verges by comparison with the p-series

k=1
1
k
2
. Thus, the given series converges on [0, 2/3].
12. lim
n

a
n+1
a
n

= lim
n

(4x 5)
n+1
/3
n+1
(4x 5)
n
/3
n

= lim
n
1
3
|4x 5| =
1
3
|4x 5|
The series is absolutely convergent for
1
3
|4x 5| < 1, |4x 5| < 3, or on (1/2, 2). At x =
1/2, the series

k=1
(3)
k
3
k
=

k=0
(1)
k
diverges by the n-th term test. At x = 2, the series

k=0
3
k
3
k
=

k=0
1 diverges by the n-th term test. Thus, the given series converges on (1/2, 2).
13. lim
n

a
n+1
a
n

= lim
n

x
n+1
/ ln(n + 1)
x
n
/ lnn

= lim
n
lnn
ln(n + 1)
|x|
By LHopitals Rule, lim
n
lnn
ln(n + 1)
h
= lim
n
1/n
1/(n + 1)
= lim
n
n + 1
n
= 1. Thus, lim
n

a
n+1
a
n

=
|x|. The series is absolutely convergent on (1, 1). At x = 1, the series

k=2
(1)
k
lnk
converges
by the alternating series test. At x = 1, the series

k=2
1
lnk
diverges by comparison with

k=2
1
k
. Thus, the given series converges on [1, 1).
14. lim
n

a
n+1
a
n

= lim
n

x
n+1
/(n + 1) ln(n + 1)
x
n
/nlnn

= lim
n
n
n + 1

lnn
ln(n + 1)
|x|
By LHopitals Rule, lim
n
lnn
ln(n + 1)
h
= lim
n
1/n
1/(n + 1)
= lim
n
n + 1
n
= 1. Thus, lim
n

a
n+1
a
n

=
|x|. The series is absolutely convergent on (1, 1). At x = 1, the series

k=2
1
k lnk
diverges
by the integral test. At x = 1, the series

k=2
(1)
k
k lnk
converges by the alternating series test.
Thus, the given series converges on (1, 1].
15. lim
n

a
n+1
a
n

= lim
n

(n + 1)
2
(x + 7)
n+1
/3
2n+2
n
2
(x + 7)
n
/3
2n

= lim
n
1
9
_
n + 1
n
_
2
|3x + 7| =
1
9
|x + 7|
The series is absolutely convergent for
1
9
|x + 7| < 1, |x + 7| < 9, or on (16, 2). At x = 16,
the series

k=1
(9)
k
k
2
9
k
=

k=1
(1)
k
k
2
diverges by the n-th term test. At x = 2, the series
624 CHAPTER 9. SEQUENCES AND SERIES

k=1
9
k
k
2
9
k
=

k=1
k
2
diverges by the n-th term test. Thus, the given series converges on (16, 2).
16. lim
n

a
n+1
a
n

= lim
n

(n + 1)
3
2
4(n+1)
(x 1)
n+1
n
3
2
4n
(x 1)
n

= lim
n
16
_
n + 1
n
_
3
|x 1| = 16|x 1|
The series is absolutely convergent for 16|x 1| < 1, |x 1| < 1/16, or on (15/16, 17/16). At
x = 15/16, the series

k=1
k
3
2
4k
_

1
16
_
k
=

k=1
(1)
k
k
3
diverges by the n-th term test. At
x = 17/16, the series

k=1
k
3
2
4k
_
1
16
_
k
=

k=1
k
3
diverges by the n-th term test. Thus, the
given series converges on (15/16, 17/16).
17. Write the series as

k=1
_
32
75
_
k
x
k
. Then
lim
n

a
n+1
a
n

= lim
n

(35/75)
n+1
x
n+1
(32/75)
n
x
n

= lim
n

32
75
x

=
32
75
|x|.
The series is absolutely convergent for
32
75
|x| < 1, or on (75/32, 75/32). At x = 75/32 the
series

k=1
(1)
k
diverges by the n-th term test. At x = 75/32 the series

k=1
1 diverges by the
n-th term test. Thus, the given series converges on (75/32, 75/32)
18. lim
n

a
n+1
a
n

= lim
n

1000
n+1
x
n+1
/(n + 1)
n+1
1000
n
x
n
/n
n

= lim
n
1000
n + 1
_
n
n + 1
_
n
|x|
= lim
n
1000n + 1
_
1
e
_
|x| = 0
The series is absolutely convergent on (, ).
19. lim
n

a
n+1
a
n

= lim
n

3
n+1
(x 1)
n+1
/(n + 2)(n + 3)
3
n
(x 1)
n
/(n + 1)(n + 2)

= lim
n
3
_
n + 1
n + 3
_
|x 1| = 3|x 1|
The series is absolutely convergent for 3|x 1| < 1, |x 1| < 1/3, or on (2/3, 4/3). At x =
2/3, the series

k=0
(3)
k
(k + 1)(k + 2)
_

1
3
_
k
=

k=0
1
(k + 1)(k + 2)
converges by comparison with
the p-series

k=1
1
k
2
. At x = 4/3, the series

k=0
(3)
k
(k + 1)(k + 2)
_
1
3
_
k
=

k=0
(1)
k
(k + 1)(k + 2)
converges by the alternating series test. Thus, the given series converges on [2/3, 4/3].
20. lim
n

a
n+1
a
n

= lim
n

3
n+1
(x + 5)
n+1
/2
n+1
(n + 1)(n + 2)
3
n
(x + 5)
n
/2
n
(n + 1)(n + 2)

= lim
n
3
2
_
n
n + 2
_
|x+5| =
3
2
|x+
5|
The series is absolutely convergent for
3
2
|x + 5| < 1, |x + 5| < 2/3, or on (17/3, 13/3).
At x = 17/3, the series

k=1
3
k
(2)
k
k(k + 1)
_

2
3
_
k
=

k=1
1
k(k + 1)
converges by comparison
9.8. POWER SERIES 625
with the p-series

k=0
1
k
2
. At x = 13/3, the series

k=1
3
k
(2)
k
k(k + 1)
_
2
3
_
k
=

k=1
(1)
k
k(k + 1)
converges by the alternating series test. Thus, the given series converges on [17/3, 13/3].
21. lim
n

a
n+1
a
n

= lim
n

(x 2)
n+1
/(n + 1)!(n + 1)!3
n+1
(x 2)
n
/n!n!3
n

= lim
n
1
3(n + 1)
2
|x 2| = 0
The series converges on (, ).
22. lim
n

a
n+1
a
n

= lim
n

(6 x)
n+2
/

2n + 3
(6 x)
n+1
/

2n + 1

= lim
n
_
2n + 1
2n + 3
|6 x| = |6 x|
The series is absolutely convergent for |6x| < 1 or on (5, 7). At x = 5, the series

k=0
1

2k + 1
diverges by the limit comparison test with the p-series

k=0
1

k
. At x = 7, the series

k=0
(1)
k+1

2k + 1
converges by the alternating series test. Thus, the given series converges on
(5, 7].
23. lim
n

a
n+1
a
n

= lim
n

x
2(n+1)+1
/9
n+1
x
2n+1
/9
n

= lim
n
1
9
x
2
=
1
9
x
2
The series is absolutely convergent for
1
9
x
2
< 1 or on (3, 3). At x = 3 the series

k=0
(1)
k
(3)
diverges by the n-th term test. At x = 3 the series

k=0
(1)
k
3 diverges by the n-th term test.
Thus, the given series converges on (3, 3).
24. lim
n

a
n+1
a
n

= lim
n

5
n+1
x
2(n+1)
/(2n + 1)!
5
n
x
2n
/(2n)!

= lim
n
5x
2
(2n + 2)(2n + 1)
= 0
The series is absolutely convergent on (, ).
25. lim
n
|a
n
|
1/n
= lim
n

x
n
(lnn)
n

1/n
= lim
n
|x|
lnn
= 0
The series is absolutely convergent on (, ).
26. lim
n
|a
n
|
1/n
= lim
n
|(n + 1)
n
(x + 1)
n
|
1/n
= lim
n
(n + 1)|x + 1| = , x = 1
The series converges only at x = 1.
27. lim
n
|a
n
|
1/n
= lim
n

_
4
3
n
(x + 3)
n
_

1/n
= lim
n
4
3
|x + 3| =
4
3
|x + 3|
The series is absolutely convergent for
4
3
|x + 3| < 1, |x + 3| <
3
4
, or on (15/4, 9/4). At
x = 15/4, the series

k=1
_
4
3
_
k
_

3
4
_
k
=

k=1
(1)
k
is divergent by the n-th term test. At
626 CHAPTER 9. SEQUENCES AND SERIES
x = 9/4, the series

k=1
_
4
3
_
k
_
3
4
_
k
=

k=1
1
k
is divergent by the n-th term test. Thus, the
given series converges on (15/4, 9/4).
28. lim
n
|a
n
|
1/n
= lim
n

_
n
n + 1
_
n
2
(x e)
n

1/n
= lim
n
_
n
n + 1
_
n
|x e|
= lim
n
1
(1 + 1/n)
n
|x e| =
1
e
|x e|
The series is absolutely convergent for
1
e
|x e| < 1, |x e| < e, or on (0, 2e). At x = 0,
we have the series

k=1
_
k
k + 1
_
k
2
(e)
k
=

k=1
(1)
k
_
e
(1 + 1/k)
k
_
k
. We will now show that
(1 + 1/k)
k
< e. Using the denition of lnx, we have
ln(1 + 1/k) =
_
1+1/k
1
1
t
dt
_
1+1/k
1
dt since 1/2 1 for t 1
=
1
k
.
Thus, ln1+1/k) 1/k and since e
x
is an increasing function, (1+1/k) e
1/k
and (1+1/k)
k

e. Hence,
e
(1 + 1/k)
k
1 and the series

k=1
(1)
k
_
e
(1 + 1/k)
k
_
k
diverges by the n-th term
test. Similarly, at x = 2e the series

k=1
_
k
k + 1
_
k
2
=

k=1
_
e
(1 + 1/k)
k
_
k
diverges by the n-th
term test. Thus, the given series converges on (0, 2e).
29. lim
n

a
n+1
a
n

= lim
n

(n + 1)!(x/2)
n+1
1 3 5 (2n 1)(2n + 1)
n!(x/2)
n
1 3 5 (2n 1)

= lim
n
n + 1
2n + 1

x
2

=
1
4
|x|
The series has radius of convergence 4.
30. lim
n

a
n+1
a
n

= lim
n

1 3 5 (2n 1)(2n + 1)(x 1)


n+1
3
n+1
(x + 1)!
1 3 5 (2n 3)(2n 1)(x 1)
n
3
n
n!

= lim
n
2n 1
3(n + 1)
|x 1| =
2
3
|x 1|
The series has radius of convergence 3/2.
31.

k=1
_
1
x
_
k
is a geometric series with common ratio r = 1/x. It converges for |1/x| < 1 or
x > 1.
9.8. POWER SERIES 627
32. Applying the ratio test, lim
n

a
n+1
a
n

= lim
n

7
n+1
/x
2n+2
7
n
/x
2n

= lim
n
7
x
2
=
7
x
2
. The series con-
verges for 7/x
2
< 1, x
2
> 7, or (,

7)

7, ). At x =

7, the series

k=1
7
k
7
k
=

k=1
1
diverges by the n-th term test. Thus, the given series converges on (,

7)

7, ).
33. This is a geometric series with common ratio r =
x + 1
x
and will converge for

x + 1
x

< 1 or
|x + 1| < |x|. If x 0, this means that x + 1 < x which has no solution. For x < 0, |x| = x
and the inequality can be written as |x +1| < x or x < x +1 < x. Since x < x +1 is valid
for all x, we have x + 1 < x, 2x < 1, or x < 1/2. Thus, the given series converges on
(, 1/2).
34. This is a geometric series with common ratio r =
x
2x + 4
and will converge for

x
2x + 4

< 1
or |x| < |2x + 4|. If x 0, this means x < 2x + 4 which is true for all x 0. Thus, the series
converges on [0, ). If x < 0, |x| = x and the inequality can be written as x < |2x + 4|.
This is equivalent to 2x + 4 < x(x) or 2x + 4 > x. Solving for x, we obtain x < 4 or
x > 4/3. Therefore, the given series converges on (, 4)

(4/3, ).
35. Applying the root test, we have lim
n
|a
n
|
1/n
= lim
n

_
x
2
+ 2
6
_
n
2

1/n
= lim
n
_
x
2
+ 2
6
_
n
< 1.
Setting
_
x
2
+ 2
6
_
n
< 1, we obtain
x
2
+ 2
6
< 1 or x
2
< 4. Thus, the series converges on
(2, 2). At x = 2, the series

k=0
1
k
2
diverges by the n-th term test. Therefore, the given
series converges on (2, 2).
36. Applying the ratio test, we have
lim
n

a
n+1
a
n

= lim
n

(n + 1)!/[(n + 1)x]
n+1
n!/(nx)
n

= lim
n

(n + 1)n
n
(n + 1)
n+1
x

= lim
n

_
n
n + 1
_
n
1
x

= lim
n
1
(1 + 1/n)
n

1
|x|
=
1
e|x|
.
Setting 1/e|x| < 1, we obtain |x| > 1/e. Thus, the series converges on (, 1/e)

(1/e, ).
At x = 1/e and 1/e we obtain the series

k=1
k!
k
k
(1/e)
k
=

k=1
(1)
k
k!e
k
k
k
and

k=1
k!e
k
k
k
.
Letting a
k
=
k!e
k
k
k
, we will show that a
k+1
a
k
. First, compute
a
k+1
a
k
=
(k + 1)!e
k+1
/(k + 1)
k+1
k!e
k
/k
k
=
(k + 1)k
k
e
(k + 1)
k+1
=
e
_
k + 1
k
_
k
.
628 CHAPTER 9. SEQUENCES AND SERIES
Then a
k+1
=
e
(1 + 1/k)
k
a
k
. We will now show that (1 + 1/k)
k
< e. Using the denition of
lnx, we have
ln(1 + 1/k) =
_
1+1/k
1
1
t
dt
_
1+1/k
1
dt since 1/t 1 for t 1
=
1
k
.
Thus, ln(1+1/k) 1/k and since e
x
is an increasing function, (1+1/k) e
1/k
and (1+1/k)
k

e. Hence,
e
(1 + 1/k)
k
1 and a
k+1
=
e
(1 + 1/k)
k
a
k
a
k
. Therefore, the two series diverge
because their terms do not approach 0. The given series converges on (, 1/e)

(1/e, ).
37. This is a geometric series with common ratio r = x
x
and will converge for |e
x
| = e
x
< 1 or
x < 0. Thus, the series converges on (, 0).
38. Applying the ratio test, we have lim
n

a
n+1
a
n

= lim
n

(n + 1)!e
(n+1)x
2
n!e
nx
2

= lim
n
(n+1)e
x
2
=
. Thus, the series diverges for all x.
39. This is a geometric series with r =
2

3
sinx and will converge for

3
sinx

< 1 or | sinx| <

3/2. On [0, 2] this will be on [0, /3)

(2/3, 4/3)

(5/3, 2].
40. Since |
sinkx
k
2
|
1
k
2
, we see that

k=1
sinkx
k
2
converges for all x by comparison with the p-series

k=1
1
k
2
.
41. (a) lim
n
|a
n+1
|
|a
n
|
= lim
n
x
2(n+1)
x
2n

2
2n
(n!)
2
2
2(n+1)[(n+1)!]
2
= lim
n
x
2

1
4(n + 1)
2
= 0 < 1
for all x values. Therefore, the interval of
convergence is (, ) and hence the domain is (, ).
(b)
S
0
(x)
S
3
(x)
S
4
(x)
S
2
(x)
S
1
(x)
y
x
9.9. REPRESENTING FUNCTIONS BY POWER SERIES 629
(c)
J
0
(x)
y
x
9.9 Representing Functions by Power Series
In this section we will use the fact that the geometric series

k=0
ar
k
converges to
a
1 r
for r < 1.
1.
1
3 x
=
1
3

1
1
x
3
=
1
3

k=0
_
x
3
_
k
for |
x
3
| < 1 or x in (3, 3). At x = 3 and x = 3, the series
diverges by the n-th term test. Thus, the interval of convergence is (3, 3).
2.
1
4 + x
=
1
4

1
1 +
x
4
=
1
4

k=0
(1)
k
_
x
4
_
k
for |
x
4
| < 1 or x in (4, 4). At x = 4 and x = 4, the
series diverges by the n-th term test. Thus, the interval of convergence is (4, 4).
3.
1
2 + x
=

k=0
(1)
k
(2x)
k
for |2x| < 1 or x in
_

1
2
,
1
2
_
. At x =
1
2
and x =
1
2
, the series
diverges by the n-th term test. Thus, the interval of convergence is
_

1
2
,
1
2
_
.
4.
1
5 + 2x
=
1
5

1
1 +
2x
5
=
1
5

k=0
(1)
k
_
2x
5
_
k
for |
2x
5
| < 1 or x in
_

5
2
,
5
2
_
. At x =
5
2
and
x =
5
2
, the series diverges by the n-th term test. Thus, the interval of convergence is
_

5
2
,
5
2
_
.
5. Identify a = 1 and r = x
2
. Then,
1
1 + x
2
=

k=0
(x
2
)
k
=

k=0
(1)
k
x
2k
for x
2
< 1 or x in
(1, 1).
At x = 1 and x = 1, the series diverges by the n-th term test. Thus, the interval of
convergence is (1, 1).
630 CHAPTER 9. SEQUENCES AND SERIES
6. Using the result from Problem 5, we have
x
1 + x
2
= x
1
1 + x
2
= x

k=0
(1)
k
x
2k
=

k=0
(1)
k
x
2k+1
for x in (1, 1). At x = 1 and x = 1, the series diverges by the n-th term test. Thus, the
interval of convergence is (1, 1) .
7.
1
4 + x
2
=
1
4

1
1 +
_
x
2
_
2
=
1
4

k=0
(1)
k
_
x
2
_
2k
for |
_
x
2
_
2
| < 1 or x in (2, 2) . At x = 2 and
x = 2, the series diverges by the n-th term test. Thus, the interval of convergence is (2, 2) .
8.
4
4 x
2
=
1
1
_
x
2
_
2
=

k=0
_
x
2
_
2k
for |
_
x
2
_
2
| < 1 or x in (2, 2) . At x = 2 and x = 2, the
series diverges by the n-th term test. Thus, the interval of convergence is (2, 2) .
9.
1
(3 x)
2
=
d
dx
_
1
3 x
_
=
d
dx
_
1
3

k=0
_
x
3
_
k
_
=
1
3

k=1
k
_
x
3
_
k1
The integral of conver-
gence remains (3, 3).
10.
1
(1 + 2x)
2
=
d
dx
_
1
1 + 2x
_
=
d
dx
_

k=0
(1)
k
(2x)
k
_
=

k=1
k(1)
k
(2x)
k1
. The integral
of convergence remains
_

1
2
,
1
2
_
.
11.
1
(5 + 2x)
3
=
1
8
d
2
dx
2
_
1
5 + 2x
_
=
1
8
d
2
dx
2
_
1
5

k=0
(1)
k
_
2x
5
_
k
_
=
1
250

k=2
k(k 1)(1)
k
2x
5
k2
.
The integral of convergence remains
_

5
2
,
5
2
_
.
12.
1
(4 + x)
3
=
1
2
d
2
dx
2
_
1
4 + x
_
=
1
2
d
2
dx
2
_
1
4

k=0
(1)
k
_
x
4
_
k
_
=
1
128

k=2
k(k 1)(1)
k
x
4
k2
.
The integral of convergence remains (4, 4) .
13.
x
(1 + x
2
)
2
=
1
2
d
dx
_
1
1 + x
2
_
=
1
2
d
dx
_

k=0
(1)
k
x
2k
_
=
1
2

k=1
2k(1)
k
x
2k1
.
The integral of convergence remains (1, 1) .
9.9. REPRESENTING FUNCTIONS BY POWER SERIES 631
14.
1 x
2
(1 + x
2
)
2
=
d
dx
_
x
1 + x
2
_
=
d
dx
_

k=0
(1)
k
x
2k+1
_
=

k=0
(2k + 1)(1)
k
x
2k
.
The integral of convergence remains (1, 1) .
15. Using Problem 5,
tan
1
x =
_
x
0
1
1 + t
2
=
_
x
0

k=0
(1)
k
t
2k
dt =

k=0
(1)
k
_
x
0
t
2k
dt =

k=0
(1)
k
x
2k+1
2k + 1
for x in (1, 1). At x = 1 and x = 1, the series converges by the alternating series test.
Thus, the interval of convergence is [1, 1].
16. tan
1
_
x
2
_
= 2
_
1
4 + x
2
dx = 2
_
_

k=0
(1)
k
_
x
2
_
2k
_
dx
= 2

k=0
2(1)
k
_
x
2
_
2k+1
2k + 1
which converges for x in (2, 2). At x = 2, we have the series 2

k=0
2(1)
k
2k + 1
which is conver-
gent. At x = 2, we have the series 2

k=0
2(1)
k
(1)
2k+1
2k + 1
= 2

k=0
2(1)
3k+1
2k + 1
which is also
convergent. Thus, the interval of convergence is [2, 2].
17. Using Problem 5,
1
1 + x
2
=

k=0
(1)
k
x
2k
and
2x
1 + x
2
= 2x

k=0
(1)
k
x
2k
=

k=0
2(1)
k
x
2k+1
.
Then
ln(1 + x
2
) =
_
x
0
2t
1 + t
2
dt =
_
x
0

k=0
2(1)
k
t
2k+1
dt = 2

k=0
(1)
k
_
x
0
t
2k+1
dt
= 2

k=0
(1)
k
x
2k+2
2k + 2
=

k=0
(1)
k
k + 1
x
2k+2
for x
2
< 1 or x in (1, 1). At x = 1 and x = 1, the series converges by the alternating series
test. Thus, the interval of convergence is [1, 1].
632 CHAPTER 9. SEQUENCES AND SERIES
18. ln(5 + 2x) = 2
_
1
5 + 2x
dx = 2
_
_

k=0
(1)
k
_
2x
5
_
k
_
dx
= 2

k=0
5
2
(1)
k
_
2x
5
_
k+1
k + 1
which converges for x in
_

5
2
,
5
2
_
. At x =
5
2
, we have the series
2

k=0
5
2
(1)
k
(1)
k+1
k + 1
= 2

k=0
5
2
(1)
2k+1
k + 1
= 5

k=0
1
k + 1
which diverges. At At x =
5
2
, we have
the series 2

k=0
5
2
(1)
k
k + 1
= 5

k=0
1
k + 1
which converges. Thus, the interval of convergence is
_

5
2
,
5
2
_
.
19. Writing
1
4 + x
=
1
4

1
1 + x/4
, we have
ln(4 + x) ln4 =
_
x
0
dt
4 + t
=
_
x
0
1
4

dt
1 + t/4
=
1
4
_
x
0

k=0
(t/4)
k
dt
=
1
4

k=0
(1)
k
_
x
0
t
k
4
k
dt =
1
4

k=0
(1)
k
k + 1

x
k+1
4
k
.
Thus ln(4 + x) = ln4 +

k=0
(1)
k
(k + 1)4
k+1
x
k+1
. This series converges for |x/4| < 1, |x| < 4, or
on (4, 4). At x = 4, the series diverges since it is the negative harmonic series. At x = 4,
the series converges by the alternating series test. Thus, the interval of convergence is (4, 4].
20. ln
_
3 + x
3 x
_
= ln(3 + x) ln(3 x)
=
_
1
3 + x
dx +
_
1
3 x
dx
=
_
_
1
3

k=0
(1)
k
_
x
3
_
k
_
dx +
_
_
1
3

k=0

_
x
3
_
k
_
dx
=

k=0
(1)
k
_
x
3
_
k+1
k + 1
+

k=0
_
x
3
_
k+1
k + 1
=

k=0
(1 + (1)
k
)
_
x
3
_
k+1
k + 1
which converges for x in (3, 3). At x = 3, we have the series

k=0
(1 + (1)
k
)
k + 1
which diverges.
9.9. REPRESENTING FUNCTIONS BY POWER SERIES 633
At x = 3, we have the series

k=0
(1 + (1)
k
)(1)
k+1
k + 1
which also diverges. Thus, the interval
of convergence is (3, 3).
21.
1 x
1 + 2x
= (1 x)
_
1
1 + 2x
_
(1 x)

k=0
(1)
k
(2x)
k
=

k=0
(1)x
k
(2x)
k

1
2
(1)
k
(2x)
k+1
= 1 +
3
2

k=1
(1)x
k
(2x)
k
which converges for x in
_

1
2
,
1
2
_
and diverges at x =
1
2
and x =
1
2
. Thus, the interval of
convergence is
_

1
2
,
1
2
_
.
22.
3 x
1 x
= (3 x)
_
1
1 x
_
(3 x)

k=0
x
k
=

k=0
3x
k
x
k+1
= 3 +

k=1
3x
k
x
k
= 3 +

k=1
2x
k
which converges for x in (1, 1) and diverges at x = 1 and x = 1. Thus, the interval of
convergence is (1, 1) .
23.
x
2
(1 x)
3
= x
2

1
(1 + x)
3
= x
2

1
2
d
2
dx
2
_
1
(1 + x)
_
= x
2

1
2

k=2
k(k 1)(1)
k
x
k2
=
1
2

k=2
k(k 1)(1)
k
x
k
which converges for x in (1, 1) and diverges at x = 1 and x = 1. Thus, the interval of
convergence is (1, 1) .
24.
x
3
8 + 2x
= x
3

1
8

1
1 +
x
4
=
x
3
8

k=0
(1)
k
_
x
4
_
k
=
1
8

k=0
x
k+3
4
k
which converges for x in (4, 4) and diverges at x = 4 and x = 4. Thus, the interval of
convergence is (4, 4) .
634 CHAPTER 9. SEQUENCES AND SERIES
25. s ln(1 + x
2
) = x 2
_
x
1 + x
2
dx
= x 2
_
_

k=0
(1)
k
x
2k+1
_
dx
= x 2

k=0
(1)
k
x
2k+2
2k + 2
=

k=0
(1)
k
x
2k+3
k + 1
which converges for x in (1, 1). At x = 1, we have the series

k=0
(1)
3k+3
k + 1
which converges.
At x = 1, we have the series

k=0
(1)
k
k + 1
which converges. Thus, the interval of convergence is
[1, 1].
26. x
2
tan
1
(x) = x
2

_ _
1
1 + x
2
_
dx
= x
2

_
_

k=0
(1)
k
x
2k
_
dx
= x
2

k=0
(1)
k
x
2k+1
2k + 1
=

k=0
(1)
k
x
2k+3
2k + 1
which converges for x in (1, 1). At x = 1, we have the series

k=0
(1)
3k+3
2k + 1
which converges.
At x = 1, we have the series

k=0
(1)
k
2k + 1
which converges. Thus, the interval of convergence
is [1, 1].
27. Since tan
1
t =

k=0
(1)
k
t
2k+1
2k + 1
, we have
_
x
0
tan
1
tdt =
_
x
0
_

k=0
(1)
k
t
2k+1
2k + 1
_
dt
=

k=0
__
x
0
(1)
k
t
2k+1
2k + 1
dt
_
=

k=0
_
(1)
k
t
2k+2
(2k + 1)(2k + 2)
_

x
0
=

k=0
(1)
k
x
2k+2
(2k + 1)(2k + 2)
which converges for x in (1, 1). At x = 1, we have the series

k=0
(1)
3k+2
(2k + 1)(2k + 2)
which
9.9. REPRESENTING FUNCTIONS BY POWER SERIES 635
converges. At x = 1, we have the series

k=0
(1)
k
(2k + 1)(2k + 2)
which converges. Thus, the
interval of convergence is [1, 1].
28. Since ln(1 + t
2
) =

k=0
(1)
k
t
2k+2
k + 1
, we have
_
x
0
ln(1 + t
2
)dt =
_
x
0
_

k=0
(1)
k
t
2k+2
k + 1
_
dt
=

k=0
__
x
0
(1)
k
t
2k+2
k + 1
dt
_
=

k=0
_
(1)
k
t
2k+3
(k + 1)(2k + 3)
_

x
0
=

k=0
(1)
k
x
2k+3
(k + 1)(2k + 3)
which converges for x in (1, 1). At x = 1, we have the series

k=0
(1)
3k+3
(2k + 1)(2k + 3)
which
converges. At x = 1, we have the series

k=0
(1)
k
(2k + 1)(2k + 3)
which converges. Thus, the
interval of convergence is [1, 1].
29.
1
1 x
=
1
5 (x 6)
=
1
5

1
1 +
_
x+6
5
_
=
1
5

k=0
(1)
k
_
x 6
5
_
k
which converges for

x 6
5

< 1 or |x 6| < 5. Since the series diverges at x = 11 and x = 1,


the interval of convergence is (1, 11).
30.
1
x
=
1
2 + (x + 2)
=
1
2

1
1
_
x+2
2
_
=
1
2

k=0
(
_
x + 2
2
_
k
which converges for

x + 2
2

< 1 or |x+2| < 2. Since the series diverges at x = 4 and x = 0,


the interval of convergence is (4, 0).
636 CHAPTER 9. SEQUENCES AND SERIES
31.
x
2 + x
=
(x + 2) 2
x + 2
= 1
2
x + 2
= 1
2
1 + (x + 1)
= 1 2

k=0
(1)
k
(x + 1)
k
= 1 2

k=1
(1)
k
(x + 1)
k
= 1 + 2

k=0
(1)
k
(x + 1)
k+1
which converges for |x + 1| < 1. Since the series diverges at x = 2 and x = 0, the interval
of convergence is (2, 0).
32.
x 2
x 1
=
(x 1) 1
x 1
= 1
1
x 1
= 1
1
1 + (x 2)
= 1

k=0
(1)
k
(x 2)
k
which converges for |x 2| < 1. Since the series diverges at x = 1 and x = 3, the interval of
convergence is (1, 3).
33.
7x
x
2
+ x 12
=
4
x + 4
+
3
x 3
=
1
1 +
_
x
4
_
1
1
_
x
3
_
=

k=0
(1)
k
_
x
4
_
k

k=0
_
x
3
_
k
=

k=0
_
(1)
k
4
k

1
3
k
_
x
k
which converges for (3, 3). Since the series diverges at x = 3 and x = 3, the interval of
convergence is (3, 3).
34.
3
x
2
x 2
=
1
x 2

1
x + 2
=
1
2

1
1
_
x
2
_
1
1 + x
=
1
2

k=0
_
x
2
_
k

k=0
(1)
k
x
k
=

k=0
_
1
2
k+1
+ (1)
k
_
x
k
which converges for (1, 1). Since the series diverges at x = 1 and x = 1, the interval of
convergence is (1, 1).
9.9. REPRESENTING FUNCTIONS BY POWER SERIES 637
35.
1
(2 x)(1 x)
=
1
2 x

1
1 x
=
_
1
2

k=0
_
x
2
_
k
_

k=0
x
k
_
=
1
2
_
1 +
x
2
+
x
2
4
+
x
3
8
+
_
[1 + x + x
2
+ x
3
+ ]
=
1
2
+
3
4
x +
7
8
x
2
+
15
16
x
3
+
36.
x
(1 + 2x)(1 + x
2
)
=
1
1 + 2x

x
1 + x
2
=
_

k=0
(1)
k
(2x)
k
_

k=0
(1)
k
x
2k+1
_
=
_
1 2x + 4x
2
8x
3
+

[x x
3
+ x
5
]
= x 2x
2
+ 3x
3
10x
4
+
37. Writing f(x) =

k=1
(1)
k+1
x
k
k3
k
and applying the ratio test, we have
lim
n

a
n+1
a
n

= lim
n

x
n+1
/(n + 1)3
n+1
x
n
/n3
n

= lim
n
1
3
_
n
n + 1
_
|x| =
1
3
|x|.
The series is absolutely convergent for
1
3
|x| < 1, |x| < 3, or on (3, 3). At x = 3, the series
is divergent since it is the negative harmonic series. At x = 3, the series converges by the
alternating series test. Thus, the domain of the function is (3, 3].
38. Writing f(x) =

k=0
2
k
k!
x
k
and applying the ratio test, we have
lim
n

a
n+1
a
n

= lim
n

2
n+1
x
n+1
/(n + 1)!
2
n
x
n
/n!

= lim
n
2
n + 1
|x| = 0.
Thus, the series is absolutely convergent for all x and the domain of the function is (, ).
39. Using Example 7 in the text with x = 0.1,
ln1.1 = 0.1
(0.1)
2
2
+
(0.1)
3
3

(0.1)
4
4
+
(0.1)
5
5

0.1 0.005 + 0.00033 0.00003 + 0.0953.
40. Using Problem 15,
tan
1
(.2) =

k=0
(1)
k
(0.2)
2k+1
2k + 1
= 0.2
(0.2)
3
3
+
(0.2)
5
5

(0.2)
7
7
+
0.2 0.00267 + 0.00006 0.000002 + 0.1974.
638 CHAPTER 9. SEQUENCES AND SERIES
41. Identify a = 1 and r = x
3
. Then
1
1 + x
3
=

k=0
(x
3
)
k
=

k=0
(1)
k
x
3k
and
_
x
0
dt
1 + t
3
=
_
x
0

k=0
(1)
k
t
3k
dt =

k=0
(1)
k
_
x
0
t
3k
dt ==

k=0
(1)
k
x
3k+1
3k + 1
.
Letting x = 1/2, we have
_
1/2
0
dx
1 + x
3
=

k=0
(1)
k
(1/2)
3k+1
3k + 1
=

k=0
(1)
k
1
2
3k+1
(3k + 1)
=
1
2

1
2
4
(4)
+
1
2
7
(7)

1
2
11
(11)
+
0.5 0.01563 + 0.00112 0.00004 + 0.4854.
42. Identify a = 1 and r = x
4
. Then
1
1 + x
4
=

k=0
(x
4
)
k
=

k=0
(1)
k
x
4k
and
x
1 + x
4
= x

k=0
(1)
k
x
4k
=

k=0
(1)
k
x
4k+1
and
_
x
0
dt
1 + t
4
=
_
x
0

k=0
(1)
k
t
4k
dt =

k=0
(1)
k
_
x
0
t
4k
dt ==

k=0
(1)
k
x
4k+2
4k + 2
.
Letting x = 1/3, we have
_
1/3
0
dx
1 + x
4
=

k=0
(1)
k
(1/3)
4k+2
4k + 2
=

k=0
(1)
k
1
3
4k+2
(4k + 2)
=
1
3
2
(2)

1
3
6
(7)
+
1
3
10
(10)

0.05556 0.00023 + 0000002 0.0553.
43. Using Problem 15, xtan
1
x = x

k=0
(1)
k
x
2k+1
2k + 1
=

k=0
(1)
k
x
2k+2
2k + 1
, so
_
x
0
t tan
1
tdt =
_
x
0

0
(1)
k
t
2k+2
2k + 1
dt =

0
(1)
k
2k + 1
_
x
0
t
2k+2
dt =

0
(1)
k
2k + 1
x
2k+3
2k + 3
.
Letting x = 0.3, we have
_
0.3
0
xtan
1
xdx =

0
(1)
k
2k + 1
(0.3)
2k+3
2k + 3
=
(0.3)
3
1 3

(0.3)
5
3 5
+
(0.3)
7
5 7

0.0090 0.0016 + 0.00001 0.008.
9.9. REPRESENTING FUNCTIONS BY POWER SERIES 639
44. Using Problem 15, tan
1
x
2
=

k=0
(1)
k
(x
2
)
2k+1
2k + 1
=

k=0
(1)
k
x
4k+2
2k + 1
, so
_
x
0
tan
1
t
2
dt =
_
x
0

0
(1)
k
t
4k+2
2k + 1
dt =

0
(1)
k
2k + 1
_
x
0
t
4k+2
dt =

0
(1)
k
2k + 1
x
4k+3
4k + 3
.
Letting x = 1/2, we have
_
1/2
0
tan
1
x
2
dx =

0
(1)
k
2k + 1
(1/2)
4k+3
4k + 3
=

0
(1)
k
1
2
4k+3
(2k + 1)(4k + 3)
=
1
2
3
1 3

1
2
7
3 7
+
1
2
1
1 5 11

0.04167 0.00037 + 0.00001 0.0413.
45. Using tan
1
x =

0
(1)
k
x
2k+1
2k + 1
, we have

4
= tan
1
1 =

0
(1)
k
2k + 1
= 1
1
3
+
1
5

1
7
+
46. Using Theorem 9.7.2 in the text, we must have a
n+1
=
1
2(n + 1) + 1
=
1
2n + 3
< 0.00005 or
2n + 3 > 20, 000. This means n > 9998.5. Thus, it will require S
9999
to approximate /4 to
four decimal places.
47. y

k=1
(1)
k+1
x
k1
=

k=1
(1)
k
x
k1
y

k=2
(k 1)(1)
k+1
x
k2
(x + 1)y

k=2
(k 1)(1)
k+1
x
k1
+

k=2
(k 1)(1)
k+1
x
k2
=

k=1
k(1)
k+2
x
k
+

k=1
k(1)
k+2
x
k1
=

k=1
k(1)
k
x
k
+

k=1
(1)
k
x
k1
(x + 1)y

+ y

k=1
k(1)
k
x
k
+

k=1
k(1)
k
x
k1

k=1
(1)
k
x
k1
=

k=1
k(1)
k
x
k
+

k=1
(k 1)(1)
k
x
k1
=

k=1
k(1)
k
x
k
+

k=2
(k 1)(1)
k
x
k1
=

k=1
k(1)
k
x
k

k=1
k(1)
k
x
k
= 0
640 CHAPTER 9. SEQUENCES AND SERIES
48. Letting y = J
0
(x), we have
y

k=1
2k(1)
k
2
2k
(k!)
2
x
2k1
, y

k=1
2k(2k 1)(1)
k
2
2k
(k!)
2
x
2k2
.
xy

+ y

+ xy =

k=1
(2k)(2k 1)(1)
k
x
2k1
2
2k
(k!)
2
+

k=1
(2k)(1)
k
2
2k
(k!)
2
x
2k1
+

k=0
(1)
k
2
2k
(k!)
2
x
2k+1
=

k=1
(2k)
2
(1)
k
x
2k1
2
2k
(k!)
2
+

k=0
(1)
k
2
2k
(k!)
2
x
2k+1
=

k=1
(2k)
2
(1)
k
x
2k1
(2k)
2
2
2(k1)
[(k 1)!]
2
+

k=1
(1)
k1
2
2(k1)
[(k 1)!]
2
x
2k1
=

k=1
(1)
k
x
2k1
2
2(k1)
[(k 1)!]
2

k=1
(1)
k
x
2k1
2
2(k1)
[(k 1)!]
2
= 0
49. (a) f

(x) =

k=1
x
k1
(k 1)!
=

k=0
x
k
k!
= f(x)
(b) e
x
50. (a) f

(x) =

k=0
(1)
k
(2k)!
x
2k
,
f

(x) =

k=1
(1)
k
(2k 1)!
x
2k1
=

k=0
(1)
k+1
(2k + 1)!
x
2k+1
=

k=0
(1)
k
(2k + 1)!
x
2k+1
= f(x)
(b) f(x) = sinx and f(x) = cos x both satisfy f

(x) = f(x). Since sinx is an odd function


while cos x is an even function, the function represented is f(x) = sinx.
9.10. TAYLOR SERIES 641
9.10 Taylor Series
1.
f(x) =
1
2 x
, f(0) =
1
2
f

(x) =
1
(2 x)
2
, f

(0) =
1
2
2
f

(x) =
2
(2 x)
3
, f

(0) =
2
2
3
f

(x) =
2 3
(2 x)
4
, f

(0) =
3!
2
4
.
.
.
f
(k)
(x) =
k!
(2 x)
k+1
, f
k
(0) =
k!
2
k+1
The Maclaurin series is

k=0
k!/2
k+1
k!
x
k
=

k=0
1
2
k+1
x
k
.
2.
f(x) =
1
1 + 5x
, f(0) = 1
f

(x) =
5
(1 + 5x)
2
, f

(0) = 5
f

(x) =
5
2
2
(1 + 5x)
3
, f

(0) = 5
2
2
f

(x) =
5
3
2 3
(1 + 5x)
4
, f

(0) = 5
3
3!
.
.
.
f
(k)
(x) =
(1)
k
5
k
k!
(1 + 5x)
k+1
, f
k
(0) = (1)
k
5
k
k!
The Maclaurin series is

k=0
(1)
k
5
k
k!
k!
x
k
=

k=0
(1)
k
5
k
x
k
.
642 CHAPTER 9. SEQUENCES AND SERIES
3.
f(x) = ln(1 + x), f(0) = 0
f

(x) =
1
1 + x
, f

(0) = 1
f

(x) =
1
(1 + x)
2
, f

(0) = 1
f

(x) =
2
(1 + x)
3
, f

(0) = 2
f
(4)
(x) =
2 3
(1 + x)
4
, f
(4)
(0) = 3!
.
.
.
f
(k)
(x) = (1)
k1
(k 1)!
(1 + x)
k
, f
(k)
(0) = (1)
k1
(k 1)!
The Maclaurin series is

k=1
(1)
k1
(k 1)!
k!
x
k
=

k=1
(1)
k1
(k 1)!
k!
x
k
=

k=1
(1)
k1
k
x
k
.
4.
f(x) = ln(1 + 2x), f(0) = 0
f

(x) =
2
1 + 2x
, f

(0) = 2
f

(x) =
2
2
(1 + 2x)
2
, f

(0) = 2
2
f

(x) =
2
3
2
(1 + 2x)
3
, f

(0) = 2
3
2
f
(4)
(x) =
2
4
2 3
(1 + 2x)
4
, f
(4)
(0) = 2
4
3!
.
.
.
f
(k)
(x) = (1)
k1
2
k
(k 1)!
(1 + 2x)
k
, f
(k)
(0) = (1)
k1
2
k
(k 1)!
The Maclaurin series is

k=1
(1)
k1
2
k
(k 1)!
k!
x
k
=

k=1
(1)
k1
2
k
k
x
k
.
9.10. TAYLOR SERIES 643
5.
f(x) = sinx, f(0) = 0
f

(x) = cos x, f

(0) = 1
f

(x) = sinx, f

(0) = 0
f

(x) = cos x, f

(0) = 1
.
.
.
f
(2k+1)
(x) = (1)
k
cos x, f
(2k+1)
(0) = (1)
k
The Maclaurin series is

k=1
(1)
k
(2k + 1)!
x
2k+1
.
6.
f(x) = cos 2x, f(0) = 1
f

(x) = 2 sin2x, f

(0) = 0
f

(x) = 2
2
cos 2x, f

(0) = 2
2
f

(x) = 2
3
sin2x, f

(0) = 0
.
.
.
f
(2k)
(x) = (1)
k
2
2k
cos 2x, f
(2k)
(0) = (1)
k
2
2k
The Maclaurin series is

k=0
(1)
k
2
2k
(2k)!
x
2k
.
7.
f(x) = e
x
, f(0) = 1
f

(x) = e
x
, f

(0) = 1
.
.
.
f
(2k)
(x) = e
x
, f
(k)
(0) = 1
The Maclaurin series is

k=0
1
k!
x
k
.
8.
f(x) = e
x
, f(0) = 1
f

(x) = e
x
, f

(0) = 1
.
.
.
f
(k)
(x) = (1)
k
e
x
, f
(k)
(0) = (1)
k
The Maclaurin series is

k=0
(1)
k
k!
x
k
.
644 CHAPTER 9. SEQUENCES AND SERIES
9.
f(x) = sinhx, f(0) = 0
f

(x) = coshx, f

(0) = 1
f

(x) = sinhx, f

(0) = 0
.
.
.
f
(2k+1)
(x) = coshx, f
(2k+1)
(0) = 1
The Maclaurin series is

k=0
1
(2k + 1)!
x
2k+1
.
10.
f(x) = coshx, f(0) = 1
f

(x) = sinhx, f

(0) = 0
f

(x) = coshx, f

(0) = 1
.
.
.
f
(2k)
(x) = coshx, f
(2k)
(0) = 1
The Maclaurin series is

k=0
1
(2k)!
x
2k
.
11.
f(x) = tanx, f(0) = 0
f

(x) = sec
2
x = 1 + tan
2
x, f

(0) = 1
f

(x) = 2 tanx(1 + tan


2
x) = 2 tanx + 2 tan
3
x, f

(0) = 0
f

(x) = 2 + 8 tan
2
x + 6 tan
4
x, f

(0) = 2
f
(4)
(x) = 16 tanx + 40 tan
3
x + 24 tan
5
x, f
(4)
(0) = 0
f
(5)
(x) = 16 + 136 tan
2
x + 240 tan
4
x + 120 tan
6
x, f
(5)
(0) = 16
f
(6)
(x) = 276 tanx + 1246 tan
3
x + 1680 tan
5
x + 720 tan
7
x, f
(6)
(0) = 0
f
(7)
(x) = 276 + 4014 tan
2
x + 12, 138 tan
4
x + 13, 440 tan
6
x + 5040 tan
8
x, f
(7)
(0) = 276
The Maclaurin series is x +
2
3!
x
3
+
16
5!
x
5
+
272
7!
x
7
+ = x +
1
3
x
3
+
2
15
x
5
+
17
315
x
7
+
9.10. TAYLOR SERIES 645
12.
f(x) = sin
1
x, f(0) = 0
f

(x) = (1 x
2
)
1/2
, f

(0) = 1
f

(x) = x(1 x
2
)
3/2
, f

(0) = 0
f

(x) = 3x
2
(1 x
2
)
5/2
+ (1 x
2
)
3/2
, f

(0) = 1
f
(4)
(x) = 15x
3
(1 x
2
)
7/2
+ 9x(1 x
2
)
5/2
, f
(4)
(0) = 0
f
(5)
(x) = 105x
4
(1 x
2
)
9/2
+ 90x
2
(1 x
2
)
7/2
+ 9(1 x
2
)
5/2
, f
(5)
(0) = 9
f
(6)
(x) = 945x
5
(1 x
2
)
11/2
+ 1050x
3
(1 x
2
)
9/2
+ 225x(1 x
2
)
7/2
, f
(6)
(0) = 0
f
(7)
(x) = 10, 395x
6
(1 x
2
)
13/2
+ 14, 175x
4
(1 x
2
)
11/2
+ 4, 725x
2
(1 x
2
)
9/2
+ 225(1 x
2
)
7/2
,
f
7
(0) = 225
The Maclaurin series is x +
1
3!
x
3
+
9
5!
x
5
+
225
7!
x
7
+ .
13.
f(x) =
1
1 + x
, f(4) =
1
5
f

(x) =
1
(1 + x)
2
, f

(4) =
1
5
2
f

(x) =
2
(1 + x)
3
, f

(4) =
3
5
3
f

(x) =
2 3
(1 + x)
4
, f

(4) =
3!
5
4
.
.
.
f
(2k+1)
(x) =
(1)
k
k!
(1 + x)
k+1
, f
(2k+1)
(4) =
(1)
k
k!
5
k+1
The Taylor series is

k=0
(1)
k
k!/5
k+1
k!
(x 4)
k
=

k=0
(1)
k
5
k+1
(x 4)
k
.
646 CHAPTER 9. SEQUENCES AND SERIES
14.
f(x) = x
1/2
, f(1) = 1
f

(x) =
1
2
x
3/2
, f

(1) =
1
2
f

(x) =
3
2
2
x
5/2
, f

(1) =
3
2
2
f

(x) =
3 5
2
3
x
7/2
, f

(1) =
3 5
2
3
.
.
.
f
k
(x) =
(1)
k+1
1 3 5 (2k + 1)
2
k
x
(2k+2)/2
f
(k)
(1) =
(1)
k+1
1 3 5 (2k + 1)
2
k
The Taylor series is
1 +

k=0
(1)
k+1
1 3 5 (2k + 1)/2
k
k!
(x 1)
k
= 1 +

k=0
(1)
k+1
1 3 5 (2k 1)
2
k
k!
(x 1)
k
.
15.
f(x) =
1
x
, f(1) = 1
f

(x) =
1
x
2
, f

(1) = 1
f

(x) =
2
x
3
, f

(1) = 2
f

(x) =
6
x
4
, f

(1) = 6
.
.
.
f
n
(x) =
(1)
n
n!
x
n+1
f
(n)
(1) = (1)
n
n!
The Taylor series is

k=0
f
(
k)(1)
k!
(x 1)
k
=

k=0
(1)
k
k!
k!
(x 1)
k
=

k=0
(1)
k
(x 1)
k
9.10. TAYLOR SERIES 647
16.
f(x) =
1
x
, f(5) =
1
5
f

(x) =
1
x
2
, f

(5) =
1
25
f

(x) =
2
x
3
, f

(5) =
2
125
f

(x) =
6
x
4
, f

(5) =
6
125
.
.
.
f
n
(x) =
(1)
n
n!
x
n+1
f
(n)
(5) =
n!
5
n+1
The Taylor series is

k=0
f
(
k)(5)
k!
(x + 5)
k
=

k=0
1
5
k+1
(x + 5)
k
17.
f(x) = sinx, f(/4) =

2/2
f

(x) = cos x, f

(/4) =

2/2
f

(x) = sinx, f

(/4) =

2/2
f

(x) = cos x, f

(/4) =

2/2
The Taylor series is

2
2
+

2
2
_
x

4
_

2
2 2!
_
x

4
_
2

2
2 3!
_
x

4
_
3
+
18.
f(x) = sinx, f(/2) = 1
f

(x) = cos x, f

(/2) = 0
f

(x) = sinx, f

(/2) = 1
f

(x) = cos x, f

(/2) = 0
.
.
.
f
(2k)
(x) = (1)
k
sinx, f
(2k)
(/2) = (1)
k
The Taylor series is

k=0
(1)
k
(2k)!
_
x

2
_
2k
.
648 CHAPTER 9. SEQUENCES AND SERIES
19.
f(x) = cos x, f(/3) = 1/2
f

(x) = sinx, f

(/3) =

3/2
f

(x) = cos x, f

(/3) = 1/2
f

(x) = sinx, f

(/3) =

3/2
The Taylor series is
1
2

3
2
_
x

3
_

1
2 2!
_
x

3
_
2
+

3
2 3!
_
x

3
_
3
+ .
20.
f(x) = cos x, f(/6) =

3/2
f

(x) = sinx, f

(/6) = 1/2
f

(x) = cos x, f

(/6) =

3/2
f

(x) = sinx, f

(/6) = 1/2
The Taylor series is

3
2

1
2
_
x

6
_

3
2 2!
_
x

6
_
2
+
1
2 3!
_
x

6
_
3
+ .
21.
f(x) = e
x
, f(1) = e
f

(x) = e
x
, f

(1) = e
.
.
.
f
(k)
(x) = e
x
, f
(k)
(1) = e
The Taylor series is

k=0
e
k!
(x 1)
k
.
22.
f(x) = e
2x
, f(1) = e
1
f

(x) = 2e
2x
, f

(1) = 2e
1
f

(x) = 2
2
e
2x
, f

(1) = 2
2
e
1
f

(x) = 2
3
e
2x
, f

(1) = 2
3
e
1
.
.
.
f
(k)
(x) = (1)
k
2
k
e
2x
, f
(k)
(1) = (1)
k
2
k
e
1
The Taylor series is

k=0
(1
k
2
k
e
1
)
k!
_
x
1
2
_
k
.
9.10. TAYLOR SERIES 649
23.
f(x) = lnx, f(2) = 0
f

(x) =
1
x
, f

(2) =
1
2
f

(x) =
1
x
2
, f

(2) =
1
2
2
f

(x) =
2
x
3
, f

(2) =
2
2
3
f
(4)
(x) =
2 3
x
4
, f
(4)
(2) =
3!
2
4
.
.
.
f
(k)
(x) = (1)
k1
(k 1)!
x
k
, f
(k)
(2) = (1)
k1
(k 1)!
2
k
The Taylor series is
ln2 +

k=1
(1)
k1
(k 1)!/2
k
k!
(x 2)
k
= ln2 +

k=1
(1)
k1
k2
k
(x 2)
k
.
24.
f(x) = ln(x + 1), f(2) = ln3
f

(x) =
1
x + 1
, f

(2) =
1
3
f

(x) =
1
(x + 1)
2
, f

(2) =
1
3
2
f

(x) =
2
(x + 1)
3
, f

(2) =
2
3
3
f
(4)
(x) =
2 3
(x + 1)
4
, f
(4)
(2) =
3!
3
4
.
.
.
f
(k)
(x) = (1)
k1
(k 1)!
(x + 1)
k
, f
(k)
(2) = (1)
k1
(k 1)!
3
k
The Taylor series is ln3 +

k=1
(1)
k1
(k 1)!/3
k
k!
(x 2)
k
= ln3 +

k=1
(1)
k1
k3
k
(x 2)
k
.
25. Substituting x
2
for x in Problem 8, we have

k=0
(1)
k
k!
(x
2
)
k
=

k=0
(1)
k
k!
x
2k
.
26. Substituting 3x for x and mutiplying by x
2
in Problem 8, we have x
2

k=0
(1)
k
k!
(3x)
k
=

k=0
(1)
k
3
k
k!
x
k+2
.
650 CHAPTER 9. SEQUENCES AND SERIES
27. Multiplying by x in Example 3 in the text, we have x

k=0
(1)
k
(2k)!
x
2k
=

k=0
(1)
k
(2k)!
x
2k+1
.
28. Substituting x
3
for x in Problem 5, we have

k=0
(1)
k
(2k + 1)!
(x
3
)
6k+3
=

k=0
(1)
k
(2k + 1)!
x
6k+3
.
29. Substituting x for x in Problem 3, we have

k=1
(1)
k1
k
(x)
k
=

k=1
1
k
x
k
.
30. Using Problems 3 and 20 and the fact the ln
_
1 + x
1 x
_
= ln(1 + x) ln(1 x), we have

k=1
(1)
k1
k
x
k

k=1
1
k
x
k
=

k=1
(1)
k1
+ 1
k
x
k
= 2x+
2
3
x
3
+
2
5
x
5
+ =

k=1
2
2k 1
x
2k1
.
31. Using Problem 11 and the fact that sec
2
x =
d
dx
tanx, we have
d
dx
_
x +
1
3
x
3
+
2
15
x
5
+
17
315
x
7
+
_
= 1 + x
2
+
2
3
x
4
+
17
45
x
6
+ .
32. Using Problem 11 and the fact that ln(cos x) =
_
x
0
tantdt, we have

_
x
0
_
t +
2
3!
t
3
+
16
5!
t
5
+
276
7!
t
7
+
_
dt =
1
2
x
2

2
4!
x
4

16
6!
x
6

276
8!
x
8
.
33. lim
x0
x
3
x sinx
= lim
x0
x
3
x
_
x
x
3
6
+
x
5
120

_
= lim
x0
x
3
x
3
6

x
5
120
+
= lim
x0
1
1
6

x
2
120
+
=
1
1
6
= 6
34. lim
x0
1 + x e
x
1 cos x
= lim
x0
1 + x
_
1 + x +
x
2
2
+
x
3
6
+
x
4
24
+
_
1
_
1
x
2
2
+
x
4
24

_
= lim
x0

x
2
2

x
3
6

x
4
24

x
2
2

x
4
24
+
= lim
x0
1
x
3

x
2
12

1
x
2
12
=
1
1
= 1
9.10. TAYLOR SERIES 651
35. coshx =
e
x
+ e
x
2
=
_
1 + x +
x
2
2!
+
x
3
3!
_
+
_
1 x +
x
2
2!

x
3
3!
_
2
=
2
_
1 +
x
2
2!
+
x
4
4!
+
_
2
=

k=0
x
2k
(2k)!
36. sinhx =
e
x
e
x
2
=
_
1 + x +
x
2
2!
+
x
3
3!
_

_
1 x +
x
2
2!

x
3
3!
_
2
=
2
_
x +
x
3
4!
+
x
5
5!
+
_
2
=

k=0
x
2k+1
(2k + 1)!
37.
e
x
1 x
= e
x

_
1
1 x
_
=
_
1 + x +
x
2
2!
+
x
3
3!
+
x
4
4!
+
_

_
1 + x + x
2
+ x
3
+ x
4
+
_
= 1 + 2x +
5
2
x
2
+
8
3
x
3
+
65
24
x
4
+
38. e
x
sinx =
_
1 + x +
x
2
2!
+
x
3
3!
+
x
4
4!
_

_
x +
x
3
3!
+
x
5
5!
+
_
= x + x
2
+
x
3
3

x
5
30

39.
e
x
cos x
=
1 + x +
x
2
2
+
x
3
3!
+
1
x
2
2
+
x
4
4!

= 1 + x + x
2
+
2x
3
3
+
x
4
2
+
40. sec x =
1
cos x
=
1
1
x
2
2
+
x
4
4!

x
5
5!
+
= 1 +
x
2
2
+
5x
4
24
+
61x
6
720
+
277x
8
8064
+
652 CHAPTER 9. SEQUENCES AND SERIES
41.
_
1
0
e
x
3
dx =
_
1
0
1 + (x
2
) +
(x
2
)
2
2
+
(x
2
)
3
3!
+ dx
=
_
1
0
_
1 x
2
+
x
4
2

x
6
6
+
_
dx
=
_
x
x
3
3
+
x
5
10

x
7
42
_

1
0
= 1
1
3
+
1
10

1
42
+
42.
_
1
0
sinx
x
dx =
_
1
0
1
x
_
x
x
3
3!
+
x
5
5!

x
7
7!
+
_
dx
=
_
1
0
_
1
x
2
3!
+
x
4
5!

x
6
7!
+
_
dx
=
_
x
x
3
3 3!
+
x
5
5 5!

x
7
7 7!
_

1
0
= 1
1
3 3!
+
1
5 5!

1
7 7!
+
43. Using tan
1
x =

k=0
(1)
k
x
2k+1
2k + 1
, we have

4
= tan
1
1 =

k=0
(1)
k
2k + 1
= 1
1
3
+
1
5

1
7
+
44. Using e
x
= 1 + x +
x
2
2!
+
x
3
3!
+ , we have
e
1
= 1 1 +
1
2!

1
3!
+
1
4!

1
5!
+
=
1
2!

1
3!
+
1
4!

1
5!
+
45. Using cos x = 1
x
2
2!
+
x
4
4!

x
6
6!
+ , we have
cos = 1

2
2!
+

4
4!


6
6!
+
46. Using sinx = x
x
3
3!
+
x
5
5!

x
7
7!
+ , we have
sin =

3
3!
+

5
5!


7
7!
9.10. TAYLOR SERIES 653
47. Using Problem 17 and the fact that 46

0.802851456 radians, we have


sin46

2
2
+

2
2
_
0.802851456

4
_

2
4
_
0.802851456

4
_
2
0.719340424.
Now, for f(x) = sinx, f

(x) = cos x and |R


2
(x)| =
| cos c|
3!
|x /4|
3
<
|x /4|
3
3!
. Thus,
|R
2
(0.802851456)| =
|0.802851456 /4|
3
6
< 0.000001.
48. Using Problem 20 and the fact that 29

0.506145483 radians, we have


cos 29

3
2

1
2
_
0.506145483

6
_

3
4
_
0.506145483

6
_
2
0.874620147.
Now, for f(x) = cos x, f

(x) = sinx and |R


2
(x)| =
| sinc|
3!
|x /6|
3
<
|x /6|
3
3!
. Thus,
|R
2
(0.506145483)| <
|0.506145483 /6|
3
6
< 0.000001.
49. Using Problem 7, we have e
0.3
1 + 0.3 +
(0.3)
2
2
+
(0.3)
3
3!
+
(0.3)
4
4!
1.349837500. Now,
|R
4
(x)| =
e
c
5!
|x|
5
<
3|x|
5
5!
since 0 < c < 1 and e
c
< e < 3. Thus, |R
4
(0.3)| <
3|0.3|
5
5!
< 0.0001.
50. Using Problem 9, we have sinh(0.1) 0.1 +
(0.1)
3
3!
0.100166667. Now, for f(x) = sinhx,
f
(4)
(x) = sinhx and |R
3
(x)| =
| sinhc|
4!
|x|
4
<
(sinh1)|x|
4
4!
since 0 < c < 0.1 and sinhc <
sinh1 =
e e
1
2
=
e
2
1
2e
<
9 1
4
= 2. Thus, |R
3
(0.1)| <
2(0.1)
4
24
< 0.00001.
51. We use |f
(n+1)
(x)| =
_
| cos x|, n even
| sinx|, n odd
. Since | cos x| 1 and | sinx| 1, |R
n
(x)| =
|f
(n+1)
(c)|
(n + 1)!
|x|
n+1

|x|
n+1
(n + 1)!
and lim
n
|R
n
(x)| = 0. Thus, the series represents sinx for all x.
52. We use R
n
(x) =
e
c
(n + 1)!
|x|
n+1
, where c is between 0 and x. If x < 0, then e
c
< 1 and
|R
n
(x)| =
e
c
(n + 1)!
|x|
n+1
<
|x|
n+1
(n + 1)!
. If x > 0, then e
c
< e
x
and |R
n
(x)| =
e
c
(n + 1)!
|x|
n+1
<
e
x
|x|
n+1
(n + 1)!
. In either case, for x xed, lim
n
|R
n
(x)| = 0 and the series represents e
x
for all x.
53. We use R
n
(x) =
_

_
sinhc
(n + 1)!
x
n+1
, n odd
coshc
(n + 1)!
x
n+1
, n even
for c between 0 and x. Now, | sinhc| < | sinhx| and
coshc < coshx for c between 0 and x, so for n odd, |R
n
(x)| =
| sinhc|
(n + 1)!
|x|
n+1
<
| sinhx||x|
n+1
(n + 1)!
,
654 CHAPTER 9. SEQUENCES AND SERIES
and for n even, |R
n
(x)| =
coshc
(n + 1)!
|x|
n+1
<
(coshx)|x|
n+1
(n + 1)!
. Thus, lim
n
|R
n
(x)| = 0 and the
series represents sinhx for all x.
54. We use R
n
(x) =
_

_
coshc
(n + 1)!
x
n+1
, n odd
sinhc
(n + 1)!
x
n+1
, n even
for c between 0 and x. Now, | sinhc| < | sinhx|
and coshc < coshx for c between 0 and x, so for n even, |R
n
(x)| =
| sinhc|
(n + 1)!
|x|
n+1
<
| sinhx||x|
n+1
(n + 1)!
, and for n odd, |R
n
(x)| =
coshc
(n + 1)!
|x|
n+1
<
(coshx)|x|
n+1
(n + 1)!
. Thus, lim
n
|R
n
(x)| =
0 and the series represents coshx for all x.
55. (a) From Figure 9.10.3, L = Rx and sec x =
R + y
R
. This gives y = Rsec x R =
Rsec
_
L
R
_
R
(b) We need to nd the Maclaurin series for f(x) = sec x. We compute
f(x) = sec x, f(0) = 1
f

(x) = sec xtanx, f

(0) = 0
f

(x) = 2 sec
3
x sec x, f

(0) = 1
f

(x) = 6 sec
3
xtanx sec xtanx, f

(0) = 0
f
(4)
= 6 sec
5
x + 18 sec
3
tan
2
x sec
3
x sec xtanx, f
(4)
(0) = 5
Therefore, sec x = 1 +
x
2
2
+ .
Approximating sec x by 1 +
x
2
2
, we have
y = R
_
sec
_
L
R
_
1
_
= R
_
1 +
(L/R)
2
2
1
_
= R
_
L
2
2r
2
_
=
L
2
2R
(c) Using y
L
2
2R
, we have
y =
(5280)
2
2(4000)(5280)
ft 0.66 ft = 7.92 in
(d) Approximating sec x by 1 +
x
2
2
+
5
24
x
4
, we have
y = R
_
1 +
(L/R)
2
2
+
5
24
(L/R)
4
1
_
=
L
2
2R
+
5L
4
24R
3
.
9.10. TAYLOR SERIES 655
56. (a) Since lim
d
tanh
2d
L
= 1, for d large,
_
gL/2.
(b) f(x) = tanhx, f(0) = 0
f

(x) = sech
2
x = 1 tanh
2
x, f

(0) = 1
f

(x) = 2 tanhx(1 tanh


2
x) = 2 tanh
3
x 2 tanhx, f

(0) = 0
f

(x) = 6 tanh
2
x(1tanh
2
x)2(1tanh
2
x) = 6 tanh
4
x+8 tanh
2
x2, f

(0) = 2
f
(4)
(x) = 24 tanh
3
x(1 tanh
2
x) + 16 tanhx(1 tanh
2
x), f
(4)
(0) = 0
= 24 tanh
5
x 40 tanh
3
x + 16 tanhx,
f
(5)
(x) = 120 tanh
4
x(1tanh
2
x)120 tanh
2
x(1tanh
2
x)+16(1tanh
2
x), f
(5)
(0) =
16 The Maclaurin series is tanhx = x
2
6
x
3
+
16
120
x
5
= x
1
3
x
3
+
2
15
x
5
.
Now, for small d/L, tanh(2d/L) 2d/L and
_
(gL/2)(2d/L) =

gd.
57. sin
2
x = (sinx)(sinx)
=
_
x
x
3
3!
+
x
5
5!

x
7
7!
+
__
x
x
3
3!
+
x
5
5!

x
7
7!
+
_
= x
2

x
4
3
+
2x
6
45

Also,
sin
2
x = 1 cos
2
x 1 (cos x)(cos x)
= 1
_
1
x
2
2!
+
x
4
4!

__
1
x
2
2!
+
x
4
4!

_
= 1
_
1 x
2
+
x
4
3

2x
6
45
+
_
= x
2

x
4
3
+
2x
6
45

58. sinxcos x =
_
x
x
3
3!
+
x
5
5!

x
7
7!
+
__
1
x
2
2!
+
x
4
4!

_
= x
2x
3
3
+
2x
5
15

Also, using the result from Problem 57, we have
sinxcos x =
1
2
d
dx
[sin
2
x] =
1
2
d
dx
_
x
2

x
4
3
+
2x
6
45

_
=
1
2
_
2x
4
3
x
3
+
4
15
x
5

_
= x
2
3
x
3
+
2
15
x
5

59. Using e
x
= 1 + x +
x
2
2
+
x
3
3!
+ , we have
656 CHAPTER 9. SEQUENCES AND SERIES
(x + 1)
2
e
x
= (x + 1)
2
e
x+11
= e
1
(x + 1)
2
e
x+1
= e
1
(x + 1)
2
_
1 + (x + 1) +
(x + 1)
2
2
+
(x + 1)
3
3!
+
_
= e
1
_
(x + 1)
2
+ (x + 1)
3
+
(x + 1)
4
2
+
(x + 1)
5
3!
+
_
=

k=0
e
1
(x + 1)
k+2
k!
60. No, the function f(x) = cot x is undened at x = 0.
61. cos x is an even function while sinx is an odd function. From (18), (19), and (20), we see that
tan
1
x is an odd function, coshx is an even function, and sinhx is an odd function.
62. We have
f(x) = x
4
sinx
2
= x
4
_
(x
2
)
(x
2
)
3
3!
+
_
= x
6

x
10
3!
+
The coecient of x
10
should be
f
(10)
(0)
10!
. Therefore, f
(10)
(0) =
10!
3!
= 604, 800.
63.
9.11 Binomial Series
1. With r = 1/3, for |x| < 1,
3

1 + x = 1 +
1
3
x +
1
3
_
1
3
1
_
2!
x
2
+
1
3
_
1
3
1
_ _
1
3
2
_
3!
x
3
+
= 1 +
1
3
x
2
3
2
2!
x
2
+
2 5
3
3
3!
x
3

.
2. With r = 1/2, for |x| < 1,

1 + x = 1 +
1
2
(x) +
1
2
_
1
2
1
_
2!
(x)
2
+
1
2
_
1
2
1
_ _
1
2
2
_
3!
(x)
3
+
= 1
1
2
x
1
2
2
2!
x
2

3
2
2
3!
x
3
.
3. With r = 1/2, for |x/9| < 1 or |x| < 9,

9 x = 3
_
1 x/9 = 3
_
1 +
1
2
_

x
9
_
+
1
2
_
1
2
1
_
2!
_

x
9
_
2
+
1
2
_
1
2
1
_ _
1
2
2
_
3!
_

x
9
_
3
+
_
= 3
3
2 9
x
3 1
2
2
2! 9
2
x
2

3 1 3
2
3
3! 9
3
x
3
.
9.11. BINOMIAL SERIES 657
4. With r = 1/2, for |5x| < 1 or |x| < 1/5,
1

1 + 5x
= 1 +
_

1
2
_
(5x) +

1
2
_

1
2
1
_
2!
(5x)
2
+

1
2
_

1
2
1
_ _

1
2
2
_
3!
(5x)
3
+
= 1
5
2
x
3 5
2
2
2
2!
x
2

3 5 5
3
2
3
3!
x
3
.
5. With r = 1/2, for |x| < 1,
1

1 + x
2
= 1 +
_

1
2
_
(x
2
) +

1
2
_

1
2
1
_
2!
(x
2
)
2
+

1
2
_

1
2
1
_ _

1
2
2
_
3!
(x
2
)
3
+
= 1
1
2
x
3
2
2
2!
x
4

3 5
2
3
3!
x
6
.
6. With r = 1/3, for |x| < 1,
x
3

1 x
2
= x
_
1
1
3
(x
2
) +

1
3
_

1
3
1
_
2!
(x
2
)
2
+

1
3
_

1
3
1
_ _

1
3
2
_
3!
(x
2
)
3
+
_
= x +
1
3
x
3
+
4
3
2
2!
x
5
+
4 7
3
3
3!
x
6
+ .
7. With r = 3/2, for |x/4| < 1 or |x| < 4,
(4 + x)
3/2
= 8(1 + x/4)
3/2
= 8
_
1 +
3
2
_
x
4
_
+
3
2
_
3
2
1
_
2!
_
x
4
_
2
+
3
2
_
3
2
1
_ _
3
2
2
_
3!
_
x
4
_
3
+
_
= 8 +
8 3
2 4
x +
8 3 1
3
2
2! 4
2
x
2
+
8 3 1 (1)
2
3
3! 4
3
x
3
+ .
8. With r = 5/2, for |x| < 1,
x
_
(1 + x)
5
= x
_

_1
5
2
x +

5
2
_

5
2
1
_
2!
x
2
+

5
2
_

5
2
1
_ _

5
2
2
_
3!
x
3
+
_

_
= x
5
2
x
2
+
5 7
2
2
2!
x
3

5 7 9
2
3
3!
x
4
+
.
9. With r = 2, for |x/2| < 1 or |x| < 2,
x
(2 + x)
2
=
x
4
(1 + x/2)
2
=
x
4
_
1 2
_
x
2
_
+
2(2 1)
2!
_
x
2
_
2
+
2(2 1)(2 2)
3!
_
x
2
_
3
+
_
=
1
4
x
1
4
x
2
+
2 3
4 2! 2
2
x
3

2 3 4
4 3! 2
3
x
4
+
.
658 CHAPTER 9. SEQUENCES AND SERIES
10. With r = 3, for |x| < 1,
x
2
(1 x
2
)
3
= x
2
_
1 3(x)
2
+
3(3 1)
2!
((x)
2
)
2
+
3(3 1)(3 2)
3!
((x)
2
)
3
+
_
= x
2
+ 3x
4
+
3 4
2!
x
6
+
3 4 5
3!
x
8
+
.
11. See Problem 1. Since the series is alternating on (0, 1), by Theorem 9.7.2 the approximation
to the sum using S
2
= 1 +
x
3
is accurate within a
3
=
1
9
x
2
.
12. See Problem 5. Since the series is alternating on (1, 1), by Theorem 9.7.2 the approximation
to the sum using S
3
= 1
x
2
2
+
3
8
x
4
is accurate within a
4
=
3 5
2
3
3!
x
6
=
5
16
x
6
.
13. With r = 1/2,
sin
1
x =
_
x
0
(1 t
2
)
(1/2)
dt
=
_
x
0
_

_1
1
2
(t
2
) +

1
2
_

1
2
1
_
2!
(t
2
)
2
+

1
2
_

1
2
1
_ _

1
2
2
_
2!
(t
2
)
3
+
_

_dt
=
_
x
0
_
1 +
1
2
t
2
+
3
2
2
2!
t
4
+
3 5
2
3
3!
+
_
dt
= x +
1
2 3
x
3
+
3
2
2
2! 5
x
5
+
3 5
2
3
3! 7
x
7
+
= x +

k=1
1 3 5 (2k 1)
2
k
k!(2k + 1)
x
2k+1
.
14. (a) The equation of the ellipse can be written as y =
b
a

a
2
x
2
. Then y

=
bx
a

a
2
x
2
and
L =
_
a
0

1 +
b
2
x
2
a
2
(a
2
x
2
)
dx =
1
a
_
a
0

a
4
x
2
(a
2
b
2
)
a
2
x
2
dx.
Let k
2
a
2
= a
2
b
2
and x = a sin. The dx = a cos d and
L =
1
a
_
a
0

a
4
a
2
k
2
x
2
a
2
x
2
dx =
_
a
0

a
2
k
2
x
2
a
2
x
2
dx =
_
/2
0

a
2
k
2
a
2
sin
2

a
2
a
2
sin
2

a cos d
= a
_
/2
0

1 k
2
sin
2

1 sin
2

cos d = a
_
/2
0
_
1 k
2
sin
2
d.
9.11. BINOMIAL SERIES 659
(b) With r = 1/2,
a
_
/2
0
_
1 k
2
sin
2
d = a
_
/2
0
_
1 +
1
2
(k
2
sin
2
) +
1
2
_
1
2
1
_
2!
(k
2
sin
2
)
2
+
_
d
= a
_
/2
0
_
1
1
2
k
2
sin
2

1
8
k
4
sin
4

_
d
= a
_

1
2
k
2
_
1
2

1
4
sin2
_

1
8
k
4
_
3
8

1
4
sin2 +
1
32
sin4
_

_

/2
0
= a
_

2

1
2
k
2
_

4
_

1
8
k
4
_
3
16
_

_
= a

2

a
2


4

a
8

3
16
k
4
.
15. y

=
8d
l
2
x. Using the formula for arc length and r = 1/2, we have
s = 2
_
l/2
0
_
1 +
64d
2
l
4
x
2
dx = 2
_
l/2
0
_
1 +
1
2

64d
2
l
4
x
2
+
1
2
_
1
2
1
_
2!
_
64d
2
l
4
x
2
_
2
+
_
dx
= 2
_
l/2
0
_
1 +
32d
2
l
4
x
2

64
2
d
4
8l
8
4
+
_
dx = 2
_
x +
32d
2
3l
4
x
3

64
2
d
4
5 8l
8
x
5
+
_

l/2
0
= l +
8d
2
3l

32d
4
5l
3
+ .
16. (a)
_
0.2
0
_
1 + x
3
dx =
_
0.2
0
_
1 +
1
2
x
3
+
1
2
_
1
2
1
_
2!
(x
3
)
2
+
_
dx
=
_
0.2
0
_
1 +
1
2
x
3

1
8
x
6
+
_
dx =
_
x +
1
8
x
4

1
56
x
7
+
_

0.2
0
= 0.2 +
(0.2)
4
8

(0.2)
7
56
+
After the rst term this is an alternating series. Since
(0.2)
7
56
< 0.0005, we have will
three decimal place accuracy
_
0.2
0

1 + x
3
dx 0.2 +
(0.2)
4
8
= 0.2002.
(b)
_
1/2
0
3
_
1 + x
4
dx =
_
1/2
0
_
1 +
1
3
x
4
+
1
3
_
1
3
1
_
2!
(x
4
)
2
+
_
dx
=
_
1/2
0
_
1 +
1
3
x
4

2
18
x
8
+
_
dx =
_
x +
1
15
x
5

2
18 9
x
9
+
_

1/2
0
=
1
2
+
1
15 32

2
18 9 2
9
+
660 CHAPTER 9. SEQUENCES AND SERIES
After the rst term this is an alternating series. Since
2
18 9 2
9
< 0.0005, we have with
three decimal place accuracy
_
1/2
0
3

x + x
4

1
2
+
1
15 32
0.502.
17. From Theorem 9.11.1 in the text with r = 1/2, we have
(1 2xr + r
2
)
1/2
= [1 + r(r 2x)]
1/2
= 1
1
2
r(r 2x) +

1
2
_

1
2
1
_
2!
r
2
(r 2x)
2
+
= 1
1
2
r
2
+ xr +
3
8
(r
4
4xr
3
+ 4x
2
r
2
) + = 1 + xr +
_
3
2
x
2

1
2
_
r
2
+
Thus, P
0
= 1 P
1
= x, and P
2
=
3
2
x
2

1
2
.
18. (a) f

(x) = r + r(r 1)x +


r(r 1)(r 2)
2!
x
2
+ +
r(r 1) (r n + 1)
(n 1)!
x
n1
+
xf

(x) = rx + r(r 1)x


2
+
r(r 1)(r 2)
2!
x
3
+ +
r(r 1) (r n + 1)
(n 1)!
x
n
+
(b) (n + 1)
r(r 1) (r n)
(n + 1)!
+ n
r(r 1) (r n + 1)
n!
=
r(r 1) (r n)
n!
+
nr(r 1) (r n + 1)
n!
=
r(r 1) (r n + 1)
n!
(r n + n) = r
r(r 1) (r n + 1)
n!
(c) f

(x) + xf

(x) = r + r(r 1)x + +


r(r 1) (r n + 1)
(n 1)!
x
n1
+
rx + r(r 1)x
2
+ +
r(r 1) (r n + 1)
(n 1)
x
n
+
=

k=1
r(r 1) (r k + 1)
(k 1)!
x
k1
+

j=1
r(r 1) (r j + 1)
(j 1)!
x
j
Let k = j + 1
=

j=0
r(r 1) (r j)
j!
x
j
+

j=1
r(r 1) (r j + 1)
(j 1)!
x
j
= r +

j=1
_
r(r 1) (r j)
j!
+
r(r 1) (r j + 1)
(j 1)!
_
x
j
= r +

j=1
_
(j + 1)
r(r 1) (r j)
(j + 1)!
+ j
r(r 1) (r j + 1)
j!
_
x
j
By part (b)
= r +

j=1
r
r(r 1) (r j + 1)
j!
x
j
= r
_
1 + r +
r(r 1)
2
x
2
+
r(r 1) (r n + 1)
n!
x
n
+
_
= rf(x)
CHAPTER 9 IN REVIEW 661
(d) Write the equation in the form (1 + x)
dy
dx
= ry. Then, separating variables, we have
dy
y
=
rdx
1 + x
; lny = r ln(1 + x) + c; y = e
r ln(1+x)+c
= e
c
e
ln(1+x)
r
= c
1
(1 + x)
r
or f(x) = c
1
(1 + x)
r
. Now, 1 + f(0) = c
1
so f(x) = (1 + x)
r
.
19. (1 + x)
1/2
= [2 + (x 1)]
1/2
=

2
_
1 +
x 1
2
_
1/2
=

2
_
1 +
_
1
2
_
x 1
2
+
_
1
2
_ _
1
2
1
_
2!
(x 1)
2
2
2
+
_
1
2
_ _
1
2
1
_ _
1
2
2
_
3!
(x 1)
4
2
4
+
_
=

2 +

2
2
2
(x 1)

2
2
2
2!
(x 1)
2
+

2 1 3
2
6
3!
(x 1)
3
+
20. (1 + x)
2
= [2 + (x + 1)]
2
= 2
2
_
1 +
x 1
2
_
2
=
1
4
_
1 2
x 1
2
+
2(2 1)
2!
(x 1)
2
2
2
+
2(2 1)(2 2)
3!
(x 1)
3
2
3
_
=
1
4

1
4
(x 1) +
3
2
2
2!
(x 1)
2

3 4
2
4
3!
(x 1)
3
+
Chapter 9 in Review
A. True/False
1. False; since |a
n
| =
n
2n + 1

1
2
, the series diverges by the n-th term test.
2. False; {(1)
n
} is bounded and divergent.
3. False; {(1)
n
/n} is convergent and not monotonic.
4. True; the rst three terms are 1/2, 100/6, and 1000/512.
5. True; since a
n+1
/a
n
1, a
n+1
a
n
and {a
n
} is a bounded monotonic sequence.
6. True
7. False; {1/n} converges, but

k=1
1/k is the divergent harmonic series.
8. True; 0.999 . . . =
9
10
_
1 +
1
10
+
1
100
+
1
1000
+
_
=
9
10
_
1
1
1
10
_
=
9
10
_
1
9
10
_
= 1
)
9. True; if this were false, the series would diverge by the n-th term test.
662 CHAPTER 9. SEQUENCES AND SERIES
10. False; consider the harmonic series
11. False; let a
k
= 1/k.
12. True
13. True
14. True
15. False; let a
k
= (1)
k
/k.
16. True
17. True
18. False; let b
k
= 1/k
2
and a
k
= a/k.
19. False; the ratio test is inconclusive in this case.
20. False;

k=0
k!x
k
converges only at x = 0.
21. False;

k=1
1
k
x
k
converges at x = 1 but is not absolutely convergent there.
22. True
23. False; let c
k
= 1/k.
24. False; at x = r the series diverges
25. False; the integral test simply indicates that the series converges.
26. True; the series is absolutely convergent
27. True; lnx is not dierentiable at x = 0.
28. True
29. True
30. True; f
(4)
= 4!c
4
= since c
4
= 0.
B. Fill in the Blanks
1. 20, 9, 4/5, 16
2. /2
3. 4, since a
5
= 1/10
5
= 0.00001 < 0.00005.
4. 12
CHAPTER 9 IN REVIEW 663
5.
n
9
;
22
9
0.nnnn. . . =
n
10
_
1 +
1
10
+
1
10
2
+ . . .
_
=
n
10
_
_
_
1
1
1
10
_
_
_ =
n
10
_
1
9
10
_
=
n
9
2.444 . . . = 2 + .044 . . . = 2 +
4
9
=
22
9
6. /4
7. e
x
8. 4
9. x < 5 and x > 5.
10.

k=0
(x
3
)
k
k!
=

k=0
(1)
k
x
3k
k!
11. The series converges absolutely for x in (1, 1). At x = 1, the series diverges. At x = 1, the
series converges conditionally. Thus, the interval of convergence is (1, 1].
12. 5
C. Exercises
1. Since
k
(k
2
+ 1)
2
<
1
k
3
, the series converges by comparison with the p-series

k=1
1
k
p
.
2. Since lim
n
1
1 + e
k
= 1, the series diverges by the n-th term test.
3. This is a geometric series with r =
1

< 1. Thus, the series converges.


4. This is a geometric series with r =
1
ln(2.5)
> 1. Thus, the series diverges.
5. Since lnk < k,

k lnk
k
4
+ 4
<
1
k
5/2
and the series converges by comparison with the p-series

k=1
1
k
5/2
6. Since

sink
k
3/2

<
1
k
3/2
, the series is absolutely convergent, and thus convergent, by comparison
with the p-series

k=1
1
k
3/2
.
664 CHAPTER 9. SEQUENCES AND SERIES
7. Since
k
3

k
6
4k
>
k
k
2
=
1
k
, the series diverges by comparison with the harmonic series.
8. The function f(x) = 1/x

lnx is continuous and decreasing on [2, ). Since


_

2
dx
x

lnx
= lim
t
_
t
2
dx
x

lnx
u = lnx, du =
1
x
dx
= lim
t
_
ln t
ln 2
du

u
= lim
t
2

ln t
ln 2
= lim
t
2
_

lnt

ln2
_
= ,
the series diverges by the integral test.
9. All of the odd-numbered terms of this series are 0. We may thus express the series as

k=1
2

2k
or

k=1
1

k
. This is a divergent p-series.
10. Since
lim
n
a
n+1
a
n
= lim
n
[(n + 1)
2
!]/[(n + 1)!]
(n
2
)!/(n!)
2
= lim
n
(n
2
+ 2n + 1)!
(n
2
)!
_
n!
(n + 1)!
_
2
= lim
n
(n
2
+ 2n + 1)(n
2
+ 2n) (n
2
+ 1)
_
1
n + 1
_
2
= lim
n
(n
2
+ 2n + 1)(n
2
+ 2n) (n
2
+ 1)
(n
2
+ 2n + 1)
= lim
n
(n
2
+ 2n) (n
2
+ 1) = ,
the series diverges by the ratio test.
11. Since
1
3k
2
+ 4k + 6
<
1
k
2
, the series converges by comparison with the p-series

k=1
1
k
2
.
12. a
n
= ln
_
3n
n + 1
_
ln(3) = 0. Thus, the series diverges by the n-th term test.
13.

k=1
(1)
k1
+ 3
(1.01)
k1
=

k=1
_

1
1.01
_
k1
+ 3

k=1
_
1
1.01
_
k1
=
1
1 (1/1.01)
+
3
1 (1/1.01)
=
1.01
1.01 + 1
+
3.03
1.01 1
=
101
201
+ 303 =
61, 004
201
14. Write a
k
=
1
k + 1

1
k + 6
. Then S
n
=
_
1
6

1
7
_
+
_
1
7

1
8
_
+ +
_
1
n + 5

1
n + 6
_
=
1
6

1
n + 6
and lim
n
n

k=1
1
k
2
+ 11k + 30
= lim
n
S
n
=
1
6
.
CHAPTER 9 IN REVIEW 665
15. lim
n

a
n+1
a
n

lim
n

3
n+1
x
n+1
/(n + 1)
3
3
n
x
n
/n
3

= lim
n
3
_
n
n + 1
_
3
|x| = 3|x|
The series is absolutely convergent for 3|x| < 1 or on (1/3, 1/3). At x = 1/3, the se-
ries

k=1
(1)
k
k
3
converges by the alternating series test. At x = 1/3, the series

k=1
1
k
3
is a
convergent p-series. Thus, the given series converges on [1/3, 1/3].
16. lim
n

a
n+1
a
n

lim
n

(n + 1)(2x 1)
n+1
/4
n+1
n(2x 1)
n
/4
n

= lim
n
1
4
_
n + 1
n
_
|2x 1| =
1
4
|2x 1|
The series is absolutely convergent for
1
4
|2x 1| < 1, |2x 1| < 4 or on (3/2, 5/2). At
x = 3/2, the series

k=1
(1)
k
k diverges by the n-th term test. At x = 5/2, the series

k=1
k
diverges by the n-th term test. Thus, the given series converges on (3/2, 5/2).
17. lim
n

a
n+1
a
n

lim
n

(n + 1)!(x + 5)
n+1
n!(x + 5)
n

= lim
n
(n+1)|x+5| = for x = 5. Thus, the series
converges only for x = 5.
18. lim
n

a
n+1
a
n

= lim
n

(2x)
n+1
/ ln(n + 1)
(2x)
n
/ lnn

= lim
n
lnn
ln(n + 1)
|2x|
h
= lim
n
1/n
1/(n + 1)
|2x|
= lim
n
n + 1
n
|2x| = |2x|
The series is absolutely convergent for |2x| < 1 or on (1/2, 1/2). At x = 1/2, the series

k=2
1
ln2
diverges by comparison with the harmonic series. Thus, the given series converges
on [1/2, 1/2).
19. lim
n

a
n+1
a
n

= lim
n

2 5 (3n + 2)x
n+1
3 7 (4n + 3)
2 5 (3n 1)x
n
3 7 (4n 1)

= lim
n
3n + 2
4n + 3
|x| =
3
4
|x
The series converges for
3
4
|x| < 1 or |x| <
4
3
. Thus, the radius of convergence is 4/3.
20. Applying the ratio test, we have lim
n

a
n+1
a
n

= lim
n

(cos x)
n+1
(cos x)
n

= lim
n
| cos x| = | cos x|.
Since | cos x| < 1 for x = k where k is an integer, the series converges for all x = k. When
x = k, the series is either

k=1
(1)
k
or

k=1
1, both of which diverge by the n-th term test.
Thus, the series converges on all intervals of the form (k, k + ), where k is an integer.
666 CHAPTER 9. SEQUENCES AND SERIES
21.
1

+
1

2
+
1

3
+ =
_
1 +
1

+
1

2
+
_
1
=
1
1
_
1

_ 1 =

1
1 =
1
1
22. The argument is invalid since both series diverge. This implies S = and hence S cannot
be subtracted from both sides of 2S = S 1.
23. Using a binomial series expansion,
1
3

1 + x
5
= (1 + x
5
)
1/3
= 1
1
3
x
5
+
_

1
3
_ _

1
3
1
_
2!
(x
5
)
2
+ = 1
1
3
x
5
+
2
9
x
10
.
24. Using a binomial series expansion,
x
2 x
=
2 x 2
2 x
= 1 +
2
2 x
= 1 +
_
1
x
2
_
1
= 1 +
_
1 + (1)
_

x
2
_
+
(1)(1)
2!
_

x
2
_
2
+
(1)(1 1)(1 2)
3!
_

x
2
_
3
+
_
=
1
2
x +
1
4
x
2
+
1
8
x
3
+ .
25. Using the Maclaurin series for sinx,
sinxcos x =
1
2
sin2x =
1
2
_
2x
(2x)
3
3!
+
(2x)
5
5!

_
= x
2
3
x
3
+
2
15
x
5
.
26. Using the Maclaurin series for e
t
,
_
x
0
e
t
2
dt =
_
x
0
_
1 + t
2
+
1
2
(t
2
)
2
+
_
dt =
_
x
0
_
1 + t
2
+
1
2
t
4
+
_
dt
=
_
1 +
1
3
t
3
+
1
10
t
5
+
_

x
0
= x +
1
3
x
3
+
1
10
x
5
+ .
27.
f(x) = cos x, f(/2) = 0
f

(x) = sinx, f

(/2) = 1
f

(x) = cos x, f

(/2) = 0
f

(x) = sinx, f

(/2) = 1
.
.
.
f
(2k+1)
(x) = (1)
k+1
sinx, f
(2k+1)
(/2) = (1)
k+1
The Taylor series is

k=0
(1)
k+1
(2k + 1)!
_
x

2
_
2k+1
.
CHAPTER 9 IN REVIEW 667
28. We use |f
(n+1)
(x)| =
_
2
n1
| sin(2x)|, n even
2
n1
| cos(2x)|, n odd
Since | cos(2x)| 1 and | sin(2x)| 1,
|R
n
(x)| =
|f
(n+1)
(c)|
(n + 1)!
|x|
n+1

2
n
|x|
n+1
(n + 1)!
Using the ratio test,
|a
n+1
|
a
n
=
2
n+1
|x|
n+2
2
n
|x|
n+1

(n + 1)!
(n + 2)!
=
2|x|
n + 2
.
which converges to 0 as n . Thus, the series found in Problem 55 represents sinxcos x
for all x.
29. 3
_
2
3
_
+ 2
_
2
3
_
+
4
3
_
2
3
_
+
8
9
_
2
3
_
+
16
27
_
2
3
_
+
= 3
_
2
3
_
+ 3
_
2
3
_
2
+ 3
_
2
3
_
3
+ 3
_
2
3
_
4
+ 3
_
2
3
_
5
+ =
2
1 2/3
= 6 million dollars.
30. (a) Solving 2P = P(1 + r)
n
for n, we obtain the doubling time n = ln2/ ln(1 + r).
(b) Since 0 < r < 1, ln(1 +r) = r r
2
/2 +r
3
/3 r. Then (ln2)/ ln(1 +r) (ln2)/r
0.69/r 70/100r.
(c) We want to solve 70/100r = ln2/ ln(1+r) or 7 ln(1+r) = 10r ln2 for r. Using ln(1+r)
r r
2
/2 + r
3
/3, this equation can be written as
7
3
r
3

7
2
r
2
+ (7 10 ln2)r = r
_
7
3
r
2

7
2
r + 7 10 ln2
_
= 0.
The quadratic formula gives r 1.4802 and r 0.0198. Since r < 1 we see that the Rule
of 70 gives the true doubling time for r 1.98%.

Potrebbero piacerti anche